257
www.BankExamsToday.com Reasoning Made Easy By Ramandeep Singh

Reasoning Made Easy - pdfstores.files.wordpress.com · REASONING MADE EASY 6 Geetika, Jasmine, Hema, Brinda, Ipsita and veena, once again not following any order. The following are

  • Upload
    others

  • View
    6

  • Download
    0

Embed Size (px)

Citation preview

Page 1: Reasoning Made Easy - pdfstores.files.wordpress.com · REASONING MADE EASY 6 Geetika, Jasmine, Hema, Brinda, Ipsita and veena, once again not following any order. The following are

www.BankExamsToday.comReasoning MadeEasyBy Ramandeep Singh

Page 2: Reasoning Made Easy - pdfstores.files.wordpress.com · REASONING MADE EASY 6 Geetika, Jasmine, Hema, Brinda, Ipsita and veena, once again not following any order. The following are

REASONING MADE EASY www.BankExamsToday.com

2

Index

Chapter 1 PuzzlesChapter 2 Input OutputChapter 3 Data SufficiencyChapter 4 Sitting ArrangementsChapter 5 SyllogismChapter 6 Blood DirectionsChapter 7 AnalogyChapter 8 Coded InequalityChapter 9 Coding DecodingChapter 10 Decision MakingChapter 11 Direction SenseChapter 12 Letter and Number SeriesChapter 13 Logical ArrangementChapter 14 Mathematical OperationsChapter 15 Odd Man Out SeriesChapter 16 Ranking Based problemsChapter 17 Arithmatic ReasoningChapter 18 ClassificationChapter 19 Dice ProblemChapter 20 Statement and ArgumentsChapter 21 Statement and AssumptionsChapter 22 Statement and Conclusions

Page 3: Reasoning Made Easy - pdfstores.files.wordpress.com · REASONING MADE EASY 6 Geetika, Jasmine, Hema, Brinda, Ipsita and veena, once again not following any order. The following are

REASONING MADE EASY www.BankExamsToday.com

3

Chapter-1

Puzzles

Directions (Q. 1-2) Study the following information carefully and answer thequestion given below:

In one floor of a building there are eight flats – A, B, C, D, E, F, G and H in tworows. One row of flats faces north and the other row faces south. D is exactlyopposite of F which faces North. Neither A nor C faces South. A is not at any endsand is second to the left of F. E is exactly opposite of B which faces South. G isthird to the right of D.

1. Which of the following pairs of flats facing south are at the two ends?a) GD b) GH c) BHd) Cannot be determined e) None of these

2. Which of the following pairs of flats facing North are at the two ends?a) EA b) EC c) CFd) Cannot be determined e) None of these

Directions (Q. 3-4) Study the following information to answer the givenquestions:

In a five letter English word (which may or may not be a meaningfulEnglish word), there are two letters between L and P. S is not placed immediatelynext to L. There is only one letter between S and A. S is towards the right of A. S isnot placed immediately next to E.

3. Which of the following is correct with respect to the word thus formed?a) E is at one of the extreme ends of the wordb) P is not placed immediately next to Ac) There are two letters between A and E in the word thus formedd) P is placed second to the right of Ee) None of correct

4. Which of the following words will be formed based on the givenconditions?

a) SPAEL b) PEALS c) LEAPSd) SEPAL e) LAPSE

Directions (Q. 5-9) Study the following information and answer the givenquestions.

Page 4: Reasoning Made Easy - pdfstores.files.wordpress.com · REASONING MADE EASY 6 Geetika, Jasmine, Hema, Brinda, Ipsita and veena, once again not following any order. The following are

REASONING MADE EASY www.BankExamsToday.com

4

Eight chemicals A, B, C, D, E, F, G and H, contained in eight differentbottles, are placed around a circular table in such a manner that the tap fixed to eachbottle is directed outward from the centre of the table. Each chemical is opf adifferent colour, viz Blue, Yellow, Orange, White, Green, Violet, Brown and Black,but not necessarily in the same order.

i) Chemical B is placed third to the right of Chemical D.ii) The Yellow chemical is placed on the immediate left of chemical B.iii) The colour of chemical B is not white.iv) Chemical F is placed fourth to the left of chemical A.v) Neither chemical F nor chemical A is an immediate neighbour of chemical

B.vi) The colour of chemical C is violet and is placed third to the left of the

Yellow chemical.vii) The Green chemical is placed second to the right of the violet chemical.viii) The Brown chemical is placed second to the right of chemical B.ix) H is the Orange chemical and is placed exactly between chemical C and

chemical A.x) The Blue chemical is placed second to the left of the Orange chemical.xi) Chemical G is placed third to the right of chemical C.

5. Which of the following chemicals is of White colour?a) D b) E c) Gd) F e) None of these

6. What is H’s position with respect to F?a) Third to the leftb) Second to the leftc) Third to the rightd) Second to the righte) Fourth to the left

7. How many chemical bottles are placed between chemical A and G (countedfrom G clockwise)?a) One b) Two c) Threed) Four e) None of these

8. Four of the following five are alike in a certain way and hence from agroup. Which of the folowing does not belong to that group?a) Violet-Brown b) Black-Green c) Orange-Whited) Yellow-Blue e) Green -Yellow

9. What is the colour of chemical B?a) Blue b) Black c) Yellowd) Can’t say e) None of these

Page 5: Reasoning Made Easy - pdfstores.files.wordpress.com · REASONING MADE EASY 6 Geetika, Jasmine, Hema, Brinda, Ipsita and veena, once again not following any order. The following are

REASONING MADE EASY www.BankExamsToday.com

5

Directions (Q. 10-14) Study the following information carefully and answer thequestion given below:

P, Q, R, S, T, V and W are seven friends working in a call centre. Each ofthem has different day offs in a week from Monday to Sunday not necessarily in thesame order. They work in three different shifts I, II and III with at least two of themin each shift.

R works in shift II and his day off is not Sunday. P’s day off is Tuesday andhe does not work in the same shift with either Q or W. None of those who work inshift I has day off either on Wednesday or on Friday. V workds with only T in shiftIII. S’s day off is Sunday. V’s day off is immediate next day of that of R’s day off.T’s day off is not on Wednesday. W’s day off is not on the previous day of P’s dayoff. S works in shift I. Q does not work in the same shift with R and his day off isnot on Thursday.

10. Which of the following is W’s day off?a) Tuesday b) Monday c) Saturdayd) Data inadequate e) None of these

11. Which of the following is R’s day off?a) Friday b) Thursday c) Tuesdayd) Wednesday e) None of these

12. Which of the following groups of friends work in shift II?a) RP b) RV c) QWSd) Data inadequate e) None of these

13. Which of the following is Q’s day off?a) Friday b) Wednesday c) Thursdayd) Monday e) None of these

14. Which of the following groups of friends work in shift I?a) RV b) RP c) QWSd) Data inadequate e) None of these

Directions (Q. 15-18) Read the following passage carefully and answer thequestion given below it.

Six friends Abhishek, Deepak, Mridul, Pritam, Ranjan and Salil marriedwithin a year in the months of February, April, July, September, November andDecember and in the cities of Ahmedabad, Bengaluru, Chennai, Delhi, Mumbai andKolkata but not necessarily following the above order. The brides names were

Page 6: Reasoning Made Easy - pdfstores.files.wordpress.com · REASONING MADE EASY 6 Geetika, Jasmine, Hema, Brinda, Ipsita and veena, once again not following any order. The following are

REASONING MADE EASY www.BankExamsToday.com

6

Geetika, Jasmine, Hema, Brinda, Ipsita and veena, once again not following anyorder. The following are some facts about their weddings.

(i) Mridul’s wedding took place in Chennai; however he was not m arriedto Geetika or Veena.

(ii) Abhishek’s wedding took place in Ahmedabad and ranjan’s in Delhihowever neither of them was married to Jasmine or Brinda.

(iii) The wedding in Kolkata took place in February.(iv) Hema’s wedding took place in April, but not in Ahmedabad.(v) Geetika and Ipsita got married in February and November and in

Chennai and Kolkata but not following the above order.(vi) Pritam visited Bengaluru and Kolkata only after his marriage in

December.(vii) Salil was married to Jasmine to September.

15. Hema’s husband isa) Abhishek b) Deepak c) Ranjand) Pritam e) Mridul

16. Deepak’s wedding took place ina) Bengaluru b) Mumbai c) Kolkatad) Delhi e) Chennai

17. In Mumbai, the wedding of one of the friends took place in the month ofa) April b) September c) Novemberd) December e) July

18. Salil’s wedding was held ina) Bengaluru b) Chennai c) Kolkatad) Delhi e) Mumbai

Directions (Q. 19-23) Read the following information carefully to answer thegiven questions.

There are five married couples in a family and there is a child to everycouple. Ages of children are 3, 4, 5, 6 and 9 years. Name of males are M, N, O, Pand Q. Name of children are A, B, C, D and E. Name of females are V, W, X, Yand Z. Name of M’s child is not C and E and he is not eldest or youngest. Age ofX’s child is 6 years and her husband is one among N, O and Q. Age of D is 3 yearsbut she is not a child of O. A’s age is multiple of 3 but she is not a child of M andV. W’s husband is O or Q. Age of Z’s child is 5 years but the name of child is not Bor E. V is wife of O.

19. What is the age of B.a) 4 b) 6 c) 5d) 9 e) None of these

Page 7: Reasoning Made Easy - pdfstores.files.wordpress.com · REASONING MADE EASY 6 Geetika, Jasmine, Hema, Brinda, Ipsita and veena, once again not following any order. The following are

REASONING MADE EASY www.BankExamsToday.com

7

20. What is the name of M’s wife?a) W b) X c) Yd) Z e) None of these

21. Name of Mother, farther and age of E?a) X, N and 6b) V, O and 9c) X, O and 6d) V, N and 9e) Can’t be determined

22. What is the name and the age of P’s child?a) B and 6b) E and 9c) C and 4d) C and 5e) None of these

23. What are the names of Q’s wife and his child?a) W and Db) X and Ac) W and Cd) X and De) None of these

Directions (Q. 24-27) Read the following information carefully and answer thequestions which follow.

Five friends A, B, C, D, and E are working in 5 different departments M, N,O, P and Q and they earn different salaries i.e. 10,000, 15,000, 20,000, 25,000 and30,000 and they all are of different ages i.e. 24, 26, 28, 30 and 32 years. These allinformations are not necessarily in the same order.

B works in department M and earns more than 20,000. Person who is 28years old works in department Q. 32 year’s old person earns at least 20,000.

The person who is 26 years old earns 25,000. A earns 15,000, but does notwork in department N or P.

Person who is 30 years old earns highest salary but does not work in department Mand N. E does not work in department P or Q, and his age is not 32. The salary of Dis less than 20,000.

24. Who works in department N?a) B b) C c) Dd) E e) Can’t be determined

Page 8: Reasoning Made Easy - pdfstores.files.wordpress.com · REASONING MADE EASY 6 Geetika, Jasmine, Hema, Brinda, Ipsita and veena, once again not following any order. The following are

REASONING MADE EASY www.BankExamsToday.com

8

25. If the name of the person represents its salary then which of the following istrue?a) A + B = Cb) C + D = Bc) D + E = Bd) A + D = Ee) None of these

26. Which of the following combination is definitely true?a) B-30 years-M-30,000b) D-24 years-N-10,000c) A-24 years-P-15,000d) E-30 years-O-30,000e) None of these

27. The person whose age is 30 works in which department?a) M b) N c) Od) P e) None of these

Directions (Q. 28-29) Read the following information carefully and answer thequestions which follow:

Education is the most important investment one makes in life. Higherstudies and specialization in certain fields call for additional financial support fromtime-to-time.

28. Which of the following can be inferred from the given information? (Aninference is something that is not directly stated but can be inferred fromthe given information)

a) People generally lack finances to pay for higher educationb) The rich and wealthy sections of our society are unwilling to spend money on theeducation of their childrenc) Higher education requires higher amount of money than primary educationd) Now-a-days even the poor and needy desire to educate their childrene) None of these

29. Which of the following statements is an assumption implicit in the abovegiven proposition? (An assumption is something supposed or taken forgranted)

a) Higher education need not much finances as it is subsidized by the governmentb) Education is one of the most essential parts of one’s lifec) Education is mostly neglected in our societyd) Specialization in education is most sought after thing todaye) None of these

Page 9: Reasoning Made Easy - pdfstores.files.wordpress.com · REASONING MADE EASY 6 Geetika, Jasmine, Hema, Brinda, Ipsita and veena, once again not following any order. The following are

REASONING MADE EASY www.BankExamsToday.com

9

Directions (Q. 30-34) Read the following information carefully and answer thequestions given below.

Six executives Akash, Bipasha, Deepak, Jyoti, Kartik and Priya have toadvertise four products, i.e. Soap, watches, computers and chocolates on 3 differentchannels i.e. go, one and spice either alone or in pairs. An executive can visit onlyone channel and advertise only one product. Not more than two executives canadvertise on a channel.(i) Bipasha and Jyoti both visit the same channel but advertise different product(ii) Akash who visits ‘Go’ advertises neither soap nor computers(iii) Kartik does not advertise chocolates(iv) No girl advertises soap(v) The two executives who advertise chocolates visit spice

30. Who advertises watches?a) Deepak b) Kartik c) Akashd) Priya e) None of these

31. Which channel does Kartik visit?a) Go b) Spice c) Oned) Cannot be determined e) None of these

32. Which of the following channel product pairs is definitely incorrect?a) Go-Watch b) One-Computer c) Go-Soapd) One-Soap e) One-Watch

33. What will Jyoti advertise?a) Chocolates b) Watches c) Computers or watchesd) Can’t be determined e) None of these

34. If Bipasha advertises computers which of the following must be true?a) Jyoti advertises soapb) Jyoti advertises watchesc) Kartik advertises computersd) Kartik works for spicee) None of these

Directions (Q. 35-40) Study the following information carefully and answer thefollowing questions.

An organisation decided to distribute seven types of laptops on week daysstarting from Monday to Sunday. The seven laptops to be distributed are of Sony,Acer, Asus, Lenovo, Dell, HP and HCL but non necessarily in the same order. Delllaptops are to be given on Saturday. Lenovo laptops are given just after Asuslaptops and Asus laptops are not given on Wednesday. HP laptops are given twodays after the distribution of Asus laptop. Acer laptops are distributed just after

Page 10: Reasoning Made Easy - pdfstores.files.wordpress.com · REASONING MADE EASY 6 Geetika, Jasmine, Hema, Brinda, Ipsita and veena, once again not following any order. The following are

REASONING MADE EASY www.BankExamsToday.com

10

Sony laptop. HCL laptops are distributed before Acer laptops. HP laptops aredistributed after Dell laptops.

35. On which day of the week Lenovo laptops are distributed?a) Tuesday b) Wednesday c) Fridayd) Sunday e) None of these

36. Which laptops are distributed just between the days when Lenovo and HPare distributed?a) Asus b) Dell c) Sonyd) HCL e) None of these

37. Which laptops are distributed on Wednesday?a) Acer b) Lenovo c) HCLd) HP e) None of these

38. On which day of the week Asus are distributed?a) Monday b) Sunday c) Tuesdayd) Wednesday e) None of these

39. If HCL is related with Tuesday, Sony is related with Wednesday; similarlyLenovo is related with which day?a) Thursday b) Friday c) Saturdayd) Sunday e) None of these

40. Which laptops are distributed in the middle of the week?a) Acer b) Asus c) Lenovod) HCL e) None of these

Directions (Q. 41-42) In the following questions below is given a statementfollowed by two courses of action I and II. On the basis of the information given inthe statement, you have to assume everything in the statement to be true, thendecide which of the suggested courses of action logically follows for pursuing.

Give Answer:a) Only I followb) Only II followsc) Either I or II followsd) Neither I nor II followse) Both I and II follows

41. Statement: Despite good economic progress of country, significantnumber of under nourished children has been observed in the rural parts ofthe country.Courses of action

Page 11: Reasoning Made Easy - pdfstores.files.wordpress.com · REASONING MADE EASY 6 Geetika, Jasmine, Hema, Brinda, Ipsita and veena, once again not following any order. The following are

REASONING MADE EASY www.BankExamsToday.com

11

I. Government should increase wealth tax and income tax and use thatmoney for up-liftment of the deprived classII. Government should introduce schemes like free meals in primaryschools and make primary education compulsory

42. Statement: The police department has come under a cloth with recentrevelations that atleast two senior police officials are suspected to have beeninvolved in the illegal sale of a large quantity of weapons from the statepolice amoury.Courses of actionI. A through investigation should be ordered by the State Government tobring out all those who are involved in the illegal sales of arm.II. State police armoury should be kept under Central Government’scontrol.

Directions (Q. 43-47) Study the following information and answer the followingquestions

A, B, C, D, E, G and I are seven friends who study in three differentstandards namely 5th, 6th and 7th such that not less than two friends study in the samestandard. Each friend also has a different favourite subject namely History, Civics,English, Marathi, Hindi, Maths and Economics but not necessarily in the sameorder.

A likes Maths and studies in the 5th standard with only one otherfriend who likes Marathi. I studies with two other friends. Both the friends whostudy with I like languages (Here, languages include only Hindi, Marathi andEnglish)

D studies in the 6th standard with only one person and does not like Civics.E studies with only one friend. The one who likes history does not study in 5th or 6th

standard. E does not like languages. C does not like English, Hindi or Civics.

43. Which combination represents E’s favourite subject and the standard in whichhe studies?a) Civic s and 7th

b) Economics and 5th

c) Civics and 6th

d) History and 7th

e) Economics and 7th

44. Which of the following is I’s favourite subject?a) History b) Civics c) Marathid) Either English or Marathi e) Either English or Hindi

45. Who amongst the following studeis in the 7th standard?a) G b) C c) Ed) D e) None of these

Page 12: Reasoning Made Easy - pdfstores.files.wordpress.com · REASONING MADE EASY 6 Geetika, Jasmine, Hema, Brinda, Ipsita and veena, once again not following any order. The following are

REASONING MADE EASY www.BankExamsToday.com

12

46. Which of the following combinations is definitely correct?a) I and Hindi b) G and English c) C and Marathid) B and Hindi e) E and Economics

47. Which of the following subjects does G like?a) Either Maths or Marathib) Either Hindi or Englishc) Either Hindi or Civicsd) Either Hindi or Marathie) Either Civics or Economics

Directions (Q. 48-50) Read the following information carefully and answer thegiven questions.

Despite repeated announcements that mobile phones were not allowed inthe examination hall, three stuidents were caught with their mobile phones.A. Mobile phones nowadays have a lot of features and it is easy to cheat with

their help.B. The invigilator must immediately confiscate the mobile phones and ask the

students to leave the exam hall immediately.C. Mobile phones are very expensive and leaving them in bags outside the

exam hall is not safe.D. There have been incidents where students who left the exam hall early stole

the mobile phones kept in the bags of the students who were writing theexam.

E. The school authorities must ask the students to leave their phones in thecustody of the invigilator before the exam in order to avoid thefts of mobilephones.

F. None of the other students were carrying their phones in the exam hall.

48. Which of the following among A, B, C, and D may be a strong argumentsin favour of the three s tudents who were caught with the mobile phone?a) Only A b) A and B c) C and Dd) Only C e) B and D

49. Which of the following among A, B, E and F may be the reason behind theschool making such announcements before the exam?a) Only B b) D and E c) Only Fd) Only A e) None of these

50. Which of the following among A, B, D and F can be an immediate courseof action for the invigilator?a) Only B b) A and D c) Only Ad) D and F e) Only F

Page 13: Reasoning Made Easy - pdfstores.files.wordpress.com · REASONING MADE EASY 6 Geetika, Jasmine, Hema, Brinda, Ipsita and veena, once again not following any order. The following are

REASONING MADE EASY www.BankExamsToday.com

13

Directions (Q. 51-55) In these questions, a relationship between differentelements is shown in the statements. The statements are followed by twoconclusions. Give answer

a) If only conclusion I is true.b) If only conclusion II is true.c) If either conclusion I or II is true.d) If neither conclusion I nor II is true.e) If both conclusions I and II are true.

(51-52):Statement: W > H = I > C = L < E

51. Conclusions: I. W > L II. E > H

52. Conclusions: I. E < W II. I > L

53. Statement: P = R > E < S = N > T

Conclusions: I. P > T II. N > R

54. Statement: C > B > L, Q = E > P = C

Conclusions: I. Q > B II. L < E

55. Statement: D < H = J < K > P > R

Conclusions: I. D = K II. K > D

Directions (Q. 56-60) Study the following information carefully and answer thegiven questions:

A man has six sons U, V, W, X, Y and Z, who stay in four metro cities.They work in different companies, viz Infosys, Tech Mahindra, Tata, Nike,Walmart and Titan but not necessarily in the same order. Z stays in Mumbai andworks with Nike. Those who stay in Kilkata work with Tech Mahindra andWalmart. One of the two Mumbaikars works with Titan. The one who works withInfosys lives in Chennai. Z does not live in Chennai. U works with Tata. V workswith Tech Mahindra. Y works with Walmart.

56. Who amoing the following works with Infosys?a) U b) W c) Xd) Y e) None of these

57. Which of the following pairs lives in Kolkata?

Page 14: Reasoning Made Easy - pdfstores.files.wordpress.com · REASONING MADE EASY 6 Geetika, Jasmine, Hema, Brinda, Ipsita and veena, once again not following any order. The following are

REASONING MADE EASY www.BankExamsToday.com

14

a) XY b) ZX c) VYd) WU e) None of these

58. Z works with which of the following companies?a) Walmart b) Titan c) Tatad) Cannot be determined e) None of these

59. In which of the following metros does W live?a) Delhi b) Chennai c) Kolkatad) Cannot be determined e) None of these

60. Which of the following combinations is true?a) V – Mumbai – Walmartb) Z – Kolkata – Walmartc) Y – Delhi – Titand) W – Chennai- Infosyse) None of these

Directions (Q. 61-65) Study the following information carefully and give theanswers.

Sixteen teams have been invited to participate in the ABC Gold Cup crickettournament. The tournament is conducted in two stages. In the first stage, the teamsare divided in to two groups. Each group consists of eight teams, with each teamplaying every other team in its group exactly once. At the end of the first stage, thetop four teams from each group advance to the second stage while the rest areeliminated. The second stage comprises of several rounds. A round involves onematch for each team. The winner of a match in a round advances to the next round;while the loser is eliminated the team that remains undefeated in the second stage isdeclared the winner and claims the gold cup.

The tournament rules are such that each match results in a winner and aloser with no possibility of a tie. In the first stage, a team earns one point for eachwin and no points for a loss. At the end of the first stage, teams in each group areranked on the basis of total points to determine the qualifiers advancing to the nextstage. Ties are resolved by a series of complex tie- breaking rules so that exactlyfour teams from each group advance to the next stage.

61. What is the total number of matches played in the tournament?a) 28 b) 55 c) 63d) 35 e) None of these

62. The minimum number of wins needed for a team in the first stage toguarantee its advancement to the next stage is:a) 5 b) 6 c) 7d) 4 e) None of these

Page 15: Reasoning Made Easy - pdfstores.files.wordpress.com · REASONING MADE EASY 6 Geetika, Jasmine, Hema, Brinda, Ipsita and veena, once again not following any order. The following are

REASONING MADE EASY www.BankExamsToday.com

15

63. What is the highest number of wins for a team in the first stage in spite ofwhich it would be eliminated at the end of first stage?a) 1 b) 2 c) 3d) 4 e) None of these

64. What is the number of rounds in the second stage of tournament?a) 1 b) 2 c) 3d) 4 e) None of these

65. Which of the following statements is true?a) The winner will have more wins than any other team in the tournament.b) At the end of the first stage, no team eliminated from the tournament willhave more wins than any of the team qualifying for the second stage.c) It is possible that the winner will have the same number of wins in theentire tournament as a team eliminated at the end of the first stage.d) The number of tems with exactly one win in the second stage of thetournament is 4.e) None of these

Directions (Q. 66-70) Study the following information carefully and answer thequestions given below.

Five friends P, Q, R, S and T are Musician, Architect, Doctor, Engineer andArtist by profssion and like White, Blue, Red, Yellow and Green colour but notnecessarily in that order.

The person whose hobby is dancing preferred lemonade to cola while otherspreferred cola to lemonade in beverages.

The four friends who took cola were P, the one who is an Engineer, theperson whose favourite colour is Green and t he o ne whose hobby is netsurfing.

S did not take lemonade and his favourite colour is White. Q’s favourite colour is Blue. He did not like lemonade. T’s hobby is not painting, reading or gardening. S clicks a picture of his friend who is an Engineer. The person whose favourite colour is Red likes painting and the person who

is artist like gardening. S is not a doctor. The person who is a doctor takes cola. The person who is

an Engineer likes Blue colour. The musician’s favourite colour is not yellow. R’s favourite colour is

Green.

66. Who among the following is a Doctor?a) R b) P c) Sd) Can’t say e) None of these

Page 16: Reasoning Made Easy - pdfstores.files.wordpress.com · REASONING MADE EASY 6 Geetika, Jasmine, Hema, Brinda, Ipsita and veena, once again not following any order. The following are

REASONING MADE EASY www.BankExamsToday.com

16

67. Q’s hobby isa) Reading b) Painting c) Gardeningd) Can’t say e) None of these

68. The person who likes Blue colour is a/ana) Architect b) Musician c) Engineerd) Can’t say e) None of these

69. Whose favourite colour is Yellow?a) T b) R c) The one who is an artistd) Can’t say e) None of these

70. Which of the following combinations is not correctly matched?a) T-Architect-Yellow-Dancing-Colab) R-Artist-Green-Gardening-Colac) Q-Engineer-Blue-Reading-Colad) P-Doctor-Red-Painting-Colae) None of these

Directions (Q. 71-73) Read the given information carefully and answer thequestioins that follow.

Independence Day is considered as a day of rejoicing. Thousands of heroesof independence struggle made the supreme sacrifice of their lives so that we canenjoy freedom. But see how we have misused the hard won freedom over the last 67years – exactly in the same way the racist representatives of the British, WinstonChurchill, predicted before India wrested independence from the British.Independence Day and Republic Day do not kindle patriotic fervour in a largesection of our society any longer. They have only been non-working days wherepeople sit back at h ome and enjoy with their families. They produce only totalrevulsion for our corrupt political class, self-serving bureaucracy, greedy businessempires and the completely undisciplined people. The gullible people of the countryhave been taken for a ride by the promises made by their leaders all these yearswhile they have been bleeding the nation white in scam after scam. To millions ofpatriotic Indians Independence Day will remain a sad day until all these parasitesare totally annihilated.

I. Communal harmony will be disturbed in the country.II. Caste system will be promoted.III. Terrorism activities will increase in the country.IV. The economy of the country will be affected.

71. Which of the following assumptions is implicit in the above passage?a) People of India lack in nationalism

Page 17: Reasoning Made Easy - pdfstores.files.wordpress.com · REASONING MADE EASY 6 Geetika, Jasmine, Hema, Brinda, Ipsita and veena, once again not following any order. The following are

REASONING MADE EASY www.BankExamsToday.com

17

b) Increasing corruption in India has ended the patriotic feelings among theIndiansc) Bureaucracy is the hurdle in national integrityd) The freedom movement of India was not public; it was confined to onlya few individuals.e) None of these

72. Which of the following will be the effect of lack of patriotism in India?a) Only I b) Only I and II c) Only IIId) Only III and IV e) Only I, II and IV

73. Which of the following statements are in line with the above passage?a) The independence of India was very hard-earned.b) People of India are not respecting the hard-earned freedom of India.c) Increasing corrcuption in India is c reating anger among the people.d) Independence day is just like a public holiday where people sit at homeand enjoye) All of these

Directions (Q. 74-78) Study the following information carefully and answer thegiven questions.

Six friends P, Q, R, S, T and U from six different areas, viz. Rohini,Indirapuram, Dwarka, Kaushambi, Munirka and Vaishali, go for shopping in sixdifferent shopping complexes, viz. Sahara, GIP, CSM, V3S, Shipra and SelectCitywalk, but not necessarily in the same order. Three different types of transportsare used by them, viz. Car, Bus and Metro, in such a manner that two persons usethe same type of transport but not necessarily in the same order.

P uses car and lives in Vaishali but he does not go to CSM and SelectCitywalk.

The person going to Shipra Complex uses car and the one going to GIP usesbus.

T uses the same type of transport as the person from Dwarka. CSM Complex is chosen by the person who uses bus. R is neither from Rohini nor from Munirka and he goes to Shipra Complex. One of the persons who use Metro goes to Sahara Complex. Q is from Kaushambi and goes to GIP. He does not use the same transport

as S. U is neither from Indirapuram nor from Munirka. Select Citywalk and GIP Complex are not visited by the persons who use

car. The persons from Kaushambi and Rohini are using the same type of

transport.

Page 18: Reasoning Made Easy - pdfstores.files.wordpress.com · REASONING MADE EASY 6 Geetika, Jasmine, Hema, Brinda, Ipsita and veena, once again not following any order. The following are

REASONING MADE EASY www.BankExamsToday.com

18

74. Who are using Metro?a) S and T b) U and S c) T and Pd) P and U e) None of these

75. Who is going to V, S Complex for shopping?a) U b) P c) Rd) S e) None of these

76. Who is using the same transport as U?a) P b) Q c) Td) R e) None of these

77. S belongs to which of the following places?a) Indirapuram b) Vaishali c) Munirkad) Dwarka e) None of these

78. R is using which of the following transports?a) Car b) Bus c) Metrod) Data inadequate e) None of these

Directions (Q. 79-83) Study the following information carefully and answer thegiven questions:

T, U, V, W, X, Y and Z seven persons belong to seven different families,viz. Punjabi, Rajasthani, Gujarati, Marathi, Bengali, Kannada and Malayali, but notnecessarily in the same order. All of them are travelling in a train in AC tier II. Theyoccupy two lower berths, three middle berths and two upper berths. V is a Punjabiand is not on the upper berths. The Gujarati is the only other person who occupiesthe same type of berth as V. U and Y are not on the middle berths and they areMalayali and Bengali respectively. T is a Marathi. Z is neithr a Kannada nor aGujarati. X occupies the same type of b erth as the Rajasthani.

79. Z belongs to which of the following families?a) Rajasthani b) Kannada c) Gujaratid) Bengali e) None of these

80. Which of the following pairs occupy the lower berth?a) U, V b) Y, U c) W, Vd) T, V e) None of these

81. Who belongs to the Gujarati family?a) Y b) W c) Xd) Z e) None of these

82. Which of the following combinations is true?a) W-Gujarati-Upper

Page 19: Reasoning Made Easy - pdfstores.files.wordpress.com · REASONING MADE EASY 6 Geetika, Jasmine, Hema, Brinda, Ipsita and veena, once again not following any order. The following are

REASONING MADE EASY www.BankExamsToday.com

19

b) Y-Bengali-Lowerc) X-Kannada-Lowerd) Z- Rajasthani-Middlee) None of these

83. Which of the following groups occupies the middle berths?a) Z, Y, U b) Z, X, V c) X, T, Zd) W, T, X e) None of these

Directions (Q. 84-88) Study the given information carefully and answer thegiven questions.

Seven plays – A, B, C, D, E, F and G are to be held on seven consecutivedays (starting on Monday and ending on Sunday) not necessarily in the same order.Only one play can be held on one day. Only two plays will be held after play G.Only three plays will be held between play B and play E. Play B will not be held onSunday. Play A will be held before play D and play C (not necessarily immediatelybefore). Play C will be held after play D (not necessarily immediate after)

84. Play D will be held on which day?a) Monday b) Tuesday c) Wednesdayd) Thursday e) Saturday

85. Which play will be held immediately after Play C?a) Play E b) Play F c) Play Bd) Play G e) None of these

86. Which play will be held on Monday?a) Play F b) Play B c) Play Ed) Play D e) Play A

87. If all seven plays are held in the alphabetical order of their names startingon Monday and ending on Sunday, the positions of how many will remainunchanged as compared to the original schedule?a) Three b) More than three c) Oned) None e) Two

88. Play F is related to Monday in a certain way based on the given schedule.Similarly, Play G is related to Thursday. In the same way, Play B is relatedto which of the following days?a) Wednesday b) Friday c) Tuesdayd) Saturday e) Sunday

Page 20: Reasoning Made Easy - pdfstores.files.wordpress.com · REASONING MADE EASY 6 Geetika, Jasmine, Hema, Brinda, Ipsita and veena, once again not following any order. The following are

REASONING MADE EASY www.BankExamsToday.com

20

Directions (Q. 89-93) Read the following information carefully and answer thequestions given below it.

(i) There are eight books Mathematics, Sanskrit, English, Physics, History,Geography, Hindi and Biology. (Not necessarily in the same order).

(ii) Geography’s book is fourth from the bottom and Sanskrit’s book isthird from the top.

(iii) There are three books between the books of English and Biology.(iv) Hind i book is at exact between the books of History and Biology.(v) The book of Physics is at just below the book of Mathematics.

89. Which book is at top?a) Mathematics b) Physics c) Biologyd) Cannot find e) None of these

90. How many books are taken between the books of History and English?a) Two b) Three c) Fourd) Cannot find e) None of these

91. The book of Physics is placed betweena) Biology-Mathematicsb) Geography-Biologyc) Mathematics-Sanskritd) Sanskrit-Biology

92. Which of the following book is placed at bottom?a) Physics b) Biology c) Hindid) Cannot find e) None of these

93. What is the position of the book of History from the top?a) Fifth b) Fourth c) Sixthd) Seventh e) None of these

Directions (Q. 94-96) On the basis of the following information, answer thequestions given below.

(i) Five persons lived in five houses out of six houses P, Q, R, S, T and U.All houses are in series from North to South. Main gate of each housewas facing west.

(ii) From North, Dhirendra’s house is the third from Sanjay’s house(iii) In North, Manoj’s house is at as much distance from Vijay’s house as in

South, Sanjay’s house is at from Dhirendra’s house.(iv) In South, Kunal’s house is just aftre Vinay’s house.

94. Which of the following lived at last in South?a) Kunal b) Vinay c) Dhirendra

Page 21: Reasoning Made Easy - pdfstores.files.wordpress.com · REASONING MADE EASY 6 Geetika, Jasmine, Hema, Brinda, Ipsita and veena, once again not following any order. The following are

REASONING MADE EASY www.BankExamsToday.com

21

d) Sanjay e) None of these

95. Which house was empty?a) P b) Q c) Rd) S or T e) None of these

96. Which of the following statement was not necessary?a) IV b) III c) IId) I e) None of these

Directions (Q. 97-101) these questions are based on the following information.Study it carefully to answer the questions.

Seven officers L, M, N, P, Q, R and S work in three different shifts I, II andIII with atleast two persons working I n each shift. Each one of them has a differentweekly off from Monday to Sunday not necessarily in the same order.

M works in second shift only with R whose weekly off is on Friday. Q’sweekly off is on the next day of L’s weekly off and both of them work I n differentshifts. P works in third shift and his weekly off is on Saturday. S has a weekly offon Monday and he works in first shift. The one who has a weekly off on Sundayworks in first shift. L and P do not work in the same shift; L’s weekly off is onTuesday.

97. Whose weekly off falls on Thursday?a) L b) N c) Qd) Cannot be determined e) None of these

98. Which of the following combinations of shift, person and weekly off isdefinitely correct?a) II, M, Sunday b) III, N, Sunday c) II, P, Sundayd) I, L, Tuesday e) None of these

99. Whose weekly off is on Sunday?a) L b) M c) Nd) Q e) None of these

100. On which day is Q’s weekly off?a) Tuesday b) Wednesday c) Thursdayd) Cannot be determined e) None of these

101. Which of the following group of officers work in shift I?a) L, N, S b) L, S c) N, Sd) L, P, Q e) None of these

Page 22: Reasoning Made Easy - pdfstores.files.wordpress.com · REASONING MADE EASY 6 Geetika, Jasmine, Hema, Brinda, Ipsita and veena, once again not following any order. The following are

REASONING MADE EASY www.BankExamsToday.com

22

102. In a certain code PARTICLE is written as USBQFMDJ, how isDOCUMENT written in that code?a) VDEPUONF b) VDPENFUQ c) VDPENFOUd) VDPEUOFN e) None of these

Directions (Q. 103-107): In these questions letters are to be coded by the digitsand symbols as per the scheme and conditions given below. In each question agroup of letters is given followed by four combinations of digits/symbolsnumbered a, b, c and d. The serial number of the combination which correctlyrepresents the letter group is your answer. If none of the combinations iscorrect your answer is e) i.e. None of these.

Letters: K E T J H I F A L U B M O R PDigit/Symbol Code: 3 7 % $ 4 * 1 9 8 6 # @ 2 5 ©

Conditions(i) If the first as well as the last letter is a vowel their codes are to be

swapped.(ii) If the first as well as the last letter is a consonant b oth are to be coded

by $.(iii) If the first letter is as vowel and the last letter is a consonant, t he vowel

is to be coded by ^ and the consonant is to be added by --.

103. TARIFMa) $95*1$ b) %95*1@ c)%95*1%d) @95*1@ e) None of the above

104. AJTKLUa) 9$%386 b) $$%38$ c)^$%38—d) --$%38^ e) None of these

105. ORBETHa) –5#7%^ b) ^5 #7%-- c)25#7%4d) 45#7%2 e) None of the above

106. KEOMPA--72@©^ b) ^72@©-- c)372@©9d) 972@©3 e) None of the above

107. IJLTPUa) *$8%©6 b) *$8%©* c)6$8%©6

Page 23: Reasoning Made Easy - pdfstores.files.wordpress.com · REASONING MADE EASY 6 Geetika, Jasmine, Hema, Brinda, Ipsita and veena, once again not following any order. The following are

REASONING MADE EASY www.BankExamsToday.com

23

d) 6$8%©* e) None of the above

Directions (Q. 108-112): Study the following information carefully to answerthese questions.

Seven friends F, H, J, K, L, M and P are working in different cities viz.Chennai, Mumbai, Bengaluru, Jaipur, Indore, Ahmedabad, and Hyderabad notnecessarily in the same order. Each one of them has a different profession viz.,C.A., Doctor, Professor, Engineer, Banker, Lawyer and Architect not necessarily inthe same order. H is a Lawyer and he works in Jaipur. K is a Doctor and he does notwork I n Chennai. The C.A. works in Mumbai. M works in Indore. Architect worksin Hyderabad. L is a B anker and he works in Bengaluru. F is an Engineer. J doesnot work in Mumbai.

108. Who is an architect?a) J b) P c) Kd) L e) None of these

109. Which of the following combinations of person, profession and city isdefinitely correct?a) F-Engineer-Bengalurub) K-Architect-Hyderabadc) L-Banker-Mumbaid) M-Professor-Jaipure) None of the above

110. Who is working in Mumbai?a) J b) K c) Md) P e) None of these

111. Engineer works in which city?a) Ahmedabad b) Chennai c) Mumbaid) Bengaluru e) None of the above

112. Who is a professor?a) P b) J c) Md) M or J e) None of these

113. In a certain code, DOWN is written as 5139 and NEAR is written as 9486.How is RODE written in that code?a) 6514 b) 6154 c) 9154d) 3154 e) None of these

114. In a certain code ORGANISE is written as BHSPDRHM. How isDOUBTFUL written in that code?

Page 24: Reasoning Made Easy - pdfstores.files.wordpress.com · REASONING MADE EASY 6 Geetika, Jasmine, Hema, Brinda, Ipsita and veena, once again not following any order. The following are

REASONING MADE EASY www.BankExamsToday.com

24

a) CVPEKTES b) CVPIMVGU c) ATNCKTESd) ATNCMVGU e) None of these

115. If it is possible to make only one meaningful English word with the first,the se cond, the fourth and the tenth letters of the word M A J E S T I C AL, which of the following will be the second letter of that word? If no suchword can be made, give ‘X’ as the an swer and if mor e than one such wordcan be made, give ‘Y ‘ is the answera) M b) E c) Ld) X e) Y

116. In a certain code language, ‘pit ne’ means ‘come here’, ‘ne ta ja’ means‘come and go’ and ‘ja sa re’ means ‘you and me’. What does ‘ta’ means inthat code language?a) come b) and c) hered) go e) Cannot be determined

117. ‘RT’ is related to ‘VX’ and ‘BD’ is related to ‘FH’, in the same way as‘KM’ is related toa) NP b) OR c) OQd) PR e) None of these

118. In a certain code GEAR is written as ‘5%9$’ and FIRM is written as‘3@$7’. How is FAME written in that code?a) 397% b) 3%97 c) 597%d) 5397 e) None of these

Directions (Q. 119-122) In each of the questions below are given threestatements followed by three conclusions numbered I, II and III. You have totake the given statements to be true even if they seem to be at variance fromcommonly known facts. Read all the conclusions and then decide which of thegiven conclusions logically follows from the given statements disregardingcommonly known facts.

119. Statements: Some books are trees.All trees are roads.All roads are wheels.

Conclusions: I. Some wheels are books.II. Some roads are books.III. Some wheels are trees.

a) Only I and II followb) Only II and III followc) Only I and III followd) All I, II, III followe) None of the above

Page 25: Reasoning Made Easy - pdfstores.files.wordpress.com · REASONING MADE EASY 6 Geetika, Jasmine, Hema, Brinda, Ipsita and veena, once again not following any order. The following are

REASONING MADE EASY www.BankExamsToday.com

25

120. Statements: All stones are rivers.All rivers are cars.Some cars are trains.

Conclusions: I. Some trains are stonesII. Some cars are stones.III. Some trains are rivers.

a) None followsb) Only I followsc) Only II followsd) Only III followse) Only II and III follow

121. Statements: All desks are rooms.Some rooms are halls.All halls are leaves.

Conclusions: I. Some leaves are desks.II. Some halls are desks.III. Some leaves are rooms.

a) None followsb) Only I followsc) Only II followsd) Only III followse) Only II and III follow

122. Statements: Some bags are plates.Some plates are chairs.All chairs are tables.

Conclusions: I. Some tables are plates.II. Some chairs are bags.III. No chair is bag.

a) Only I followb) Only II followsc) Only III followsd) Only II and III followe) None of these

Directions (Q. 123-128) In the following questions, the symbols $, %, @, © and* are used with the following meaning as illustrated below.

‘P % Q’ means ‘P is not smaller than Q’.‘P $ Q’ means ‘P is not greater than Q’.‘P * Q’ means ‘P is neither greater than nor equal to Q’.

Page 26: Reasoning Made Easy - pdfstores.files.wordpress.com · REASONING MADE EASY 6 Geetika, Jasmine, Hema, Brinda, Ipsita and veena, once again not following any order. The following are

REASONING MADE EASY www.BankExamsToday.com

26

‘P © Q’ means ‘P is neither smaller than nor equal to Q’.‘P @ Q’ means ‘P is neither greater than nor smaller than Q’.

123. Statements: R @ K, K $ F, F * NConclusions: I. N © R

II. F @ RIII. F © R

a) Only I is trueb) Only either II or III is truec) Only I and either II or III are trued) Only III is truee) None of the above

124. Statements: J © M, M * K, K % DConclusions: I. J © D

II. D * MIII. K © J

a) None is trueb) Only I is truec) Only II is trued) Only III is truee) Only I and II are t rue

125. Statements: H * T, T $ B, B © RConclusions: I. R © H

II. B © HIII. T * R

a) Only I is trueb) Only I and II are truec) Only I and III are trued) Only II and III are truee) None of the above

126. Statements: R $ D, D @ N, N © FConclusions: I. F * D

II. F * RIII. N % R

a) Only I and II are trueb) Only I and III are truec) Only II and III are trued) All are truee) None of the above

127. Statements: F $ B, B @ H, H % KConclusions: I. B % F

II. K @ F

Page 27: Reasoning Made Easy - pdfstores.files.wordpress.com · REASONING MADE EASY 6 Geetika, Jasmine, Hema, Brinda, Ipsita and veena, once again not following any order. The following are

REASONING MADE EASY www.BankExamsToday.com

27

III. K $ Ba) Only I is trueb) Only II is truec) Only III is trued) Only I and III are truee) None of these

128. Statements: M % D, D * K, K $ NConclusions: I. K © M

II. N © DIII. M © N

a) Only I is trueb) Only II is truec) Only III is trued) Only I and II are truee) None of these

Directions (Q. 129-133) Study the following information carefully and answerthe questions given below.

P, Q, R, S, T, V and W are seven students of a college. Each of them has afavourite subject from Physics, Chemistry, English, Biology, History, Geographyand Philosopy, not necessarily in the same order. Each of them also has a favouritesport from Football, Cricket, Hockey, Volleyball, Badminton, Table Tennis andBasketball, not necessarily in the same order.

R likes Philospy and his favourite sport is Hockey. The one who likesFootball likes English. T’s favourite sport is not badminton or Table Tennis. V doesnot like either History or Bilogy. The one whose favourite sport is Basketball doesnot like Physics. W likes Chemistry and his favourite sport is Volleball. S likesGeography. Q’s favourite sport is Badminton. V does not like English and hisfavourite sport is not Basketball. P’s favourite sport is Cricket. The one whosefavourite sport is Badminton does not like Biology.

129. Who likes History?a) P b) R c) Qd) V e) Data inadequate

130. Whose favourite sport is Basketball?a) S b) W c) Qd) Data inadequate e) None of these

131. Which subject does T like?a) Biology b) Physics c) Chemistryd) Data inadequate e) None of these

Page 28: Reasoning Made Easy - pdfstores.files.wordpress.com · REASONING MADE EASY 6 Geetika, Jasmine, Hema, Brinda, Ipsita and veena, once again not following any order. The following are

REASONING MADE EASY www.BankExamsToday.com

28

132. What is Q’s favourite sport?a) Cricket b) Table Tennis c) Footballd) Badminton e) None of these

133. Which subject does P like?a) History b) Biology c) Chemistryd) Data inadequate e) None of these

134. ‘Mustard’ is related to ‘seed’ in the same way as ‘Carrot’ is related toa) Fruit b) Stem c) Flowerd) Root e) None of these

135. How many meaningful English words can be formed made with the lettersESTR using each letter only once in each word?a) None b) One c) Twod) Three e) More than three

136. Four of the following five are alike in a certain way and so form a group.Which is the one that does not belong to that group?a) Cup b) Jug c) Tumblerd) Plate e) Pitcher

137. Four of the following five are alike in a certain way and so form a group.Which is the one that does not belong to that group?a) Copper b) Mercury c) Irond) Aluminium e) Zinc

138. ‘FI’ is related to ‘LO’ in the same way as ‘PS’ in related toa) VY b) VZ c) WZd) UX e) None of these

139. Four of the following five are alike in a certain way and so form a group.Which is the one that does not belong to that group?a) 217 b) 143 c) 241d) 157 e) 181

140. ‘Gram’ is related to ‘Mass’ in the same way as ‘Centimetre’ is related toa) Area b) Volume c) Lengthd) Sound e) Energy

141. Four of the following five are alike in a certain way and so form a group.Which is the one that does not belong to that group?a) 12 b) 28 c) 52d) 68 e) 96

Page 29: Reasoning Made Easy - pdfstores.files.wordpress.com · REASONING MADE EASY 6 Geetika, Jasmine, Hema, Brinda, Ipsita and veena, once again not following any order. The following are

REASONING MADE EASY www.BankExamsToday.com

29

142. If ‘white’ means ‘black’, ‘black’ means ‘red’, ‘red’ means ‘blue’. ‘blue’means ‘yellow’ and ‘yellow’ means ‘grey’ then which of the followingrepresents the colour of clear sky?a) Blue b) Red c) Yellowd) Can’t be determined e) None of these

143. In a certain code, MODEL is written as ‘513#2’ and DEAR is written as‘3#%8’. How is LOAD written in that code?a) 21%3 b) 23%1 c) 25%3d) 21#3 e) None of these

Directions (Q. 144-148) Study the following information carefully and answerthe questions given below.

P, Q, R, S, T and M are six students of a school, one each studies in Class I-VI. Each of them has a favourite colour from red, black, blue, yellow, pink andgreen, not necessarily in the same order.

Q likes black and does not study in Class IV or V. The one who studies inClass IV does not like green. P studies in Class II. M likes blue and does not studyin Class IV. The one who likes yellow studies in Class VI. S likes pink and studiesin Class I. R do not study in Class VI.

144. In which class does R study?a) V b) III c) IVd) Data inadequate e) None of these

145. Which colour does R like?a) Black b) Yellow c) Greend) Blue e) None of these

146. Which colour does P like?a) Green b) Yellow c) Redd) Data inadequate e) None of these

147. Which of the following combinations is correct?a) P-II-Yellow b) Q-III-Green c) S-I-Blackd) T-V-Yellow e) None of these

148. In which class does M study?a) IV b) III c) IId) V e) None of these

149. How many meaningful English words can be formed with the lettersMASTE using each letter only once in each word?a) None b) One c) Twod) Three e) More than three

Page 30: Reasoning Made Easy - pdfstores.files.wordpress.com · REASONING MADE EASY 6 Geetika, Jasmine, Hema, Brinda, Ipsita and veena, once again not following any order. The following are

REASONING MADE EASY www.BankExamsToday.com

30

150. Four of the following five are alike in a certain way and so form a group.Which is the one that does not belong to that group?a) 78 b) 48 c) 72d) 54 e) 42

Directions (Q. 151-155) Study the following information carefully and answerthe questions given below.

P, Q, R, S, T, V and W are seven students of a school. Each of them studiesin different standard from standard IV to standard X not necessarily in the sameorder. Each of them has favourite subject from English, Science, History,Geography, Mathematics, Hindi and Sanskrit not necessarily in the same order.

Q studies in VII standard and does not like either Mathematics orGeography. R likes English and does not study either in V or in IX. T studies inVIII standard and likes Hindi. One who likes Science studies in X standard? Sstudies in IV standard. W likes Sanskrit. P does not study In X standard. One wholikes Geography studies in V standard.

151. In which standard does W study?a) VII b) IX c) Xd) Data inadequate e) None of these

152. Which subject does P like?a) Geography b) Mathematics c) Englishd) History e) None of these

153. Which subject does S like?a) History b) Geography c) Mathematicsd) Data inadequate e) None of these

154. In which standard does P study?a) IV b) VII c) IXd) X e) None of these

155. Which of the following combination of student standard subject is correct?a) T-VIII-Mathematicsb) W-VII-Sanskritc) Q-VII-Geographyd) V-X-Sciencee) None of these

Directions (Q. 156-160) Study the following information carefully and answerthe questions given below

Page 31: Reasoning Made Easy - pdfstores.files.wordpress.com · REASONING MADE EASY 6 Geetika, Jasmine, Hema, Brinda, Ipsita and veena, once again not following any order. The following are

REASONING MADE EASY www.BankExamsToday.com

31

(i) A, B, C, D, E, F, G and H are 8 students each having a different height.(ii) D is shorter than A but taller than G.(iii) E is taller than H but shorter than C.(iv) B is shorter than D but taller than F.(v) C is shorter than G.(vi) G is not as tall as F.

156. Which of the following is definitely false?a) G is shorter than Fb) C is shorter than Fc) F is taller than Cd) B is taller than Ee) All are true

157. If another student J who is taller than E but shorter than G is added to thegroup. Which of the following will be definitely true?a) C and J are of same heightb) G is shorter than Dc) G is shorter than Hd) G is taller than Ae) None of the above

158. Which of the following will definitely be the third from top when 8 studentsare arranged in descending order of height?a) B b) F c) Gd) B or G e) Cannot be determined

159. How many of them are definitely shorter than F?a) Three b) Four c) Fived) Data inadequate e) None of these

160. Which of the following is redundant to answer all the above questions?a) (ii) onlyb) (ii) and (iii) onlyc) (iii) and (iv) onlyd) (ii) and (v) onlye) All are necessary to answer the above questions

161. A, B, C, D and E are five students in a class. D did not enter along with Aor E but entered before C. B did not enter before C but entered along withA. E was n ot t h e last to enter. Which of the following I s definitely true?a) C entered the class only after Db) D entered the class only after Ec) B entered the class after Ad) A entered the class after De) None of the above

Page 32: Reasoning Made Easy - pdfstores.files.wordpress.com · REASONING MADE EASY 6 Geetika, Jasmine, Hema, Brinda, Ipsita and veena, once again not following any order. The following are

REASONING MADE EASY www.BankExamsToday.com

32

162. Among A, B, C, D and E, each having a different weight, D is not lighterthan B and E is not heavier than A. C is not the heaviest. Who among themis the lightest?a) D b) B c) Ed) Data inadequate e) None of these

163. Each consonant in the word BISCUIT is replaced by the next letter in theEnglish alphabet and each vowel is replaced by the previous letter in theEnglish alphabet and the letters so obtained are rearranged in alphabeticalorder, which of the following will be the 3rd from the left end after therearrangement?a) C b) D c) Hd) T e) None of these

164. In a certain code DONE is written as ‘5139’ and SEAL is written as ‘8942’.How is LOAD written in that code?a) 2415 b) 2145 c) 2945d) 2182 e) None of these

165. Four of the following five are alike in a certain way and so form a group.Which is the one that does not belong to that group?a) 215 b) 143 c) 247d) 91 e) 65

166. Four of the following five are alike in a certain way and so form a group.Which is the one that does not belong to that group?a) Gold b) Nickel c) Platinumd) Tungsten e) Diamond

167. Among P, Q, T, A and B each having a different h eight, T is taller than Pand B but shorter than A and Q. P is not the shortest. Who among them isthe tallest?a) A b) Q c) Pd) P or B e) Data inadequate

168. How many meaningful English words can b e made with the letter RBAEusing each letter only once in each word?a) None b) One c) Twod) Three e) More than three

169. What should come next in the following letter series?H G F E D C B A G F E D C B A G F E D C Ba) E b) G c) Fd) B e) None of these

Page 33: Reasoning Made Easy - pdfstores.files.wordpress.com · REASONING MADE EASY 6 Geetika, Jasmine, Hema, Brinda, Ipsita and veena, once again not following any order. The following are

REASONING MADE EASY www.BankExamsToday.com

33

170. Mohan correctly remembers that his father’s birthday is before 20th Januarybut after 16th January whereas his sister correctly remembers that theirfather’s birthday is after 18th January but before 23rd January. On whichdate in January is definitely their father’s birthday?a) 18th b) 19th c) 20th

d) Data inadequate e) None of these

171. If ‘P’ denotes ‘×’, ‘R’ denotes ‘+’, ‘M’ denotes ‘⎯’ and ‘W’ denotes ‘+’then20 R 5 W 12 M 3 P 4 =?a) 4 b) 16 c) 28d) 52 e) None of these

172. If the positions of 2nd and 3rd digits within each number are interchanged,which of the following will be the sum of the first and 2nd digits of the 3rd

highest number?a) 16 b) 10 c) 9d) 15 e) None of these

173. Which of the following are the sum of the first and the third digits of thesecond lowest number?a) 16 b) 10 c) 18d) 5 e) None of these

174. If the positions of the first and the second digits within each number areinterchanged, which of the following will be the difference between thehighest and the second highest number?a) 203 b) 133 c) 385d) 182 e) 144

175. If the positions of the first and the third digits within each number areinterchanged, which of the following will be the sum of the second andthird digits of the lowest number?a) 8 b) 11 c) 15d) 12 e) None of these

Directions (Q. 176-178) Study the following information carefully and answerthe questions given below

There are six employees – A, B, C, D, E and F in a company. A is moreexperienced than two employees. D has more experience than employee C. D hasless experience than employee F. E has less experience than D. E does not have theleast experience. F is not the most experienced employee. The employee who is 2nd

most experienced has an experience of 13 years. The employee who is second leastexperienced has an experience of five years.

Page 34: Reasoning Made Easy - pdfstores.files.wordpress.com · REASONING MADE EASY 6 Geetika, Jasmine, Hema, Brinda, Ipsita and veena, once again not following any order. The following are

REASONING MADE EASY www.BankExamsToday.com

34

176. Who among the following may have an experience of 10 years?a) E b) D c) Ad) Either A or D e) F

177. Who among the following is less experinced than only B?a) F b) C c) Dd) A e) E

178. Who among the following is more experienced than only C?a) A b) D c) Ed) F e) Cannot be determined

Directions (Q. 179-183) Study the following information carefully and answerthe questions given below

Seven persons – A, B, C, D, E, F and G – bought different watches viz.,Rado, Omni, Fas Track, Seiko, Casio, Tissot and Titan on different days of thesame week from Monday to Sunday, but not necessarily in the same order.

The watch of Omni Company was bought on Friday. F bought Titan watchon Tuesday. Onle one watch was bought between the watches Omni and Rado. Bbought Tissot watch immediately after the person who bought Casio watch. Seikowatch was not bought after the Tissot watch. G bought watch immediately after Bbut before C and D. A does not buy the watch of Casio company. D does not buythe watch of Rado Company.

179. Who among the following bought Seiko watch?a) D b) C c) Ed) A e) None of these

180. On which of the following days of the week D bought a watch?a) Saturday b) Sunday c) Thursdayd) Friday e) None of these

181. Who among the following did buy Casio watch?a) G b) E c) Dd) C e) None of these

182. Who among the following did buy Omni watch on Friday?a) C b) D c) Gd) E e) None of these

183. Which of the following combinations of Day-Person-Watch is correct?a) Friday-A-Fast Trackb) Monday-E-Omnic) Wednesday-B-Casiod) Saturday-C-Tissot

Page 35: Reasoning Made Easy - pdfstores.files.wordpress.com · REASONING MADE EASY 6 Geetika, Jasmine, Hema, Brinda, Ipsita and veena, once again not following any order. The following are

REASONING MADE EASY www.BankExamsToday.com

35

e) Sunday-C-Rado

184. Present ages of father and son are in the ratio of 5:1 respectively. Sevenyears later this ratio becomes 3:1. What is the son’s present age in years?a) 8 b) 7 c) 6d) 5 e) None of these

185. From the given alternative words, select the word which can be formedusing the letters of the given word: TRADITIONALa) NATION b) RADIO c) ANIMALd) DIRTY e) None of these

186. A shepherd had 17 sheep. All but nine died. How many sheep are left?a) 9 b) 8 c) 7d) 10 e) None of these

187. Select the missing number from the given responses.1 4 2 3 2 ?a) 2 b) 3 c) 4d) 5 e) None of these

188. Town A and Town B were 600 km. apart. Joshua left town A for town Band travelled at an average speed of 65 km/h. At the same time, Menon lefttown B for town A. The two of them met 5 hours later. Find Menon’saverage speed.a) 55 km/h b) 60 km/h c) 65 km/hd) 120 km/h e) None of these

189. In a certain code ALIVE is written as WFHBM. How is VALUE written inthat code?a) VFMWB b) VFKWB c) WBAVFd) WBKVF e) None of these

190. The letters in the word DANGEROUS are changed in such a way that theconsonants a rereplaced by the previous letter in the English alphabet andthe vowels are replaced by the next letter in the English alphabet. Which ofthe following will be the 3rd letter from the left end of the new set of letters?a) B b) M c) Od) L e) None of these

Directions (Q. 191-195) Study the following information carefully and answerthe questions given below

A, B, C, D, E, F, G and H are eight persons working in three differentdepartments viz. Operations, Sales and Finance of an organisation with at least two

Page 36: Reasoning Made Easy - pdfstores.files.wordpress.com · REASONING MADE EASY 6 Geetika, Jasmine, Hema, Brinda, Ipsita and veena, once again not following any order. The following are

REASONING MADE EASY www.BankExamsToday.com

36

and not more three in any department. They are in three different scales viz. I, II andIII with at least two in any one scale.

Both the employees in Operations departments are in Scale II. D works inSales department and belongs to Scale I. A works in Finance departments and doesnot belong to Scale I. Two employees in Sales departments belong to one scale. Fworks with only H in one of the departments. C works with E in one of thedepartments. B does not work with either C or A in the same department. G doesnot belong to Scale III. E does not belong to Scale I.

191. Which of the following combinatins is correct?a) Operation-F-Ib) Operation-H-IIIc) Sales-B-IId) Finance-E-IIIe) All are correct

192. Which of the following groups of employees work in sales department?a) DBE b) DBC c) DBGd) Data inadequate e) None of these

193. H belongs to which scale?a) I b) II c) IIId) Data inadequate e) None of these

194. G belongs to which scale?a) II b) III c) Id) I or II e) None of these

195. C belongs to which scale?a) I b) II c) IIId) I or II e) None of these

Answers:

1. Option A2. Option C3. Option E4. Option C5. Option A6. Option A7. Option B8. Option E9. Option B10. Option C11. Option D

Page 37: Reasoning Made Easy - pdfstores.files.wordpress.com · REASONING MADE EASY 6 Geetika, Jasmine, Hema, Brinda, Ipsita and veena, once again not following any order. The following are

REASONING MADE EASY www.BankExamsToday.com

37

12. Option A13. Option D14. Option C15. Option C16. Option C17. Option D18. Option A19. Option A20. Option C21. Option B22. Option D23. Option A24. Option E25. Option E26. Option D27. Option C28. Option C29. Option B30. Option C31. Option A32. Option D33. Option A34. Option B35. Option C36. Option B37. Option A38. Option E39. Option C40. Option B41. Option E42. Option A43. Option C44. Option A45. Option A46. Option C47. Option B48. Option C49. Option D50. Option A51 Option A52. Option B53. Option D54. Option E55. Option C56. Option B57. Option C

Page 38: Reasoning Made Easy - pdfstores.files.wordpress.com · REASONING MADE EASY 6 Geetika, Jasmine, Hema, Brinda, Ipsita and veena, once again not following any order. The following are

REASONING MADE EASY www.BankExamsToday.com

38

58. Option B59. Option B60. Option D61. Option C62. Option A63. Option A64. Option C65. Option C66. Option B67. Option A68. Option C69. Option A70. Option A71. Option B72. Option D73. Option E74. Option A75. Option B76. Option B77. Option D78. Option A79. Option A80. Option C81. Option B82. Option D83. Option C84. Option D85. Option A86. Option E87. Option E88. Option C89. Option A90. Option E91. Option C92. Option B93. Option C94. Option A95. Option C96. Option C97. Option E98. Option D99. Option C10. Option B101. Option A102. Option D103. Option A

Page 39: Reasoning Made Easy - pdfstores.files.wordpress.com · REASONING MADE EASY 6 Geetika, Jasmine, Hema, Brinda, Ipsita and veena, once again not following any order. The following are

REASONING MADE EASY www.BankExamsToday.com

39

104. Option E105. Option B106. Option C107. Option D108. Option A109. Option E110. Option D111. Option B112. Option C113. Option B114. Option A115. Option E116. Option D117. Option C118. Option A119. Optoin D

120. Option C

Page 40: Reasoning Made Easy - pdfstores.files.wordpress.com · REASONING MADE EASY 6 Geetika, Jasmine, Hema, Brinda, Ipsita and veena, once again not following any order. The following are

REASONING MADE EASY www.BankExamsToday.com

40

121. Option D

122. Option E

Page 41: Reasoning Made Easy - pdfstores.files.wordpress.com · REASONING MADE EASY 6 Geetika, Jasmine, Hema, Brinda, Ipsita and veena, once again not following any order. The following are

REASONING MADE EASY www.BankExamsToday.com

41

123. Option C

R= K, K ≤ F, F < NR = K ≤ F < NN > R, F > R, F = RSo, either II or III and I are true.

124. Option A

J > M, M < K, K ≥ DJ > M < K ≥ DThe relations cannot made between J and D, D and M and K and J. So noneis true.

125. Option E

H < T, T ≤ B, B > RH < T ≤ B > R or B > HOnly II is true.

126. Option B

R ≤ D, D = N, N > FR ≤ D = N > FF < D and N ≥ ROnly I and III are true.

127. Option A

F ≤ B, B = H, H ≥ KF ≤ B = H ≥ K

Page 42: Reasoning Made Easy - pdfstores.files.wordpress.com · REASONING MADE EASY 6 Geetika, Jasmine, Hema, Brinda, Ipsita and veena, once again not following any order. The following are

REASONING MADE EASY www.BankExamsToday.com

42

Only I is true.

128. Option B

M ≥ D, D < K, K ≤ NM ≥ D < K ≤ NOnly II is true.

Student P Q R S T V WSports Cricket Badmint

onHockey

Basketball

Football

TableTennis

Volleyball

Subject Bilogy History Philosophy

Geography

English Physics

Chemistry

129. Option D130. Option A131. Option E132. Option D133. Option B

134. Option D

As ‘mustard’ is related to ‘seed’ in the same way ‘carrot’ is related to ‘root’.

135. Option B

Meaningful word is REST.

136. Option D

‘Plate’ is normally used for solid matters.

137. Option B

Except ‘mercury’ all are available in solid form at general or normaltemperature.

138. Option A

As, F + 6 L Similarly P + 6 VI + 6 O S + 6 Y

139. Option A

Page 43: Reasoning Made Easy - pdfstores.files.wordpress.com · REASONING MADE EASY 6 Geetika, Jasmine, Hema, Brinda, Ipsita and veena, once again not following any order. The following are

REASONING MADE EASY www.BankExamsToday.com

43

140. Option CAs ‘gram’ is related to ‘mass’ in the same way ` ‘centimetre’ is related to‘length’.

141. Option B28 is divisible by 7.

142. Option CColour of clear sky is blue and according to question, blue means yellow sothe colour of clear sky will be ‘yellow’.

143. Option AM O D E L5 1 3 # 2

D E A R3 # % 8

L O A D2 1 % 3

Student Class Favaourite colourP II GreenQ III BlackR IV RedS I PinkT VI YellowM V Blue

144. Option C145. Option E146. Option A147. Option E148. Option D

149. Option EMeaningful words = MATES, STEAM, TAMES and TEAMS

150. Option AExcept 78, if subtract 1 all other give a prime number.

Student Class Favourite SubjectP V Geography

Page 44: Reasoning Made Easy - pdfstores.files.wordpress.com · REASONING MADE EASY 6 Geetika, Jasmine, Hema, Brinda, Ipsita and veena, once again not following any order. The following are

REASONING MADE EASY www.BankExamsToday.com

44

Q VII HistoryR VI EnglishS IV MathematicsT VIII HindiV X ScienceW IX Sanskrit

151. Option B152. Option A153. Option C154. Option E155. Option D

(i) There are 8 students A, B, C, D, E, F, G and H(ii) A > D > G(iii) C > E > H(iv) D > B > F(v) G > C(vi) F > GFrom all statements.A > D > B > F > G > C > E > H

156. Option EAll are true.

157. Option BA > D > B > F > G > C > E > H

158. Option AA > D > B > F > G > C > E > H

159. Option BG, C, E and H is smaller than F.

160. Option EAll statements are necessary.

161. Option D162. Option D

163. Option CB I S C U I TAccording to question,C H T D T H U

Page 45: Reasoning Made Easy - pdfstores.files.wordpress.com · REASONING MADE EASY 6 Geetika, Jasmine, Hema, Brinda, Ipsita and veena, once again not following any order. The following are

REASONING MADE EASY www.BankExamsToday.com

45

According to alphabet,C D H H T T USo, H is 3rd from left.

164. Option BD O N E5 1 3 9

S E A L8 9 4 2

So, L O A D2 1 4 5

165. Option A

Apart from 215 all are divisible by 13.143 = 13 × 11; 247 = 13 × 19; 91 = 13 × 7; 65 = 13 × 5215 = 5 × 43

166. Option E167. Option E168. Option C

Meaningful words are BEAR, BARE

169. Option CHGFEDCBA, GFEDCBA, GFEDCB, FEDCBNext letter series = F

170. Option BAccording to Mohan, birthday may be 17, 18 or 19th January. According toMohan’s sister, birthday may be at 19, 20, 21 or 22th Jan. So common day= 19 Jan.

171. Option A

20 R 5 W 12 M 3 P 4 = ?? = 20 ÷ 5 + 12 ⎯3 × 4? = 4 + 12 ⎯12 = 4

172. Option A

832 ⟶ 823719 ⟶ 791654 ⟶ 645967 ⟶ 976

Page 46: Reasoning Made Easy - pdfstores.files.wordpress.com · REASONING MADE EASY 6 Geetika, Jasmine, Hema, Brinda, Ipsita and veena, once again not following any order. The following are

REASONING MADE EASY www.BankExamsToday.com

46

481 ⟶ 4183rd largest number is 791 and the sum of their first and second digit = 7 + 9= 16

173. Option BSecond smallest number is 654. So, the sum of the first and the third digit is6 + 4 = 10

174. Option EAfter interchanging first and second digit of number.832 ⟶ 382719 ⟶ 179654 ⟶ 564967 ⟶ 697481 ⟶ 841Highest number = 841Second highest number = 697Difference between them = 841 ⎯697 = 144

175. Option DInterchanging the first and third digit of number832 ⟶ 238719 ⟶ 917654 ⟶ 456967 ⟶ 769481 ⟶ 184Smallest number = 184So, the sum of second and third digit = 8 + 4 = 12

176. Option DF is not the most experienced. So, B is the most experienced.Now, B > F > D > A > E > C

177. Option AF has less experience than only B.

178. Option CE is more experienced than only C.

Day Person WatchMonday A SeikoTuesday F TitanWednesday E Casio

Thursday B TissotFriday G Omni

Page 47: Reasoning Made Easy - pdfstores.files.wordpress.com · REASONING MADE EASY 6 Geetika, Jasmine, Hema, Brinda, Ipsita and veena, once again not following any order. The following are

REASONING MADE EASY www.BankExamsToday.com

47

Saturday D Fast TrackSunday C Rado

179. Option DA bought Seiko watch on Monday.

180. Option AD bought Fast Track watch on Saturday.

181. Option BE bought Casio watch on Wednesday.

182. Option CG bought Omni watch on Friday.

183. Option EThe combination Sunday-C-Rado is correct.

184. Option BSuppose the present age of son be x yearsTherefore, the present age o f father would b e 5x yearsAccording to question,5x + 7 / x + 7 = 3/15x + 7 = 3x + 215x ⎯3x = 21 ⎯72x = 14X = 7

185. Option BThere is only one ‘N’ in the given word. Therefore, the word NATIONcannot be formed. There is no ‘M’ letter in the given word. Therefore, theword ANIMAL cannot be formed. There is no ‘Y’ letter in the given word.Therefore, the word DIRTY cannot be formed.

186. Option AThere are 17 sheep. All but nine died. It means 9 sheep are left.

187. Option D1 + 3 = 44 ⎯2 = 22 + 1 = 33 ⎯1 = 22 + 3 = 5

188. Option A

Page 48: Reasoning Made Easy - pdfstores.files.wordpress.com · REASONING MADE EASY 6 Geetika, Jasmine, Hema, Brinda, Ipsita and veena, once again not following any order. The following are

REASONING MADE EASY www.BankExamsToday.com

48

Joshua covered a distance of 65 × 5 = 325 km. in 5 hoursMenon has to cover a distance of (600 ⎯325) = 275 km. in 5 hoursSpeed of Menon = 275/5 = 55 kmph

189. Option B190. Option B

Person Department ScaleA Finance IIIB Sales IIIC Finance IIID Sales IE Finance IIIF Operations IIG Sales IH Operations II

191. Option D

The combination Finance-E-III is correct.

192. Option C

B, D and G work in Sales department.

193. Option B

H belongs to scale II

194. Option C

G belongs to Scale I

195. Option CC belongs to scale III

Page 49: Reasoning Made Easy - pdfstores.files.wordpress.com · REASONING MADE EASY 6 Geetika, Jasmine, Hema, Brinda, Ipsita and veena, once again not following any order. The following are

REASONING MADE EASY www.BankExamsToday.com

49

Page 50: Reasoning Made Easy - pdfstores.files.wordpress.com · REASONING MADE EASY 6 Geetika, Jasmine, Hema, Brinda, Ipsita and veena, once again not following any order. The following are

REASONING MADE EASY www.BankExamsToday.com

50

Chapter-2

Input-Output

Directions (Q. 1-5) A word can and number arrangement machine when givenan input line of words and numbers rearranges them following a particularrule in each step. The following is an illustration of input and rearrangement.

Input : go now 52 38 17 for again 65Step I: 65 go now 52 38 17 for againStep II: 65 again go now 52 38 17 forStep III: 65 again 52 go now 38 17 forStep IV: 65 again 52 for go now 38 17Step V: 65 again 52 for 38 go now 17Step VI: 65 again 52 for 38 go 17 now

Step VI is the last step of the rearrangement.As per the rules followed in the above steps, find out in each of the followingquestions the appropriate step for the given input.

Input: show 51 36 new far 81 46 goal

1. Which of the following steps will be the last but one?a) VII b) VIII c) VId) V e) None of these

Input: home turf 39 24 86 44 roll over2. Which of the following steps will be the last?

a) X b) IX c) VIIId) VII e) None of these

Step II of an input is: 76 ask 12 32 begin over join 42.

3. How many more steps will be required to complete the rearrangement?a) Four b) Five c) Sixd) Three e) None of these

Step IV of an input is: 58 box 47 dew 15 21 town pot.4. Which of the following steps will be the last?

a) VII b) VI c) VIIId) IX e) None of these

Step III of an input is: 94 car 86 window shut 52 31 house.

Page 51: Reasoning Made Easy - pdfstores.files.wordpress.com · REASONING MADE EASY 6 Geetika, Jasmine, Hema, Brinda, Ipsita and veena, once again not following any order. The following are

REASONING MADE EASY www.BankExamsToday.com

51

5. Which of the following is definitely the input?a) 94 car window 86 shut 52 31 houseb) 80 window 94 car shut 52 31 housec) car shut window 86 52 31 house 94d) Cannot be determinede) None of these

Directions (Q. 6-11) Given an input line the machine arranges the words andnumbers in steps in a systematic manner as illustrated below

Input line 59 dress fine shine 32 66 72 offerStep I 72 56 dress fine shine 32 66 offerStep II 72 shine 56 dress fine 32 66 offerStep III 72 shine 66 56 dress fine 32 offerStep IV 72 shine 66 offer 56 dress fine 32Step V 72 shine 66 offer 56 fine dress 32Step VI 72 shine 66 offer 56 fine 32 dressStep VI is the last step and the output in Step Vi is the final output.

As per the rules followed in the above steps, find out in each of the followingquestions the appropriate step for the given input.

6. Step IV of an input is ’62 sound 56 sleep roam present 33 49’. What will be theinput definitely?a) sound 62 sleep 56 roam present 33 49b) sleep sound 62 56 roam present 33 49c) 62 sound sleep 56 roam present 33 49d) Cannot be determinede) None of these

7. Which of the following will be the t hird step for input ‘jockey firm 36 43growth chart 22 45?a) 45 jockey 43 growth firm 36 chart 22b) 45 jockey 43 firm growth 36 chart 22c) 45 jockey 43 growth 36 firm chart 22d) 45 jockey 43 firm 36 growth chart 22e) None of these

8. Step II of an input is ’53 window 42 50 door lock key 36’. How many moresteps will be required to complete the arrangement?

a) Three b) Four c) Fived) Six e) None of these

Page 52: Reasoning Made Easy - pdfstores.files.wordpress.com · REASONING MADE EASY 6 Geetika, Jasmine, Hema, Brinda, Ipsita and veena, once again not following any order. The following are

REASONING MADE EASY www.BankExamsToday.com

52

9. What will be the fifth step of an input whose first step is ’85 journey train 36 54daily 28 mansion’?a) 85 train 54 mansion 28 journey daily 36b) 85 train 54 mansion journey 36 daily 28c) 85 train 54 mansion 36 journey daily 28d) There is no such stepe) None of these

10. Which step will bed the last step for an I nput whose second step is ‘63’ Sour 1856 grapes healthy 32 rise’?a) IV b) V c) VIIId) VII e) None of these

11. Which word/number will be sixth from right in step fifth whose second step is’63 Sour 18 56 grapes healthy 32 rise’?a) Rise b) 56 c) Sourd) 32 e) 18

Directions (Q. 12-16) Read the following information carefully and answer thegiven questions.

A word/number arrangement machine when given an input line of wordsand numbers rearrange them following a particular rule in each step. The followingis an illustration of input and rearrangement.(All the numbers are two digits numbers)

Input: 33 food water 19 42 air 27 54 72 fire lunch 62 cool notStep 1: 19, 33 food 42 air 27 54 72 fire lunch 62 cool not waterStep 2: 27 19 33 food 42 air 54 72 fire lunch 62 cool water notStep 3: 33 27 19 food 42 air 54 72 fire 62 cool water not lunchStep 4: 42 33 27 19 air 54 72 fire 62 cool water not lunch foodStep 5: 54 42 33 27 19 air 72 62 cool water not lunch food fireStep 6: 62 54 42 33 27 19 air 72 water not lunch food fire coolStep 7: 72 62 54 42 33 27 19 water not lunch food fire cool airAnd step 7 is the last step of the above input, as the desired arrangement is obtained.

As per the rules followed in the above steps, find out in each of the followingquestions the appropriate step for the given input.Input: Rail 43 27 Book Coach 56 Ticket waiting confirm 62 35 72 sleeper 16(All the numbers are two digit numbers)

12. Which step number is the following output?’56 43 35 27 16 Book Coach 62 72 waiting ticket sleeper rail confirma) Step 4 b) Step 5 c) Step 6d) Step 7 e) None of these

Page 53: Reasoning Made Easy - pdfstores.files.wordpress.com · REASONING MADE EASY 6 Geetika, Jasmine, Hema, Brinda, Ipsita and veena, once again not following any order. The following are

REASONING MADE EASY www.BankExamsToday.com

53

13. Which of the following step represents the maximum gap between ‘Ticket’and ‘35’?a) Step 2 b) Step 3 c) Step 4d) Step 5 e) None of these

14. In step 5, if ‘Book’ is related to ‘waiting’ and in the same way ‘27’ isrelated to ‘62’. Which of the following would ‘72’ be related to in the samepattern?a) Ticket b) Rail c) Sleeperd) 35 e) None of these

15. In step 4, which of the following word/number would be at 4th positionfrom the left end?a) 16 b) 27 c) Bookd) Rail e) None of these

16. Which of the following would be one of the steps?a) 27 16 Rail 43 Book Coach 56 Confirm 62 35 sleeper 72 waiting Ticketb) 43 35 27 16 Book Coach Confirm 62 56 72 waiting sleeper Ticket Railc) 62 56 43 35 27 16 72 waiting sleeper ticket Rail confirm Coach Bookd) 35 27 16 Rail 43 Book Coach 56 Confirm 62 72 waiting Ticket sleepere) 56 43 35 27 16 Book confirm 62 72 waiting Ticket sleeper Rail coach

Directions (Q. 17-23) Study the following information carefully and answer thequestions based on it.

A set of words and numbers is passed through on arrangement machine andfollowing rearrangement is obtained.

Input talk 48 11 rude 97 84 35 walk jug home 25 bag 77 aloneStep I 97 talk 48 11 rude 84 35 walk jug home 25 77 alone bagStep II 97 84 talk 48 11 rude 35 walk jug 25 77 alone bag homeStep III 97 84 77 talk 48 11 rude 35 walk 25 alone bag home jugStep IV 97 84 77 48 talk 11 35 walk 25 alone bag home jug rudeStep V 97 84 77 48 35 11 nwalk 25 alone bag home jug rude talkStep VI 97 84 77 48 35 25 11 alone bag home jug rude talk walkStep VI is the last step of rearrangement.

Now, a set of words and letters is given below. As per the rules followed in abovesteps, answer the questions based on it.Input: 81 who sit 19 32 not but ink flow 51 27 van 68 92

17. Which is the last step of this input?a) V b) VI c) VIId) VIII e) None of these

Page 54: Reasoning Made Easy - pdfstores.files.wordpress.com · REASONING MADE EASY 6 Geetika, Jasmine, Hema, Brinda, Ipsita and veena, once again not following any order. The following are

REASONING MADE EASY www.BankExamsToday.com

54

18. Which of the following is the Step III?a) 92 81 68 who sit 19 32 not 51 27 van but flow inkb) 92 81 68 51 who sit 32 19 not 27 van but flow inkc) 92 81 68 51 who sit 19 32 not 27 van but flow inkd) 92 81 68 51 32 who sit 19 not 27 van but flow inke) None of these

19. Which step is ’92 81 68 51 32 27 who 19 van but flow ink not sit’?a) Step IV b) Step V c) Step IIId) Step VI e) None of these

20. Which is third element to the right from the sixth element from the rightend in Step II?a) 32 b) van c) 27d) floe e) 25

21. How many elements are there between “who” and “van” in Step IV?a) 3 b) 5 c) 4d) 0 e) 1

22. In Step V, if ‘92’ is related to ‘sit’ and ‘81’ is related to ‘not’, similarly, 68is related to which element?a) ink b) flow c) butd) sit e) none of these

23. In Step II, if ‘not’ is related ‘92’, ‘51’ is related to ‘68’, ‘van’ is related to‘sit’, in the same way ‘flow’ is related to which element?a) 81 b) who c) 19d) 32 e) 23

Directions (Q. 24-28) Study the given information and answer the followingquestions.

When a word and number arrangement machine is given an input line ofwords and numbrs, it arranges them following a particular rule. The following is anillustration of input and rearrangement. (All the numbers are two digit numbers)

Input 40 made butter 23 37 cookies salt extra 52 86 92 fell now 19Step I butter 19 40 made 23 37 cookies salt extra 52 86 92 fell nowStep II cookies 23 butter 19 40 made 37 salt extra 52 86 92 fell nowStep III extra 37 cookies 23 butter 19 40 made salt 52 86 92 fell nowStep IV fell 40 extra 37 cookies 23 butter 19 made salt 52 86 92 nowStep V made 52 fell 40 extra 37 cookies 23 butter 19 salt 86 92 nowStep VI now 86 made 52 fell 40 extra 37 cookies 23 butter 19 salt 92Step VII salt 92 now 86 made 52 fell 40 extra 37 cookies 23 butter 19

Page 55: Reasoning Made Easy - pdfstores.files.wordpress.com · REASONING MADE EASY 6 Geetika, Jasmine, Hema, Brinda, Ipsita and veena, once again not following any order. The following are

REASONING MADE EASY www.BankExamsToday.com

55

Step VII is the last step of the above arrangement as the intendedarrangement is obtained.

As per the rules followed in the given steps, find out the appropriate stepsfor the given input.Input 32 proud girl beautiful 48 55 97 rich family 61 72 17 nice life

24. How many steps will be required to complete the given input?a) Five b) Six c) Sevend) Eight e) Nine

25. Which of the following is the third element from the left end of step VI?a) Beautiful b) Life c) 61d) Nice e) 17

26. Which of the following is step III of the given input?a) proud 72 girl 48 family 32 beautiful 17 55 97 rich 61 nice lifeb) life 55 girl 48 family 32 beautiful 17 proud 97 r ich 61 72 n icd)ec) girl 48 family 32 beautiful 17 proud 55 97 rich 61 72 nice lifed) family 32 beautiful 17 proud girl 48 55 97 rich 61 72 nice lifee) girl 48 life 55 family 32 beautiful 17 proud 97 rich 61 72 nice

27. What is the position of ‘nice’ from the left end in the final step?a) 5th b) 6th c) 7th

d) 8th e) 9th

28. Which element is third to the right of ‘family’ in step V?a) Beautiful b) 17 c) Proudd) 18 e) None of these

Directions (Q. 29-33) Study the given information and answer the followingquestions.

When a word and number arrangement machine is given an input line ofwords and numbers, it arranges them following a particular rule. The following is anillustration of an input and rearrangement.

Input: 45 navel change 33 48 down town frown 62 88 98 gesture orange 21Step I: 45 navel change 33 48 down frown 62 88 98 gesture orange town 21Step II: 45 navel change 48 down frown 62 88 98 gesture town 21 orange 33Step III: change 48 down frown 62 88 98 gesture t own 21 orange 33 navel 45Step IV: change down frown 62 88 98 town 21 orange 33 navel 45 gesture 48Step V: change down 88 98 town 21 orange 33 navel 45 gesture 48 frown 62Step VI: change 98 town 21 orange 33 navel 45 gesture 48 frown 62 down 88Step VII: town 21 orange 33 navel 45 gesture 48 frown 62 down 88 change 98

Page 56: Reasoning Made Easy - pdfstores.files.wordpress.com · REASONING MADE EASY 6 Geetika, Jasmine, Hema, Brinda, Ipsita and veena, once again not following any order. The following are

REASONING MADE EASY www.BankExamsToday.com

56

Step VII is the last step of the above arrangement as the intended arrangement isobtained. As per the rules followed inj the given steps, find out the appropriate stepsfor the given input.

Input: 35 quant hear cute 50 65 98 silent giant 71 82 19 Oliver melody

29. How many steps will be required to complete the given input?a) Five b) Six c) Sevend) Eight e) None of these

30. Which of the following is the fourth element from the left end of Step VI?a) cute b) melody c) 71d) oliver e) None of these

31. What is the position of ‘melody’ from the left end in the final step?a) Sixth b) Seventh c) Firstd) Fifth e) Ninth

32. Which element is fourth to the right of ‘giant’ in Step V?a) cute b) 19 c) quantd) 98 e) 35

33. Which of the following is Step III of the given input?a) quant 82 hear 50 giant 35 cute 19 65 98 silent 71 oliver melodyb) hear cute 65 98 giant 71 82 melody silent 19 quant 35 oliver 50c) hear cute 98 giant 71 82 silent 19 quant 35 oliver 50 melody 65d) hear cute 98 65 giant 71 82 melody silent 19 quant 35 oliver 50e) None of these

Directions (Q. 34-38) Study the following information to answer the givenquestions.

A word and number arrangement machine when given an input line ofwords and numbers rearranges them following a particular rule. The following is anillustration of input and rearrangement. (All the numbers are two-digit numbers.)

Input: yogurt jovial 48 cinema total 20 correct sunny thin 78Step I: 20 total yogurt jovial 48 cinema correct sunny thin 78

Step II: 20 total 48 jovial yogurt cinema correct sunny thin 78Step III: 20 total 48 jovial 78 thin yogurt cinema correct sunnyStep IV: 20 total 48 jovial 78 thin yogurt sunny cinema correctStep V: 20 total 48 jovial 78 thin yogurt sunny correct cinema

Page 57: Reasoning Made Easy - pdfstores.files.wordpress.com · REASONING MADE EASY 6 Geetika, Jasmine, Hema, Brinda, Ipsita and veena, once again not following any order. The following are

REASONING MADE EASY www.BankExamsToday.com

57

Step V is the last step of the rearrangement. As per the rules followed in the abovesteps, find out in each of the following questions the appropriate step for thefollowing input.

Input: united madrid 89 chair 21 aircraft liverpool beneath honey 11 everyday

34. Which step number will be the following output?11 honey 21 chair 89 madrid united aircraft liverpool beneath everydaya) Step IIb) Step IIIc) Step IVd) There will be no such stepe) None of these

35. How many steps will be required to get the final output?a) Five b) Six c) Sevend) Four e) None of these

36. Which word/number would be the fifth from the right in Step V?a) Madrid b) United c) 89d) Liverpoor e) None of these

37. What is the position of ‘beneath’ in Step IV?a) 9th from the leftb) 11th from the leftc) 2nd from the rightd) 4th from the righte) None of these

38. If in a certain way ‘honey’ is related to ‘aircraft’ and ‘chair’ is related to‘beneath’ then ‘madrid’ would be related to which of the following in thelast step?a) 21 b) United c) Everydayd) Liverpool e) None of these

Directions (Q. 39-43) Given an input line the machine arranges the words andnumbers in steps in a systematic manner as illustrated below:

Input: 56 dress fine shine 32 66 72 offer

Step I: 72 56 dress fine shine 32 66 offerStep II: 72 shine 56 dress fine 32 66 offerStep III: 72 shine 66 56 dress fine 32 offerStep IV: 72 shine 66 offer 56 dress fine 32Step V: 72 shine 66 offer 56 fine dress 32Step VI: 72 shine 66 offer 56 fine 32 dress

Page 58: Reasoning Made Easy - pdfstores.files.wordpress.com · REASONING MADE EASY 6 Geetika, Jasmine, Hema, Brinda, Ipsita and veena, once again not following any order. The following are

REASONING MADE EASY www.BankExamsToday.com

58

Step VI is the last step and the output in Step VI I is the final output.

As per the rules followed in the above steps, find out in each of the followingquestions the appropriate step for the given input.

39. Step IV of an input is ’62 sound 56 sleep roam present 33 49’. What willlbe the input definitely?a) sound 62 sleep 56 roam present 33 49b) sleep sound 62 56 roam present 33 49c) 62 sound sleep 56 roam present 33 49d) Cannot be determinede) None of these

40. Which of the following will be the third step for input:‘jockey firm 36 43 growth chart 22 45 ?a) 45 jockey 43 growth firm 36 chart 22b) 45 jockey 43 firm growth 36 chart 22c) 45 jockey 43 growth 36 firm chart 22d) 45 jockey 43 firm 36 growth chart 22e) None of these

41. Step II of an input is ’53 wi ndow 42 50 door lock key 36’. How manymore steps will be required to complete the arrangement?a) Three b) Four c) Fived) Six e) None of these

42. What will be the fifth step of an input whose first step is ’85 journey train36 54 daily 28 mansion’?a) 85 train 54 mansion 28 journey daily 36b) 85 train 54 mansion journey 36 daily 28c) 85 train 54 mansion 36 journey daily 28d) There is no such stepe) None of these

43. Which step will be the last step for an input whose second step is ’63 sour18 56 grapes healthy 32 rise’?a) IV b) V c) VIIId) VII e) None of these

Directions (Q. 44-49) Study the following information carefully and answer thegiven questions.

A word and number arrangement machine when given an input line ofwords and numbers rearranges them following a particular rule in each step. Thefollowing is an illustration of input and rearrangement.

Page 59: Reasoning Made Easy - pdfstores.files.wordpress.com · REASONING MADE EASY 6 Geetika, Jasmine, Hema, Brinda, Ipsita and veena, once again not following any order. The following are

REASONING MADE EASY www.BankExamsToday.com

59

Input: shop 17 table 20 53 oven desk 39

Step I: 17 shop table 20 53 oven desk 39Step II: 17 table shop 20 53 oven desk 39Step III: 17 table 20 shop 53 oven desk 39Step IV: 17 table 20 shop 39 53 oven deskStep V: 17 table 20 shop 39 oven 53 desk

And step V is the last step of the rearrangement.As per the rules followed in the above steps, find out in each of the followingquestions that appropriate step for the given input.

44. Step II of an input is : 15 yes 62 51 48 talk now goneWhich of the following will be step VI?a) 15 yes 48 talk 51 now gone 62b) 15 yes 48 talk 51 62 now gonec) 15 yes 48 talk 51 now 62 goned) There will be no such stepe) None of the above

45. Step III of an input is: 21 victory 30 joint 64 47 all goneHow many more steps will be required to complete the rearrangement?a) Three b) Four c) Fived) Six e) None of these

46. Input : 89 bind 32 goal house 61 12 joyHow many steps will be required to complete the arrangement?a) Four b) Five c) Sixd) Seven e) None of these

47. Input: save 21 43 78 them early 36 forWhich of the following steps will be the last but one?a) VI b) VII c) VIIId) V e) None of these

48. Input: desire 59 63 all few 38 46 zoneHow many steps will be required to complete the rearrangement?a) Four b) Five c) Sixd) Seven e) None of these

49. Input: win 92 task 73 59 house range 34Which of the following will be step IV of the above input?a) 34 win 59 task 73 range 92 houseb) 34 win 92 59 task 73 house rangec) 34 win 92 task 73 59 house range

Page 60: Reasoning Made Easy - pdfstores.files.wordpress.com · REASONING MADE EASY 6 Geetika, Jasmine, Hema, Brinda, Ipsita and veena, once again not following any order. The following are

REASONING MADE EASY www.BankExamsToday.com

60

d) There will be no such stepe) None of the above

Directions (Q. 50-54) Study the following information carefully and answer thegiven questions.

A word and number arrangement machine when given an input line ofwords and numbers rearranges them following a particular rule in each step. Thefollowing is an illustration of input and rearrangement.

Input: base 35 or gone 62 49 87 aheadStep I: 87 base 35 or gone 62 49 aheadStep II: 87 ahead base 35 or gone 62 49Step III: 87 ahead 62 base 35 or gone 49Step IV: 87 ahead 62 base 49 35 or goneStep V: 87 ahead 62 base 49 gone 35 or

And Step V is the last step of the rearrangement. As per the rules followed in theabove steps, find out in each of the following question the appropriate step for thegiven input.

50. Input: how was your stay 56 25 36 64Which of the following will be step VI?a) 64 how 56 was your stay 25 36b) 64 how 56 stay 36 was 25 yourc) 64 how 56 stay 36 was your 25d) There will be no such stepe) None of the above

51. Input: power fail now 52 24 75 gate 34Which of the following steps will be the last but one?a) IV b) V c) VId) VII e) None of these

52. Step III of an input is: 91 car 85 14 27 few new houseWhich of the following is definitely the input?a) 85 14 91 car 27 few new houseb) car 91 85 14 27 few new housec) car 85 14 27 few new house 91d) Cannot be determinede) None of the above

53. Step II of an input is: 75 down 16 24 farm eager 62 skyHow many more steps will be required to complete the rearrangement?a) Four b) Five c) Sixd) Seven e) None of these

Page 61: Reasoning Made Easy - pdfstores.files.wordpress.com · REASONING MADE EASY 6 Geetika, Jasmine, Hema, Brinda, Ipsita and veena, once again not following any order. The following are

REASONING MADE EASY www.BankExamsToday.com

61

54. Input: 14 35 when they same 61 48 homeHow many steps will be required to complete the rearrangement?a) Four b) Five c) Sixd) Seven e) None of these

Directions (Q. 55-60) A word and number aran gement machine when given aninput line of words and numbers rearranges them following a particular rulein each step. The following is an illustration of input and rearrangement.

Input: but 32 71 glory fair south 65 84Step I: south but 32 71 glory fair 65 84Step II: south 84 but 32 71 glory fair 65Step III: south 84 glory but 32 71 fair 65Step IV: south 84 glory 71 but 32 fair 65Step V: south 84 glory 71 fair but 32 65Step VI: south 84 glory 71 fair 65 but 32

And Step VI is the last step of the rearrangement.

As per the rules followed in the above steps, find out in each of the followingquestions the appropriate step for the given input.

55. Step III of an input is: year 92 ultra 15 23 strive house 39. How many moresteps will be required to complete the rearrangement?a) Three b) Four c) Twod) Five e) None of these

56. Input: any how 49 24 far wide 34 69Which of the following steps will be the last but one?a) VI b) VII c) Vd) VIII e) None of these

57. Step II of an input is: town 74 pair 15 31 nice job 42Which of the following is definitely the input?a) pair 15 31 town nice job 42 74b) pair 15 town 31 74 nice job 42c) pair 15 town 74 31 nice job 42d) Cannot be determinede) None of the above

58. Input: play over 49 37 12 match now 81Which of the following will be step IV?a) play 81 over 49 37 match nowb) play 8 over 49 37 12 match nowc) play 81over 49 now 37 match 12d) There will be no such step

Page 62: Reasoning Made Easy - pdfstores.files.wordpress.com · REASONING MADE EASY 6 Geetika, Jasmine, Hema, Brinda, Ipsita and veena, once again not following any order. The following are

REASONING MADE EASY www.BankExamsToday.com

62

e) None of the above

59. Step II of an input is: war 58 box cart 33 49 star 24Which of the following steps will be the last?a) V b) VI c) IVd) VII e) None of these

60. Input: shower fall water 34 51 67 98 goalHow many steps will be required to complete the rearrangement?a) Three b) Four c) Sixd) Five e) None of these

Directions (Q. 61-66) Study the following information carefully and answer thegiven questions.

A word and number arrangement machine when given an input line of words andnumbers rearranges them following a particular rule in each step. The following isan illustration of input and rearrangement.

Input: joy far 35 27 16 96 height starStep I: 96 joy far 35 27 16 height starStep II: 96 far joy 35 27 16 height starStep III: 96 far 35 joy 27 16 height starStep IV: 96 far 35 height joy 27 16 starStep V: 96 far 35 height 27 joy 16 star

And step V is the last step of the rearrangement.

As per the rules followed in the abov steps, find out in each of the followingquestions the appropriate step for the given input.

61. Input: organize 19 12 stable room 35 72 houseHow many steps will be required to complete the arrangement?a) Five b) Six c) Sevend) Four e) None of the above

62. Input: bake never store 51 26 33 age 49Which of the following will be step V?a) 51 age 49 bake 33 never 26 storeb) 51 age 49 bake never store 26 33c) 51 age bake never store 26 33 49d) 51 bake never store 26 33 age 49e) There will be no such step

63. Input: always go there 39 62 47 time 24Which of the following steps will be the last but one?

Page 63: Reasoning Made Easy - pdfstores.files.wordpress.com · REASONING MADE EASY 6 Geetika, Jasmine, Hema, Brinda, Ipsita and veena, once again not following any order. The following are

REASONING MADE EASY www.BankExamsToday.com

63

a) VI b) VII c) VIIId) IX e) None of these

64. Step II of an input is: 67 ask 34 12 46 for my dateWhich o f the f ollowing is definitely the input?a) 34 12 46 for my date ask 67b) 34 12 46 for my date 67 askc) 12 34 67 ask 46 for my dated) Cannot be determinede) None of the above

65. Step III of an input is: 84 for 56 29 17 won loss gameWhich of the following steps will be the last?a) VIII b) IX c) VIId) V e) None of these

66. Step III of an input is: 86 box 63 18 gear card 51 newHow many more steps will be required to complete the arrangement?a) Three b) Two c) Fourd) Five e) None of these

Directions (Q. 67-71) Given an input line the machine rearranges them step bystep as per the illustration given below.

Input: show must 73 85 go 69 on 32Step I: 32 show must 73 85 go 69 onStep II: 32 go show must 73 85 69 onStep III: 32 go 69 show must 73 85 onStep IV: 32 go 69 must show 73 85 onStep V: 32 go 69 must 73 show 85 onStep VI: 32 go 69 must 73 on show 85Step VII: 32 go 69 must 73 on 85 show

Step VII is the last step and the arrangement in Step VII is the final arrangement.As per the rules followed in the above steps, find out in each of the followingquestions the appropriate step for the given input.

67. If the third step of an input is “14 but 26 not just 63 fundamental 47”, whichof the following is definitely the input?a) 26 14 but not just 63 fundamental 47b) 63 26 but 14 not just 63 fundamental 47c) fundamental 26 but not just 63 47d) Cannot be determinede) None of the above

Page 64: Reasoning Made Easy - pdfstores.files.wordpress.com · REASONING MADE EASY 6 Geetika, Jasmine, Hema, Brinda, Ipsita and veena, once again not following any order. The following are

REASONING MADE EASY www.BankExamsToday.com

64

68. First step of an input is “34 fire dress well 63 43 prime 52”. Which step isthe last but one?a) IV b) III c) VId) VII e) V

69. What will be the fourth step for the following input?Input: just in time for 36 48 14 59a) 14 for 36 in just time 48 59b) 14 for 36 just in time 48 59c) 14 for 36 in 48 just time 59d) Cannot be determined

e) None of the above

70. If the second step of an input is “23 fine 49 sixty rely 38 56 next”, howmany more steps will be r equired t o complete the arrangement?a) 2 b) 5 c) 6d) 4 e) None of these

71. If the second step for an input is “17 do foreign 95 74 heights mountain29”, what will be the fifth step?a) 17 do 29 foreign heights 74 95 mountainb) 17 do 29 foreign 74 mountain heights 95c) 17 do 29 foreign 74 heights 95 mountaind) There will be no such stepe) None of the above

Answers:

1. Option C2. Option E3. Option A4. Option B5. Option D6. Option D7. Option A8. Option B9. Option C10. Option E11. Option B12. Option B13. Option B14. Option B15. Option A16. Option D

Page 65: Reasoning Made Easy - pdfstores.files.wordpress.com · REASONING MADE EASY 6 Geetika, Jasmine, Hema, Brinda, Ipsita and veena, once again not following any order. The following are

REASONING MADE EASY www.BankExamsToday.com

65

17. Option C18. Option C19. Option C20. Option B21. Option A22. Option A23. Option D24. Option C25. Option D26. Option C27. Option A28. Option B29. Option C30. Option E31. Option C32. Option C33. Option B34. Option B35. Option B36. Option B37. Option C38. Option C39. Option D40. Option A41. Option B42. Option C43. Option E44. Option C

Step II: 15 yes 62 51 48 talk now goneStep III: 15 yes 48 62 51 talk now goneStep IV: 15 yes 48 talk 62 51 now goneStep V: 5 yes 48 talk 51 62 now goneStep VI: 15 yes 48 talk 51 now 62 gone

45. Option E

Step III: 21 victory 30 joint 64 47 all goneStep IV: 21 victory 30 joint 47 64 all goneStep V: 21 victory 30 joint 47 gone 64 allSo, step V is the last step. Hence, two steps will be required.

46. Option C

Input: 89 bind 32 goal house 61 12 joyStep I: 12 89 bind 32 goal house 61 joy

Page 66: Reasoning Made Easy - pdfstores.files.wordpress.com · REASONING MADE EASY 6 Geetika, Jasmine, Hema, Brinda, Ipsita and veena, once again not following any order. The following are

REASONING MADE EASY www.BankExamsToday.com

66

Step II: 12 joy 89 bind 32 goal house 61Step III: 12 joy 32 89 bind goal house 61Step IV: 12 joy 32 house 89 bind goal 61Step V: 12 joy 32 house 61 89 bind goalStep VI: 12 joy 32 house 61 goal 89 bindStep VI is the last step. So, to complete the arrangement, six steps will berequired.

47. Option E

Input: save 21 43 78 them early 36 forStep I: 21 save 43 78 them early 36 forStep II: 21 them save 43 78 early 36 forStep III: 21 them 36 save 43 78 early forStep IV: 21 them 36 save 43 for 78 earlyStep IV is the last step and from last step first one is step IV.

48. Option B

Input: desire 59 63 all few 38 46 zoneStep I: 38 desire 59 63 all few 46 zoneStep II: 38 zone desire 59 63 all few 46Step III: 38 zone 46 desire 59 63 all fewStep IV: 38 zone 46 few d esire 59 63 allStep V: 38 zone 46 few 59 desire 63 allSo, the last step is step V.

49. Option E

Input: win 92 task 73 59 house range 34Step I: 34 win 92 task 73 59 house rangeStep II: 34 win 59 92 task 73 house rangeStep III: 34 win 59 task 92 73 house rangeStep IV: 34 win 59 task 73 92 house range

50. Option D

Input: how was your stay 56 25 36 64Step I: 64 how was your stay 56 25 36Step II: 64 how 56 was your stay 25 36Step III: 64 how 56 stay was your 25 36Step IV: 64 how 56 stay 36 was your 25Step V: 64 how 56 stay 36 was 25 yourSo, sixth step is not possible because fifth step will be last step.

51. Option D

Page 67: Reasoning Made Easy - pdfstores.files.wordpress.com · REASONING MADE EASY 6 Geetika, Jasmine, Hema, Brinda, Ipsita and veena, once again not following any order. The following are

REASONING MADE EASY www.BankExamsToday.com

67

Input: power fail now 52 24 75 gate 34Step I: 75 power fail now 52 24 gate 34Step II: 75 fail power now 52 24 gate 34Step III: 75 fail 52 power now 24 gate 34Step IV: 75 fail 52 gate power now 24 34Step V: 75 fail 52 gate 34 power now 24Step VI: 75 fail 52 gate 34 now power 24Step VII: 75 fail 52 gate 34 now 24 powerSo, the first step from the last is step (VII).

52. Option D

Input cannot be determined.

53. Option A

Step II: 75 down 16 24 farm eager 62 skyStep III: 75 down 62 16 24 farm eager skyStep IV: 75 down 62 eager 16 24 farm skyStep V: 75 down 62 eager 24 16 farm skyStep VI: 75 down 62 eager 24 farm 16 skyFour more steps are required to complete the rearrangement.

54. Option C

Input: 14 35 when they came 61 48 homeStep I: 61 14 35 when they came 48 homeStep II: 61 came 14 35 when they 48 homeStep III: 61 came 48 14 35 when they h omeStep IV: 61 came 48 home 14 35 when theyStep V: 61 came 48 home 35 14 when theyStep VI: 61 came 48 home 35 they 14 whenTo complete the rearrangement 6 steps are required.

55. Option B

Step III: year 92 ultra 15 23 strive house 39Step IV: year 92 ultra 39 15 23 strive houseStep V: year 92 ultra 39 strive 15 23 houseStep VI: year 92 ultra 39 strive 23 15 houseStep VII: year 92 ultra 39 strive 23 house 15So, four more steps are required.

56. Option C

Page 68: Reasoning Made Easy - pdfstores.files.wordpress.com · REASONING MADE EASY 6 Geetika, Jasmine, Hema, Brinda, Ipsita and veena, once again not following any order. The following are

REASONING MADE EASY www.BankExamsToday.com

68

Input: any how 49 24 for wide 34 69Step I: wide any how 49 24 for 34 69Step II: wide 69 any how 49 24 for 34Step III: wide 69 how any 49 24 for 34Step IV: wide 69 how 49 any 24 for 34Step V: wide 69 how 49 for any 24 34Step VI: wide 69 how 49 f or 34 any 24So, step V is first step from last.

57. Option D

Step II: town 74 pair 15 31 nice job 42Input cannot be determined.

58. Option D

Input: play over 49 37 12 match now 81Step I: play 81 over 49 37 12 match nowStep II: play 81 over 49 now 37 12 matchStep III: play 81 over 49 now 37 match 124th step cannot be determined.

59. Option B

Step II: war 58 box cart 33 49 star 24Step III: war 58 star box cart 33 49 24Step IV: war 58 star 49 box cart 33 24Step V: war 58 star 49 cart box 33 24Step VI: war 58 star 49 cart 33 box 24So, last step is step VI.

60. Option D

Input: shower fall water 34 51 67 98 goalStep I: water shower fall 34 5167 98 goalStep II: water 98 shower fall 34 51 67 goalStep III: water 98 shower 67 fall 34 51 goalStep IV: water 98 shower 67 goal fall 34 51Step V: water 98 shower 67 goal 51 fall 34Five steps required to complete the rearrangement.

61. Option D

Input: organize 19 12 stable room 35 72 houseStep I: 72 organize 19 12 stable room 35 houseStep II: 72 house organize 19 12 stable room 35

Page 69: Reasoning Made Easy - pdfstores.files.wordpress.com · REASONING MADE EASY 6 Geetika, Jasmine, Hema, Brinda, Ipsita and veena, once again not following any order. The following are

REASONING MADE EASY www.BankExamsToday.com

69

Step III: 72 house 35 organize 19 12 stable roomStep IV: 72 house 35 organize 19 room 12 stableSo, four steps will required to complete the arrangement.

62. Option A

Input: bake never store 51 26 33 age 49Step I: 51 bake never store 26 33 age 49Step II: 51 age bake never store 26 33 49Step III: 51 age 49 bake never store 26 33Step IV: 51 age 49 bake 33 never store 26Step V: 51 age 49 bake 33 never 26 store

63. Option E

Input: always go there 39 62 47 time 24Step I: 62 always go there 39 47 time 24Step II: 62 always 47 go there 39 time 24Step III: 62 always 47 go 39 there time 24Step IV: 62 always 47 go 39 there 24 time

64. Option D

Input cannot be determined by given step.

65. Option D

Step III: 84 for 56 29 17 won loss gameStep IV: 84 for 56 game 29 17 won lossStep V: 84 for 56 game 29 loss 17 wonSo, the last step is step V.

66. Option A

Step III: 86 box 63 18 gear card 51 newStep IV: 86 box 63 card 18 gear 51 newStep V: 86 box 63 card 51 18 gear newStep VI: 86 box 63 card 51 gear 18 newSo, to complete the arrangement three steps are required.

67. Option D68. Option C69. Option A70. Option D71. Option C

Page 70: Reasoning Made Easy - pdfstores.files.wordpress.com · REASONING MADE EASY 6 Geetika, Jasmine, Hema, Brinda, Ipsita and veena, once again not following any order. The following are

REASONING MADE EASY www.BankExamsToday.com

70

Chapter - 3

Data Sufficiency

Directions (Q. 1-3) Each of the questions below consists of a question and threestatements numbered I, II and III given below it. You have to decide whetherthe data provided in the statements are suficient to answer the question.

Give Answer:1. How many daughters does W have?

I. B and D are sisters of M.II. M’s father T is husband of WIII. Out of the three children which T has, only one is a boy.

a) Only I and IIIb) All I, II and IIIc) Only II and IIId) Question cannot be answered even with all I, II and IIIe) Only I and II

2. Who among A, B, C, D, E and F each having a different height, is thetallest?I. B is taller than A but shorter than E.II. Only two of them are shorter than C.III. D is taller than only F.

a) Only I and IIb) Only I and IIIc) Only II and IIId) All I, II and III are required to answer the questione) All I, II and III are not sufficient to answer the question

3. Towards which direction is village J from village W?I. Village R is to the west of village W and to the north of village T.II. Village Z is to the east of village J and to the South of village T.III. Village M is to the noirth east of Village J and north of Village Z.

a) Only IIIb) Only II and IIIc) All I, II and III are required to answer the questiond) Question cannot be answered even with all I, II and IIIe) None of these

Page 71: Reasoning Made Easy - pdfstores.files.wordpress.com · REASONING MADE EASY 6 Geetika, Jasmine, Hema, Brinda, Ipsita and veena, once again not following any order. The following are

REASONING MADE EASY www.BankExamsToday.com

71

Directions (Q. 4-8) Below is given a passage followed by several possibleinferences which can drawn from the facts stated in the passage. You have toexamine each inference separately in the context of the passage and decideupon its degree of truth or falsity.

Give answer:a) If the inference is ‘definitely true’ i.e. it properly from the statement of facts

given.b) If the inference is ‘probably true’ through not ‘definitely true’ in the light of

the facts given.c) If the ‘data are inadequate’, i.e. from the facts given you cannot say whether

the inference is likely to be true or false.d) If the inference is ‘probably false’, though not ‘definitely false’ in the light

of the facts given.e) If the inference is ‘definitely false’, i.e. it cannot possibly be drawn from

the facts given or it contradicts the given facts.

Between 2002-03 and 2006-2007, Indian economy grew annualy at 8.7%led by the services sector at 9% per year. In 1990, India’s share of services, at 40%of GDP was consistent with its per capita income for low-income country. By 2001its share of one-half of the GDP was higher by five percentage points, compared tothe average for low-income countries. Economic reforms that energized the privatecorporate sector and technological changes that opened up new vistas intelecommunications, IT and outsourcing are believed to be responsible for th eimpressive performance. However, the services led growth remains a puzzle at alow per capita income, with 55% of the work force still engaged in agriculture, andwhen agriculture decelerated and industry stagnated-3 defying a styled fact ineconomics.

4. India has now emerged as a high per capita income country.

5. Growth in India’s services sector post 2005 is more than 9%.

6. Less than half of total workforce is engaged in Agricultural sector in India.

7. Share of services sector in India’s GDP has crossed the half way mark in early2000.

8. In early minutes, the share of services sector in GDP for low per capita incomegroup of countries is about 40%

Page 72: Reasoning Made Easy - pdfstores.files.wordpress.com · REASONING MADE EASY 6 Geetika, Jasmine, Hema, Brinda, Ipsita and veena, once again not following any order. The following are

REASONING MADE EASY www.BankExamsToday.com

72

Directions (Q. 9-13) below are given a passage followed by several possibleinferences which can be drawn from the facts stated in the passage. You haveto examine each in ference separately in the context of the passage and decideupon its degree of truth or falsity.

Give answer:a) If the inference is “definitely true” i.e. it properly follows from the

statements of facts given.b) If the inference is “probably true” thuogh not “definitely true” in the light of

the facts given.c) If the data are inadequate i.e. from the facts given you can not say whether

the inference is likely to be true or false.d) If the inference is “probably false” though not “definitely false” in the light

of the facts given.e) If the inference is “definitely false” i.e. it cannot possibly be drawn from the

facts given or it contradicts the given facts.

A recent report that satellite data shws groundwater levels in northern Indiadepleting by as much as a foot per year, over the past decade, is a matter of concern.The clear writing on the wall is that India faces a turbulent water future andveritable crisis without proactive policy and sustainable practices. Besides, the poormonsoon this season and the resultant drought situation pan-India, calls series ofglaring anomalies do need to be addressed. For one, there’s far too much reliance ongroundwater. For another, out water infrastructure for storage and supply is sorelyinadequate. Worse, policy distortions in artificially under pricing key agri-inputslike power have perversely incentivised cultivation of water intensive crops likepaddy in traditionally wheat growing areas.

9. India has failed to take measures to recharge groundwater adequately innorthern part.

10. Wheat cultivation in India requires comparatively more water than paddy.

11. Water level in other parts of India is stable during the last decade.

12. India has now put in place a system to reduce over dependence on groundwater.

13. Adequate monsoon helps in drawing less ground water for cultivation and thuspreserve balance.

In making decisions about important questions, it is desirable to be able todistinguish between “strong” arguments and “weak” arguments. “Strong”arguments must be both important and directly related to the question. “Weak”

Page 73: Reasoning Made Easy - pdfstores.files.wordpress.com · REASONING MADE EASY 6 Geetika, Jasmine, Hema, Brinda, Ipsita and veena, once again not following any order. The following are

REASONING MADE EASY www.BankExamsToday.com

73

arguments may not be directly related to the question and may be of minorimportance or m ay be related to the trivial aspects of the question.

Directions (Q. 14-18) Each of the questions below consists of a question andtwo statements numb ered I and II given below it. You have to decide whetherthe data provided in the statements are sufficient to answer the question. Readboth the statements and give answer:

a) If the data in Statement I alone are sufficient to answer the question, whilethe data in Statement II alone are not sufficient to answer the question.

b) If the data in Statement II alone are sufficient to answer the question, whilethe data in Statement I alone are not sufficient to answer the question.

c) If the data either in Statement I alone or in Statement II alone are sufficientto answer the question.

d) If the data even in both Statements I and II together are not sufficient toanswer the question.

e) If the data in both Statements I and II together are necessary to answer thequestion.

14. What is the position of point F with respect to point I?I. Point G is 5 km east of point F. Point S is 5 km north of point G.

Point H is the midpoint of points G and S. Point I is to the south ofpoint H in such a manner that point G is the midpoint of points Hand I.

II. Point A is 10 km east of point F. Point B is 5 km south of point A.Point H is the midpoint of points A and B. Point I is 5 km south ofpoint H. Point I is to the east of point L at a distance of 5 km.

15. On which day of a week Lalita did her reservation for the journey?I. The husband of Lalita correctly remembers that she has done

reservation after Monday but before Sunday. However, it is almostcertain that she did not make her reservation on Wednesday.

II. The daughter of Lalita correctly remembers that h er mother hasdone her reservation after Tuesday but before Saturday. Since hermother was busy on Thursday so she did not make her reservationon Thursday.

16. Seven persons - P, Q, R, S, T, U and V are standing in a straight line facingtowards north. Find the position of S from the right end.I. P is standing on any of the extreme ends of the line. There are two

persons between P and R. There are two persons between R and Q.U is not standing to the immediate left of R.

II. R is standing exactly at the centre. T is standing exactly between Rand V. There is only one person between T and U. S is standing tothe immediate right of U.

Page 74: Reasoning Made Easy - pdfstores.files.wordpress.com · REASONING MADE EASY 6 Geetika, Jasmine, Hema, Brinda, Ipsita and veena, once again not following any order. The following are

REASONING MADE EASY www.BankExamsToday.com

74

17. How are N and D related?I. A is the sister of D.II. D is the mother of R who is the only son of N.

18. Who amongst Ronak, Sanjay, Anish and Rohan is the tallest?I. Ronak is as tall as Sanjay.II. Anish is taller than Rohan.

Answers:

1. Option C2. Option C3. Option E4. Option A5. Option B6. Option D7. Option A

It is true as given in second line of the passage “In 1990, India’s share ofservices at 40% of GDP ____

8. Option D9. Option D10. Option B11. Option A12. Option E13. Option E14. Option C15. Option E

From Statement ILalita made her reservation on either Tuesday, Thursday, Friday orSaturday.From Statement IILalita made her reservation on Wednesday or Friday.From both the statements Lalita made her reservation on Friday.

16. Option B17. Option B

From statement IA is the sister of D. There is no information about N.From statement IID is the mother of R.R is the son of N.So, N is husband of D.

18. Option DFrom statement IRonak = SanjayFrom statement II

Page 75: Reasoning Made Easy - pdfstores.files.wordpress.com · REASONING MADE EASY 6 Geetika, Jasmine, Hema, Brinda, Ipsita and veena, once again not following any order. The following are

REASONING MADE EASY www.BankExamsToday.com

75

Anish > RohanFrom both the statementsAnish > RohanRonak = Sanjay

Page 76: Reasoning Made Easy - pdfstores.files.wordpress.com · REASONING MADE EASY 6 Geetika, Jasmine, Hema, Brinda, Ipsita and veena, once again not following any order. The following are

REASONING MADE EASY www.BankExamsToday.com

76

Chapter - 4

Sitting Arrangements

Directions (Q. 1-5) Study the following information to answer the givenquestions.

i) There are 9 friends A, B, C, D, E, F, G, H seated in a circle facing thecentre.

ii) AC, DG, HE and FB are seated adjacent to each other. A is also seatedadjacent to H.

iii) B is 2nd to the right of H.iv) E is 3rd to the right of C.

1. Who is 2nd to the left of A?a) D b) G c) Fd) Cannot be determined e) None of these

2. Who is 3rd to the left of C?a) G b) D c) Bd) Cannot be determined e) None of these

3. What is C’s position with reference to E?a) 5th to the right b) 4th to the left c) 4th to the rightd) 3rd to the right e) Cannot be determined

4. Who is 2nd to the right of A?a) B b) E c) Fd) Cannot be determined e) None of these

5. Who among the following pairs may not be seated adjacent to each other?a) AH b) DC c) EBd) Cannot be determined e) None of these

Directions (Q. 6-11) Read the following information carefully and answer thequestions that follow.

Seven friends A, B, C, D, E, F and G are sitting around a circular tablefacing either the centre or outside. Each one of them belongs to a differentdepartment viz. Finance, Marketing Sales, HR, Corporate Finance, InvestmentBanking and Operations but not necessarily in the same order.

C sits third to the right of G. G faces the centre. Only one person sitsbetween C and the person working in the HR department immediate neighbours ofC face outside. Only one person sits between F and D. Both F and D face the centre.D does not work in the HR department. A works in Investment Banking

Page 77: Reasoning Made Easy - pdfstores.files.wordpress.com · REASONING MADE EASY 6 Geetika, Jasmine, Hema, Brinda, Ipsita and veena, once again not following any order. The following are

REASONING MADE EASY www.BankExamsToday.com

77

Department. A faces the centre. Two people sit between the persons who work inInvestment Banking and Marketing Departments. The person who works inCorporate Finance sits to the immediate left of E. C faces same direction as E. Theperson who works in corporate finance sits to the immediate left of the person whoworks for Operations Department.

6. For which of the following departments does B work?a) Finance b) Marketing c) HRd) Corporate Finance e) Operationsd

7. What is position of B with respect to the person who works for SalesDepartment?a) Immediate right b) Third to the left c) Second to the rightd) Second to the left e) Fourth to the right

8. Who sits to the immediate right of E?a) The person who works for Marketing Departmentb) Cc) Bd) The person who works for HR Departmente) A

9. Who amongst the following sits exactly between C and the person whoworks for HR Department?a) Bb) The person who works for Marketing Departmentc) The person who works for Operations Departmentd) De) G

10. Who amongst the following sit between the persons who work forMarketing and Investment Banking departments when counted for the lefthand side of the person working for Marketing Department?a) F and G b) E and C c) C and Bd) F and D e) B and D

11. How many people sit between the person who workds for OperationsDepartment and A, when counted from the right hand side of A?a) One b) Two c) Threed) Four e) None of these

Directions (Q. 12-16) Read the following information carefully to answer thegiven questions.

Eight persons from different banks viz., Bank of India, Punjab NationalBank, Canara Bank, Bank of Baroda, Oriental Bank of Commerce, Dena Bank,

Page 78: Reasoning Made Easy - pdfstores.files.wordpress.com · REASONING MADE EASY 6 Geetika, Jasmine, Hema, Brinda, Ipsita and veena, once again not following any order. The following are

REASONING MADE EASY www.BankExamsToday.com

78

Union Bank of India and Bank of Maharashtra are sitting in two parallel rowscontaining four people each, in such a way that there is an equal distance betweenadjacent persons. The names of these persons are A, B, C, D, E, F, G and H. Eachmembers seated in a row faces another member of the other row. (All theinformation given above does not necessarily represents the order of seating as inthe final arrangement)

A faces north and seated second to the left of the person, who is fromPunjab National Bank. The person from Oriental Bank of Commerce isopposite to one who is second to the left of H.

The person from Bank of Baroda is not near to H and F and not at the endof the row. The person from Canara Bank is opposite to the person who isfrom Dena Bank.

The person from Bank of India is not at the end of the row but left of theperson who is from Canara Bank.

D is immediately right of the person who is from Bank of Maharashtra. Theperson from Canara Bank faces north while D faces south and opposite toB.

The person from Union Bank of India is not opposite to E. The person from CanaraBank is opposite to C.

12. Which of the following is true regarding E?a) The person from Bank of Maharashtra faces Eb) The person from Dena Bank is an immediate neighbour of Ec) The person who is second to the right of G faces Ed) E is from Union Bank of Indiae) E sits at one of the extreme end of the row

13. Who is seated between G and the person from Bank of Maharashtra?a) The person from Bank of Barodab) Hc) Fd) Ee) The person from Dena Bank

14. Who amongst the following sit at extrememe end of the row?a) A and the person from Bank of Maharashtrab) The person from Punjab National Bank and Union Bank of Indiac) The person from Dena Bank and Ed) The person from Union Bank of India and Ce) G and B

15. Who amongst the following faces the person from Bank of Baroda?a) The person from Bank of Indiab) Fc) A

Page 79: Reasoning Made Easy - pdfstores.files.wordpress.com · REASONING MADE EASY 6 Geetika, Jasmine, Hema, Brinda, Ipsita and veena, once again not following any order. The following are

REASONING MADE EASY www.BankExamsToday.com

79

d) The person from Union Bank of Indiae) The person from Oriental Bank of Commerce

16. ‘B’ is related to ‘Dena Bank’ in the same way as ‘F’ is related to ‘Bank ofBaorda’ on the given arrangement, who amongst the following is ‘G’related to following the same pattern?a) Oriental Bank of Commerceb) Punjab National Bankc) Bank of Indiad) Canara Banke) Union Bank of India

17. Four of the following five are alike in a certain way based on the givenseating arrangement and thus form a group, which is the one that does notbelong to that group?a) Canara Bankb) Cc) Union Bank of Indiad) Oriental Bank of Commercee) Bank of Maharashtra

18. G is from which of the following banks?a) Oriental Bank of Commerceb) Dena Bankc) Bank of Maharashtrad) Canara Banke) Punjab National Bank

19. Who amongst the following is from Punjab National Bank?a) E b) H c) Gd) C e) None of these

Directions (Q. 20-26) Study the following information carefully to answer thequestions that follows.

Eight persons namely A, B, C, D, E, F, G and H are sitting in a straight linefrom East to West. Some are facing North direction and some are facing Southdirection.

H sits to the third left of C and faces south. F is sitting at one of the extremeends and is facing north. B and G are facing in same direction and person sittingbetween them is facing in opposite direction. A is sitting third from left end and isfacing South direction. A is sitting adjacent to E and both are facing in oppositedirection. D is sitting to the right of B and both are facing in same direction. C isfacing in North direction. A and D are not neighbours. Neighbors of H are facing insame direction which is opposite to the direction of H is facing. E sits to the right ofA.

Page 80: Reasoning Made Easy - pdfstores.files.wordpress.com · REASONING MADE EASY 6 Geetika, Jasmine, Hema, Brinda, Ipsita and veena, once again not following any order. The following are

REASONING MADE EASY www.BankExamsToday.com

80

20. Who is sitting between A and H?a) F b) E c) Gb) B e) None of these

21. Which of the following pairs has the person facing the same direction?a) E, A b) G, H c) A, Gd) H, D e) B, D

22. How many persons are facing in North direction?a) 2 b) 3 c) 4d) 5 e) 6

23. Who is sitting third from the eastern end of the arrangement?a) E b) G c) Bd) C e) None of these

24. How many persons are sitting between E and B?a) 2 b) 3 c) 4d) 5 e) 6

25. Which one of the following statement is correct?a) A sits between G and Bb) D sits to thesecond left of Hc) H and B are facing same directiond) Person sitting to the left of A faces directione) All of the above

26. Who sits 3rd to the left of A?a) B b) D c) Hd) G e) None of these

Directions (Q. 27-31) Study the following information and answer the questiongiven below.

Eight people E, F, G, H, J, K, L and M are sitting around a circular tablefacing the centre. Each of them is of a different professio n Chartered Accountant,Colomnist, Doctor, Engineer, Financial Analyst, Lawyer, Professor and Scientistbut not necessarily in the same order. F is sitting second to the left of K. TheScientist is an immediate neighbour of K. There are only three people between theScientist and E. Only one person sits between the Engineer and E. The Coloumnistis to t he immediate r ight of the Engineer. M is second to the right of K. H is theScientist. G and J are immediate neighbours of each other. Neither G nor J is anEngineer. The Financjial Analyst is to the immediate left of F. The Lawyer issecond to the right of the Coloumnist. The Professor is an immediate neighbour ofthe Engineer. G is second to the right of the Chartered Accountant.

Page 81: Reasoning Made Easy - pdfstores.files.wordpress.com · REASONING MADE EASY 6 Geetika, Jasmine, Hema, Brinda, Ipsita and veena, once again not following any order. The following are

REASONING MADE EASY www.BankExamsToday.com

81

27. Who is sitting second to the right of E?a) The Lawyer b) G c) The Engineerd) F e) K

28. Who amongst the following is the Professor?a) F b) L c) Md) K e) None of these

29. Four of the following five are alike in a certain way based on the givenarrangement and hence from a group. Which of the following does notbelong to that group?a) Chartered Accountantb) M – Doctorc) J – Engineerd) Financial Analyst – Le) Lawyer - K

30. What is the position of L with respect to the Scientist?a) Third to the leftb) Second to the rightc) Second to the leftd) Third to the righte) Immediate right

31. Which of the following statements is true according to the givenarrangement?a) The Lawyer is second to the left of the Doctorb) E is an immediate neighbour of the Financial Analystc) H sits exactly between F and the Financial Analystd) Only four people sit between the Coloumnist and Fe) All of the given statements are true

Directions (Q. 32-36) Study the following information carefully and answer thegiven questions:-

D, E, F, H and I are seated in a circle facing the centre. A, B and C are alsoseated in the same circle but two of them are not facing the centre. F is second to theleft of C. E is third to the right of A. B is third to left of D, who is immediateneighbor of H and I. C is second to the right of D and third to the right of B.

32. Which of the following pairs is not facing the centre?a) BC b) AC c) CAd) BD e) None of these

33. What is the position of D with respect to F?

Page 82: Reasoning Made Easy - pdfstores.files.wordpress.com · REASONING MADE EASY 6 Geetika, Jasmine, Hema, Brinda, Ipsita and veena, once again not following any order. The following are

REASONING MADE EASY www.BankExamsToday.com

82

a) Second to the rightb) Third to the leftc) Fifth to the rightd) Cannot be determinede) None of these

34. Who among the following is second to the left of B?a) A b) H c) Id) Either H or I e) None of these

35. If H is on the immediate right of E then what is H’s position with respect toC?a) Third to the leftb) Third to the rightc) Fifth to the leftd) Cannot be determinede) None of these

36. Which of the following is/are the possible position(s) of I with respect to A?a) Fourth to the rightb) Second to the leftc) Fourth to the leftd) Only a and ce) All of the above

Directions (Q. 37-41) Study the following information and answer the questionsthat follow:

Eight students A, B, C, D, E, F, G and H in a school sports competitionparticipated in a game in which they were sitting around a circulating. The seats ofthe ring are not directed towards the centre. All the eight students are in four groupsI, II, IIII and IV, i.e two students in each group, but not necessarily in the sameorder. These students are from different sport houses, viz. Maharishi Vyas,Aryabhatt, Vashistha, Shankaracharya, Balmiki, Dhruv and Dayanand.

No two students of the same group are sitting adjacent to each other exceptthose of group III. Students from group IV are sitting opposite each other.

D is neither in Dayanand nor in Aryabhatt house. The student from Dhruv house is sitting on the immediate right of the

students from Dayanand house. C, who is in Vashistha house, is in group I. She is sitting on the immediate

right of F, who is in group III. F is not in Aryabhatt house and she has also participated in other sports. B from Vyas house is neither in group IV nor in group I or II.

Page 83: Reasoning Made Easy - pdfstores.files.wordpress.com · REASONING MADE EASY 6 Geetika, Jasmine, Hema, Brinda, Ipsita and veena, once again not following any order. The following are

REASONING MADE EASY www.BankExamsToday.com

83

B is sitting opposite E. Only Balmiki participant A is sitting b etweenDhruv participaint E and t he Shankaracharya participant.

Both the students of group II are sitting adjacent to students of group IV.

37. Who is in Maharishi house?a) G b) F c) Hd) Can’t say e) None of these

38. ‘H’ is in which house?a) Dayanand b) Shankaracharya c) Maharishid) Can’t say e) None of these

39. Who among the following students are in group IV?a) A and B b) A and C c) G and Hd) Data inadequate e) None of these

40. The student from which house is sitting opposite the Vashistha houseparticipant?a) Dayanand b) Shankracharya c) Maharishid) Can’t say e) None of these

41. Which of the following statements is false?a) The Balmiki representative is sitting opposite the Maharishi houserepresentativeb) The Vyas house representative is sitting opposite the Dhruv houserepresentativec) H and G are sitting opposite each otherd) The two girl participants are sitting opposite each othere) None of these

Directions (Q. 42-46) Read the following information carefully and answer thequestioins that follow.

Twelve friends are sitting in two squares. One square is inside another. P,Q, R, S, T and V are in the outer square facing inward. A, B, C, D, E and F aresitting in the inner square facing outward. They all are s itting in such a way that ineach square four persons are sitting in the middle of the sides and two persons aresitting on diagonally opposite corners. Each friend in the inner square is facinganother friend of the outer square. There are exactly two persons sitting between Pand T. E si ts second to the left of A. R is on the immediate left of the one who is faci ng A. Neither E nor A faces either T or P. S is facing F and Q is not opposite Rin the outer square. T is not sitting adjacent to S. B is not facing P. Between B andD there are as many persons as between R and V.

42. Who is sitting opposite B?

Page 84: Reasoning Made Easy - pdfstores.files.wordpress.com · REASONING MADE EASY 6 Geetika, Jasmine, Hema, Brinda, Ipsita and veena, once again not following any order. The following are

REASONING MADE EASY www.BankExamsToday.com

84

a) V b) Q c) Td) Data inadequate e) None of these

43. V is facing which of the following persons?a) E b) D c) Ad) Data inadequate e) None of these

44. Who is sitting on the immediate left of R?a) A b) S c) Pd) Data inadequate e) None of these

45. Four of the five are alike in a certain manner. Select the one which is notsimilar to the other four.a) R, E b) A, S c) D, Td) F, Q e) C, E

46. Who is sitting second to the left of D?a) A b) E c) Fd) Data inadequate e) None of these

Directions (Q. 47-51) Study the following information carefully and answer thegiven questions:

E, F, G, H, I, J, K and L are eight feiends sitting around a circle facing thecentre, but not necessarily in the same order.

I is second to the left of F, who is next to the right of L. J is not theneighbour of F or K, and is on the immediate left of H.

47. Which of the following pairs represents the neighbours of G?a) H, F b) E, H c) J, Kd) I, E e) None of these

48. Who among the following is on the immediate left of J?a) K b) H c) Id) L e) None of these

49. Who among the following is on the immediate right of E?a) F b) L c) Gd) H e) None of these

50. Which of the following statements is true with respect to H?a) Immediate left of Jb) Opposite Lc) Second to the right of Fd) All are true

Page 85: Reasoning Made Easy - pdfstores.files.wordpress.com · REASONING MADE EASY 6 Geetika, Jasmine, Hema, Brinda, Ipsita and veena, once again not following any order. The following are

REASONING MADE EASY www.BankExamsToday.com

85

e) None of these

51. How many persons sit between G and I?a) None b) One c) Twod) Three e) None of these

Directions (Q. 52-56) Study the given information carefully and answer thegiven questiions.

Eight people – A, B, C, D, E, F, G and H are sitting around a circular tablefacing the centre, not necessarily in the same order. Three people are sittingbetween A and D. B is sitting second to the right of A. C is to the immediate right ofF. D is not an immediate neighbour of either F or E. H is not an immediateneighbour of B.

52. What is E’s position with respect to G?a) Third to the leftb) Second to the rightc) Third to the rightd) Second to the lefte) None of these

53. Four of the following are based on ab ove arrangement and so form a group.Which one does not belong to the group?a) GE b) DC c) AFd) AB e) CE

54. Who is sitting third to the right of the one who is sitting to the immediateright of H?a) A b) B c) Ed) C e) G

55. Which of the following is true regarding the given arrangement?a) E is second to the left of Cb) B is an immediate neighbour of Gc) H is an immediate neighbour of Ad) D is not an immediate neighbour of He) None of these

56. How many people are sitting between H and A when counted from the rightside of H?a) Three b) None c) More than threed) One e) Two

Page 86: Reasoning Made Easy - pdfstores.files.wordpress.com · REASONING MADE EASY 6 Geetika, Jasmine, Hema, Brinda, Ipsita and veena, once again not following any order. The following are

REASONING MADE EASY www.BankExamsToday.com

86

Directions (Q. 57-61) Study the following Information to answert thesequestions.

(i) P, Q, R, S, T, U and V are sitting along the circle facing the centre.(ii) T is 2nd to t he left of P.(iii) R is 3rd to the right of U who is the immediate left of V.(iv) Q is between P and S.

57. Which of the following is true statement?a) P is between Q and Tb) R is between P and Vc) P is 4th to the right of Ud) U is between V and Qe) None of these

58. Which of the following pairs has its second member sitting to theimmediate left of first member?a) VT b) PQ c) SUd) UV e) None of these

59. Which of the following is the false statement?a) R is to the immediate left of Pb) V is to the immediate right of Tc) R is 4th to the right of Sd) T is between V and Re) None of these

60. What is the position of S?a) To the immediate left of Ub) Between V and Qc) 3rd to the right of Td) To the immediate right of Pe) None of these

61. Which of the following pairs has members sitting adjacent to each other?a) PS b) QU c) UTd) TR e) None of these

Directions (Q. 62-66) Study the following information and answer the questionswhich follow.

(i) K, L, M, N, O, P and Q are sitting around a circular table facing thecentre.

(ii) L sits between N and O.(iii) K is third to the left of O.(iv) Q is second to the left of M, who is to the immediate left of P.

Page 87: Reasoning Made Easy - pdfstores.files.wordpress.com · REASONING MADE EASY 6 Geetika, Jasmine, Hema, Brinda, Ipsita and veena, once again not following any order. The following are

REASONING MADE EASY www.BankExamsToday.com

87

62. Which is the correct position of L with respect to Q?a) Second to the rightb) First to the leftc) First to the rightd) Third to the righte) None of these

63. Which of the following pairs has the first person sitting to the immediateleft of the second person?a) LO b) MK c) QNd) LN e) None of these

64. Who sits second to the left of L?a) P b) M c) K

d) Q e) None of these

65. Who sits between M and Q?a) O b) N c) Kd) L e) None of these

66. Which of the following has the middle person sitting between the othertwo?a) NQL b) PMK c) MOP

d) POK e) None of these

Directions (Q. 67-72) Study the follow ing information carefully and answerthe questions given below.

A, B, C, D, E, F, G and H are sitting around a circle facing at the centre. Bis second to the right of H and third to the left of A. D is not an immediateneighbuor of either B or H and is second to the right of F. C is fourth to the right ofG.

67. If E and F interchange their places, who will be second to the right of B?a) F b) C c) Dd) Data inadequate e) None of these

68. In which of the following pairs is the second person sitting to the immediateleft of the first person?a) BC b) HE c) FAd) GD e) None of these

69. Who is 4th to the right of H?a) B b) A c) Fd) Data inade quate e) None of these

Page 88: Reasoning Made Easy - pdfstores.files.wordpress.com · REASONING MADE EASY 6 Geetika, Jasmine, Hema, Brinda, Ipsita and veena, once again not following any order. The following are

REASONING MADE EASY www.BankExamsToday.com

88

70. Who is to the immediate right of B?a) C b) E c) Hd) Data inadequate e) None of these

71. Who is third to the right of E?a) C b) F c) Dd) Data inadequate e) None of these

72. In a row of 45 boys facing South, T is 8th to the right of H, who is 10th fromthe right end. H is 14th to the left of R. What is R’s position from the leftend?a) 21st b) 23rd c) 24th

d) Data inadequate e) None of these

Directions (Q. 73-78) Study the following information carefully and answer thequestions given below:

P, M, D, A, F, H, R and B are sitting around a circle facing at the centre. Ris 4th to the right of A, who is 3rd to the right of P. M is second t o the l eft of H, whois 2nd to the left of P. D is 3rd to the right of B.

73. Who is 3rd to the left of H?a) M b) A c) Bd) F e) Data inadequate

74. Which of the following pairs represents the immediate neighbours of B?a) FM b) AP c) MPd) AF e) None of these

75. If R and B interchange their positions, who will be 2nd to the left of M?a) B b) H c) Pd) Data inadequate e) None of these

76. Who is the immediate right of P?a) F b) R c) Bd) Data inadequate e) None of these

77. Who is the immediate right of M?a) H b) A c) Dd) Data inadequate e) None of these

78. In a row of 40 children, Q is 14th from the left end and there are 16 childrenbetween Q and M. What is M’s position from the right end of the row?a) 11th b) 10th c) 30th

d) Data inadequate e) None of these

Page 89: Reasoning Made Easy - pdfstores.files.wordpress.com · REASONING MADE EASY 6 Geetika, Jasmine, Hema, Brinda, Ipsita and veena, once again not following any order. The following are

REASONING MADE EASY www.BankExamsToday.com

89

Directions (Q. 79-83) Study the following information carefully and answer thequestions given below.

B, D, M, K, P, Q, W and H are sitting around a circle facing at the centre.M is to the immediate right of B who is 4th to the right of K. P is 2nd to the left of Band is 4th to the right of W. Q is 2nd to the right of D who is 2nd to the right of H.

79. Who is 3rd to the right of B?a) W b) M c) Kd) H e) None of these

80. Which of the following represents the immediate neighbours of D?a) PQ b) KH c) PHd) KQ e) PK

81. Who is 3rd to the right of W?a) P b) D c) Kd) R e) Data inadequate

82. Who is 2nd to the left of P?a) D b) H c) Kd) Data inadequate e) None of these

83. Who is to the immediate left of B?a) Q b) P c) Wd) Data inadequate e) None of these

Directions (Q. 84-88) Study the following information carefully and answer thequestions given below.

A, M, D, P, R, T, B and H are sitting around a circle facing at the centre. Mis 3rd to the left of Awho is 2nd to the left of T. D is 2nd to the right of H who is 2nd tothe right of T. R is 2nd to the right of B who is not an immediate neighbour of T.

84. Which of the following combinations represents the first and the second tothe left of B respectively?a) MD b) DH c) AMd) AR e) DM

85. Who is 3rd to the right of T?a) D b) B c) Hd) M e) None of these

86. Who is to the immediate left of H?a) P b) M c) T

Page 90: Reasoning Made Easy - pdfstores.files.wordpress.com · REASONING MADE EASY 6 Geetika, Jasmine, Hema, Brinda, Ipsita and veena, once again not following any order. The following are

REASONING MADE EASY www.BankExamsToday.com

90

d) R e) Data inadequate

87. Who is 2nd to the left of B?a) D b) H c) Md) Data inadequate e) None of these

88. In which of the following combinations 3rd person is 2nd to the left of 2nd

person?a) BAR b) DBM c) TPHd) PMH e) None of these

Directions (Q. 89-93) Study the following information carefully and answer thequestions given below

10 people are sitting in two parallel rows containing five people each, insuch a way that there is an eaual distance between adjacent people. In row-1, Q, R,S, T and U are seated and all of them are facing north. In Row-2, L, M, N, O and Pare seated and all of them are facing south. Therefore in the given sittingarrangement each member seated in a row faces another member of the other row.

S is sitting 2nd to the left of Q. Q is not sitting at any of the ends of the line.There are two persons between P and L. The person who faces T is to theimmediately left of M. M is sitting at the extreme right end. Q is an immediateneighbour of U. The person who faces U is an immediate neighbour of both P andN.

89. Who among the following is sitting 2nd to the right of O?a) P b) M c) Ld) Cannot be determined e) None of these

90. Who among the following sits exactly between Q and S?a) Tb) Uc) Rd) There is no person between Q and Se) Cannot be determined

91. Who among the following is sitting at the extreme right end of the Row-1?a) Q b) U c) Sd) R e) Cannot be detrmined

92. Which of the following statements is true regarding R?a) R is at the extreme left end of the rowb) R is an immediate neighbour of Uc) R is sitting second to the right of Td) R is sitting exactly between T and U

Page 91: Reasoning Made Easy - pdfstores.files.wordpress.com · REASONING MADE EASY 6 Geetika, Jasmine, Hema, Brinda, Ipsita and veena, once again not following any order. The following are

REASONING MADE EASY www.BankExamsToday.com

91

e) There two persons between R and S

93. Who among the following is not seated at any extreme end of Row-1 andRow-2?a) P b) S c) Md) R e) O

Directions (Q. 94-98) Study the following information carefully and answer thequestions given below

8 persons – E, F, G, H, I, J, K and L - are sitting around a circle atequidistance but not necessarily in the same order. Some of them are facing towardsthe centre while some others are facing outside the centre. L is sitting 3rd to the leftof K. Both K and L are facing towards the outside. H is not an immediate neighbourof K or L. J faces the just opposite direction of H. (It implies that if H is facingtowards the centre, J would face outside the centre). J is sitting 2nd to the left of H.Both the immediate neighbours of G face just opposite direction of G. E is animmediate neighbour of K. Both the immediate neighbour of H faces just oppositedirection of H. E faces towards the centre and he is an immediate n eighbour of bothK and I. I face towards the centre.

94. Who amongst the following are not facing towards the centre?a) E, F and I b) F, G and J c) H, K and Ld) G, I and J e) None of these

95. What is the position of G with respect to E?a) 2nd to the leftb) 3rd to the rightc) 4th to the leftd) 2nd to the righte) 5th to the right

96. Who among the following is sitting exactly between F and K?a) J b) I c) Ed) G e) None of these

97. How many persons are sitting between H and K if we move clockwisestarting from H?a) Three b) Four c) Fived) Two e) One

98. Which of the following statements is not true regarding the given sittingarrangement?a) L is sitting exactly between G and Ib) H is sitting 3rd to the right of Kc) F and I are sitting just opposite to each other

Page 92: Reasoning Made Easy - pdfstores.files.wordpress.com · REASONING MADE EASY 6 Geetika, Jasmine, Hema, Brinda, Ipsita and veena, once again not following any order. The following are

REASONING MADE EASY www.BankExamsToday.com

92

d) E is sitting 3rd to the left of Ge) All are true

Directions (Q. 99-103) Study the following information carefully and answerthe questions.

A, B, C, D, E, F, G and H are sitting around a circle, facing the centre. Asits fourth to the right of H while second to the left of F. C is not the neighbour of Fand B. D sits third to the right of C. H never sits next to G.

99. Who amongst the following sits between B and D?a) G b) F c) Hd) A e) C

100. Which of the following pairs sit between H and G?a) BH b) EF c) CEd) DB e) None of these

101. Four of the following are alike I n a certain way based on their positions inthe sitting arrangement and so form a group. Which is the one that does notbelong to that group?a) AE b) HF c) BDd) GE e) CH

102. Who is immediate right of A?a) C b) D c) Gd) Data inadequate e) None of these

103. Who sits second to the right of B?a) A b) C c) Dd) E e) None of these

Directions (Q. 104-108) Study the following carefully and answer the questions.

A, B, C, D, E, F, G and H are sitting around a circle, facing the centre. Eand G always sit next to each other. D sits third to the right of C. F sits to the left ofH. C never sits next to A while D never sits next to G. H is not the neighbour of Dand C.

104. Who sits to immediate right of F?a) D b) C c) Bd) A e) None of these

105. Four of the following are alike in a certain way based on their positions inthe sitting arrangement and so form a group. Which is the one that does notbelong to that group?

Page 93: Reasoning Made Easy - pdfstores.files.wordpress.com · REASONING MADE EASY 6 Geetika, Jasmine, Hema, Brinda, Ipsita and veena, once again not following any order. The following are

REASONING MADE EASY www.BankExamsToday.com

93

a) CH b) BA c) FEd) DG e) AC

106. Which of the following pairs sits between B and F?a) HB b) FD c) BGd) GC e) AH

107. Who sits second to the left of B?a) F b) G c) Ad) E e) None of these

108. Who sits between A and D?a) B b) F c) Cd) E e) None of these

Directions (Q. 109-113) Study the following information carefully to answerthese questions.

Ashwini, Priya, Sudha, Rani, Meeta, Geeta and Mukta are sitting around acircle facing the centre. Ashwini is third to the left of Mukta and to the immediateright o f Rani. Priya is second to the left of Geeta who is not an immediateneighbour of Meeta.

109. Who is to the immediate right of Priya?a) Meeta b) Sudha c) Muktad) Cannot be determined e) None of the above

110. Who is second to the left of Rani?a) Ashwini b) Meeta c) Priyad) Sudha e) None of these

111. Which of the following pairs of persons has the first person sitting to theimmediate left of second person?a) Rani-Meeta b) Ashwini-Geeta c) Sudha-Priyad) Geeta-Sudha e) None of these

112. Which of the following groups has the first person sitting between the othertwo?a) Meeta-Ashwini-Geetab) Sudha-Rani-Geetac) Mukta-Priya-Ranid) Mukta-Priya-Sudhae) None of these

113. Which of the following is the correct position of Rani with respect toMukta?

Page 94: Reasoning Made Easy - pdfstores.files.wordpress.com · REASONING MADE EASY 6 Geetika, Jasmine, Hema, Brinda, Ipsita and veena, once again not following any order. The following are

REASONING MADE EASY www.BankExamsToday.com

94

i) Third to the rightii) Third to the leftiii) Fourth to the leftiv) Fourth to the righta) i) only b) ii) only c) Both i) and ii)d) Both ii) and iv) e) Both i) and iii)

Directions (Q. 114-118) Study the following information carefully to answerthese questions.

A, B, C, D, E, F, G and H are sitting around a circle facing the centre. F isthird to the right of C and second to the left of H. D is not an immediate neighbourof C or H. E is to the immediate right of A, who is second to the right of G.

114. Who is second to the left of C?a) A b) B c) Ed) D e) None of these

115. Who is to the immediate right of C?a) A b) B c) Dd) B or D e) None of these

116. Which of the following pairs of persons has first person sitting to the rightof the second person?a) CB b) AE c) FGd) HA e) DB

117. Who sits between G and D?a) H b) D c) Fd) E e) None of these

118. Which of the following is the correct position of B with respect to H?I. Second to the rightII. Fourth to the ri ghtIII. Fourth to the leftIV. Second to the lefta) Only I b) Only II c) Only IIId) Both II and III e) None of these

Answers:1. Option D2. Option E3. Option A4. Option B5. Option B

Page 95: Reasoning Made Easy - pdfstores.files.wordpress.com · REASONING MADE EASY 6 Geetika, Jasmine, Hema, Brinda, Ipsita and veena, once again not following any order. The following are

REASONING MADE EASY www.BankExamsToday.com

95

6. Option A7. Option D8. Option D9. Option B10. Option C11. Option A12. Option B13. Option A14. Option D15. Option A16. Option B17. Option E18. Option A19. Option B20. Option C21. Option E22. Option E23. Option C24. Option B25. Option B26. Option A27. Option B28. Option D29. Option C30. Option B31. Option A32. Option A33. Option E34. Option D35. Option B36. Option E37. Option B38. Option D39. Option C40. Option B41. Option D42. Option C43. Option A44. Option C45. Option E46. Option A47. Option B48. Option A49. Option C50. Option B51. Option D

Page 96: Reasoning Made Easy - pdfstores.files.wordpress.com · REASONING MADE EASY 6 Geetika, Jasmine, Hema, Brinda, Ipsita and veena, once again not following any order. The following are

REASONING MADE EASY www.BankExamsToday.com

96

52. Option D53. Option B54. Option C55. Option B56. Option C57. Option C58. Option E59. Option B60. Option A61. Option D62. Option E63. Option D64. Option A65. Option C66. Option B67. Option E68. Option D69. Option C70. Option A71. Option B72. Option E73. Option B74. Option D75. Option E76. Option A77. Option C78. Option A79. Option D80. Option E81. Option B82. Option C83. Option A84. Option E85. Option D86. Option A87. Option C88. Option B89. Option C

L is sitting 2nd to the right of O.

90. Option A

T sits exactly between Q and S.

91. Option D

Page 97: Reasoning Made Easy - pdfstores.files.wordpress.com · REASONING MADE EASY 6 Geetika, Jasmine, Hema, Brinda, Ipsita and veena, once again not following any order. The following are

REASONING MADE EASY www.BankExamsToday.com

97

R is sitting at the extreme right end of the Row-1.

92. Option B

R is at the extreme right end.R is to the immediate right of U.R is sitting third to the right of T.R is at one of the ends.There are three persons – T, Q and U – between R and S

93. Option E

O is second from the left in Row-2.

94. Option C

H, K and L are facing outside.

95. Option D

E is facing t owards the centre. G is 2nd to the right or 5th to the left of E.

Page 98: Reasoning Made Easy - pdfstores.files.wordpress.com · REASONING MADE EASY 6 Geetika, Jasmine, Hema, Brinda, Ipsita and veena, once again not following any order. The following are

REASONING MADE EASY www.BankExamsToday.com

98

96. Option A

J is sitting exactly between F and K.

97. Option B

There are four persons – G, L, I and E – between H and K if we moveclockwise from H.

98. Option E

All the statements are true.

99. Option B100. Option C101. Option A102. Option B103. Option D104. Option D105. Option E106. Option E107. Option C108. Option B109. Option C110. Option E111. Option D112. Option B113. Option E114. Option A115. Option B116. Option E117. Option C118. Option D

Page 99: Reasoning Made Easy - pdfstores.files.wordpress.com · REASONING MADE EASY 6 Geetika, Jasmine, Hema, Brinda, Ipsita and veena, once again not following any order. The following are

REASONING MADE EASY www.BankExamsToday.com

99

Chapter - 5

Syllogisms

Directions (Q. 1-5) In each of the questions below are given three statementsfollowed by two conclusions numbered I and II. You have to take the givenstatements to be true even if they seem to be at variance withcommonly knownfacts. Read all the conclusions and then decide which of the given conclusionslogically follows from the given statements disregarding commonly knownfacts.

Give answer:a) If only conclusion I followsb) If only conclusion II followsc) If either conclusion I or II followsd) If neither conclusion I nor II followse) If both conclusions I and II follow

1. Statements: All benches are cots.No cot is lamp.Some lamps are candles.

Conclusions: I. Some cots are benches.II. Some candles are cots.

2. Statements: Some cats are dogs.All dogs are goats.All goats are walls.

Conclusions: I. Some walls are dogs.II. Some walls are cats.

3. Statements: Some buildings are sofas.Some sofas are benches.Some benches are tables.

Conclusions: I. Some tables are sofas.II. No table is sofa.

4. Statements: All rats are bats.Some bats are desks.All desks are chairs.

Conclusions: I. Some desks are rats.

Page 100: Reasoning Made Easy - pdfstores.files.wordpress.com · REASONING MADE EASY 6 Geetika, Jasmine, Hema, Brinda, Ipsita and veena, once again not following any order. The following are

REASONING MADE EASY www.BankExamsToday.com

100

II. Some chairs are rats.

5. Statements: Some roads are ponds.All ponds are stores.Some stores are bags.

Conclusions: I. Some bags are ponds.II. Some stores are roads.

Directions (Q. 6-11) In each question below are four statements followed byfour conc lusions numbered I, II, III and IV. You have to take the four givenstatements to be true even if they seem to be at variance from commonlyknown facts and than decide which of the given conclusions logically followsfrom the four given statements disregarding commonly known facts. Thendecide which of the answers a, b, c, d and e is correct answer.

6. Statements: All belts are rollers.Some rollers are wheels.All wheels are mats.Some mats are cars.

Conclusions: I. Some mats are rollers.II. Some mats are belts.III. Some cars are rollers.IV. Some rollers are belts.

a) Only I and II followb) Only I, III and IV followc) Only I and IV followd) Only II, III and IV followe) None of the above

7. Statements: Some tyres are rains.Some rains are flowers.All flowers are jungles.All jungles are tubes.

Conclusions: I. Some jungles are tyres.II. Some tubes are rains.III. Some jungles are rains.IV. Some tubes are flowers.

a) Only I, II and III followb) Only II, III and IV followc) Only I, III and IV followd) All followe) None of the above

Page 101: Reasoning Made Easy - pdfstores.files.wordpress.com · REASONING MADE EASY 6 Geetika, Jasmine, Hema, Brinda, Ipsita and veena, once again not following any order. The following are

REASONING MADE EASY www.BankExamsToday.com

101

8. Statements: All desks are chairs.All chairs are tables.All tables are boxes.All boxes are trunks.

Conclusions: I. Some trunks are tables.II. All chairs are boxes.III. Some boxes are desks.IV. All desks are trunks.

a) Only I, II and III followb) Only I, II and IV followc) Only II, III and IV followd) All followe) None of the above

9. Statements: Some birds are goats.Some goats are horses.Some horses are lions.Some lions are tigers.

Conclusions: I. Some tigers are goats.II. No tiger is goat.III. Some lions are birds.IV. No lion is bird.

a) Only either I or II followsb) Only either III or IV followsc) Only either I or II and either III or IV followd) Only I and III followe) None of the above

10. Statements: All papers are bottles.All bottles are cups.Some cups are jugs.Some jugs are plates.

Conclusions: I. Some plates are cups.II. Some plates are bottles.III. Some cups are papers.IV. Some bottles are papers.

a) Only III and IV followb) Only I and II followc) Only I and II followd) Only II and IV followe) None of the above

11. Statements: All bulbs are wires.

Page 102: Reasoning Made Easy - pdfstores.files.wordpress.com · REASONING MADE EASY 6 Geetika, Jasmine, Hema, Brinda, Ipsita and veena, once again not following any order. The following are

REASONING MADE EASY www.BankExamsToday.com

102

No wire is cable.Some cables are brushes.All brushes are paints.

Conclusions: I. Some paints are cables.II. Some wires are bulbs.III. Some brushes are wires.IV. Some cables are bulbs.

a) None followsb) Only I and II followc) Only II followsd) Only III followse) Only IV follows

Directions (Q. 12-16) In each of the questions below are given four statementsfollowed by three conclusions numbered I, II and III. You have to take thegiven statements to be true even if they seem to be at variance from commonlyknown facts. Read all the conclusions and then decide which of the givenconclusions logically follows from the given statements disregarding commonlyknown facts.

12. Statements: All booklets are packets.All packets are bottles.Some bottles are cans.Some cans are pitchers.

Conclusions: I. Some pitchers are bottles.II. Some cans are packets.III. Some bottles are booklets.

a) None followsb) Only I followc) Only II followsd) Only III followse) Only II and III follow

13. Statements: Some ropes are walls.Some walls are sticks.All sticks are chairs.All chairs are tables.

Conclusions: I. Some tables are walls.II. Some chairs are ropes.III. Some sticks are ropes.

a) None followsb) Only I followsc) Only II follows

Page 103: Reasoning Made Easy - pdfstores.files.wordpress.com · REASONING MADE EASY 6 Geetika, Jasmine, Hema, Brinda, Ipsita and veena, once again not following any order. The following are

REASONING MADE EASY www.BankExamsToday.com

103

d) Only III followse) Only II and III follow

14. Statements: Some rivers are jungles.Some jungles are horses.Some horses are tents.Some tents are buildings.

Conclusions: I. Some buildings are horses.II. Some tents are jungles.III. Some horses are rivers.

a) None followsb) Only I followsc) Only II followsd) Only III followse) Only I and II follow

15. Statements: Some pens are knives.All knives are pins.Some pins are needles.All needles are chains.

Conclusions: I. Some chains are pins.II. Some needles are knives.III. Some pins are pens.

a) Only I followsb) Only II followsc) Only III followsd) Only II and III followe) None of these

16. Statements: All fields are ponds.No pond is tree.Some trees are huts.All huts are goats.

Conclusions: I. Some goats are fields.II. No goat is field.III. Some goats are trees.

a) Only I followsb) Only II followsc) Only III followsd) Only eithr I or II followse) Only either I or II and III follow

Page 104: Reasoning Made Easy - pdfstores.files.wordpress.com · REASONING MADE EASY 6 Geetika, Jasmine, Hema, Brinda, Ipsita and veena, once again not following any order. The following are

REASONING MADE EASY www.BankExamsToday.com

104

Directions (Q. 17-28) In each of the questions below are given three statementsfollowed by three conclusions numbered I, II and III. You have to take thegiven statements to be true even if they seem to be at vari ance from commonlyknown facts. Read all the conclusions and then decide which of the givenconclusions logically follow from the given statements disregarding commonlyknown facts.

17. Statements: All books are tents.Some tents are lakes.All lakes are ponds.

Conclusions: I. Some ponds are books.II. Some ponds are tents.III. Some lakes are books.

a) None followsb) Only I followsc) Only II followsd) Only III followse) Only II and III follow

18. Statements: All pictures are walls.Some walls are rooms.Some rooms are windows.

Conclusions: I. Some windows are walls.II. Some windows are pictures.III. Some rooms are walls.

a) None followsb) Only I followsc) Only II followsd) Only III followse) Only II and III follow

19. Statements: All baskets are marbles.Some marbles are sticks.No stick is garden.

Conclusions: I. Some gardens are baskets.II. Some sticks are baskets.III. No garden is baskets.

a) Only I followsb) Only III followsc) Only either I or III followsd) Only II followse) None of the above

20. Statements: Some bulbs are tubes.

Page 105: Reasoning Made Easy - pdfstores.files.wordpress.com · REASONING MADE EASY 6 Geetika, Jasmine, Hema, Brinda, Ipsita and veena, once again not following any order. The following are

REASONING MADE EASY www.BankExamsToday.com

105

Some tubes are wires.Some wires are lamps.

Conclusions: I. Some lamps are tubes.II. Some wires are bulbs.III. Some lamps are bulbs.

a) None followsb) Only III followsc) Only II followsd) Only I followse) Only I and II follow

21. Statements: All buildings are rivers.All rivers are jungles.All jungles are mountains.

Conclusions: I. Some mountains are rivers.II. Some jungles are buildings.III. Some mountains are buildings.

a) Only I and II followb) Only I and III followc) Only II and III followd) All I, II and III followe) None of the above

22. Statements: All chairs are keys.All keys are baloons.Some balloons are mirrors.Some mirrors are desks.

Conclusions: I. Some desks are keys.II. Some balloons are chairs.III. Some mirrors are balloons.

a) Only I followsb) Only II followsc) Only III followsd) Only II and III followe) All I, II and III follow

23. Statements: Some drums are posters.All posters are windows.Some windows are tablets.All tablets are books.

Conclusions: I. Some windows are drums.II. Some books are posters.

Page 106: Reasoning Made Easy - pdfstores.files.wordpress.com · REASONING MADE EASY 6 Geetika, Jasmine, Hema, Brinda, Ipsita and veena, once again not following any order. The following are

REASONING MADE EASY www.BankExamsToday.com

106

III. Some tablets are drums.a) None followsb) Only I followc) Only II followsd) Only III followse) Only I and II follow

24. Statements: Some boxes are toys.Some toys are nails.Some nails are stores.Some stores are shops.

Conclusions: I. Some nails are boxes.II. Some toys are shops.III. No nail is box.

a) Only I followsb) Only III followsc) Only either I or III followsd) Only II followse) None of the above

25. Statements: All doors are windows.No window is house.Some houses are buildings.All buildings are skies.

Conclusions: I. Some skies are doors.II. Some skies are houses.III. Some buildings are doors.

a) Only I followsb) Only II followsc) Only III followsd) Only II and III followe) None of these

26. Statements: All rivers are walls.All walls are stones.All stones are clothes.All clothes are trees.

Conclusions: I. Some trees are stones.II. Some clothes are rivers.III. All walls are clothes.

a) Only I and II followb) Only I and III followc) Only II and III follow

Page 107: Reasoning Made Easy - pdfstores.files.wordpress.com · REASONING MADE EASY 6 Geetika, Jasmine, Hema, Brinda, Ipsita and veena, once again not following any order. The following are

REASONING MADE EASY www.BankExamsToday.com

107

d) All I, II and III followe) None of the above

27. Statements: Some letters are glasses.Some glasses are plates.All plates are buses.All buses are cars.

Conclusions: I. Some cars are letters.II. Some cars are glasses.III. Some buses are glasses.

a) Only I and II followb) Only I and III followc) Only II followsd) Only III followse) Only II and III follow

28. Statements: All books are pens.Some pens are ropes.All ropes are discs.Some discs are bricks.

Conclusions: I. Some bricks are ropes.II. Some discs are books.III. Some bricks are pens.

a) None followsb) Only I followsc) Only II followsd) Only III followse) Only II and III follow

Directions (Q. 29-34) In each of the questions below are given four statementsfollowed by four conclusions number I, II, III and IV. You have to take thegiven statements to be true even if they seem to be of variance from commonlyknown facts. Read all the conclusions and then decide which of the givenconclusions logically follows from the given statements disregarding commonlyknown facts.

29. Statements: Some robots are machines.Some computers are both robots and machines.Some animals are machines.Some toys are animals.

Conclusions: I. Some toys are robots.II. Some toys are machines.

Page 108: Reasoning Made Easy - pdfstores.files.wordpress.com · REASONING MADE EASY 6 Geetika, Jasmine, Hema, Brinda, Ipsita and veena, once again not following any order. The following are

REASONING MADE EASY www.BankExamsToday.com

108

III. Some animals are computers.IV. Some robots are not toys.

a) None followsb) Only II and III followc) Only I and III followd) Only III followse) Only either I or IV follows

30. Statements: All suns are stars.All moons are stars.Some planets are suns.Some stars are gases.

Conclusions: I. Some starts are planets.II. Some suns are gases.III. No moon is a planet.IV. Some gases are moons.

a) None followsb) Only I followsc) Only I and II followsd) Only III and IV followe) Only I and III follow

31. Statements: All books are diaries.Some diaries are pens.Some pens are drawers.All drawers are chairs.

Conclusions: I. Some drawers are diaries.II. Some chairs are pens.III. Some pens are books.IV. Some diaries are books.

a) None followsb) Only II followsc) Only II and III followd) Only II and IV followe) All follow

32. Statements: Some buildings are rivers.Some mountains are both buildings and rivers.Some roads are buildings.All roads are trucks.

Conclusions: I. Some mountains are roads.II.Some buildings are trucks.III.Some rivers are roads.

Page 109: Reasoning Made Easy - pdfstores.files.wordpress.com · REASONING MADE EASY 6 Geetika, Jasmine, Hema, Brinda, Ipsita and veena, once again not following any order. The following are

REASONING MADE EASY www.BankExamsToday.com

109

IV. Some trucks are rivers.

a) None followsb) Only I followsc) Only II followsd) Only III followse) All follow

33. Statements: All tables are round.Some hills are round.Some rivers are hills.All rivers are conical.

Conclusions: I. Some rivers are round.II. Some hills are conical.III. Some rivers are both hills and round.IV. Some tables are conical.

a) None followsb) Only II followsc) Only I and III followd) Only II and III followe) All follow

34. Statements: All sharks are fishes.Some fishes are birds.All birds are trees.All trees are insects.

Conclusions: I. Some insects are sharks.II. Some sharks are trees.III. All insects are birds.IV. Some birds are sharks.

a) None followsb) Only II followsc) Only I and IV followd) Only II and III followe) All follow

Answers:1. Option A

Page 110: Reasoning Made Easy - pdfstores.files.wordpress.com · REASONING MADE EASY 6 Geetika, Jasmine, Hema, Brinda, Ipsita and veena, once again not following any order. The following are

REASONING MADE EASY www.BankExamsToday.com

110

So, only I follows.

2. Option E

So, both I and II follow.

3. Option C

Page 111: Reasoning Made Easy - pdfstores.files.wordpress.com · REASONING MADE EASY 6 Geetika, Jasmine, Hema, Brinda, Ipsita and veena, once again not following any order. The following are

REASONING MADE EASY www.BankExamsToday.com

111

So, either I or II follows

4. Option D

So, neither I nor II follows.

5. Option B

Page 112: Reasoning Made Easy - pdfstores.files.wordpress.com · REASONING MADE EASY 6 Geetika, Jasmine, Hema, Brinda, Ipsita and veena, once again not following any order. The following are

REASONING MADE EASY www.BankExamsToday.com

112

So, only II follows.

6. Option C

Only I and IV follow

7. Option B

Page 113: Reasoning Made Easy - pdfstores.files.wordpress.com · REASONING MADE EASY 6 Geetika, Jasmine, Hema, Brinda, Ipsita and veena, once again not following any order. The following are

REASONING MADE EASY www.BankExamsToday.com

113

So, only II, III and IV follow

8. Option D

All follows

9. Option C

Page 114: Reasoning Made Easy - pdfstores.files.wordpress.com · REASONING MADE EASY 6 Geetika, Jasmine, Hema, Brinda, Ipsita and veena, once again not following any order. The following are

REASONING MADE EASY www.BankExamsToday.com

114

So, either III or IV follow

10. Option A

So, only III and IV follow

11. Option B

Page 115: Reasoning Made Easy - pdfstores.files.wordpress.com · REASONING MADE EASY 6 Geetika, Jasmine, Hema, Brinda, Ipsita and veena, once again not following any order. The following are

REASONING MADE EASY www.BankExamsToday.com

115

Only I and II follow.

12. Option D

Only III follows.

13. Option B

Page 116: Reasoning Made Easy - pdfstores.files.wordpress.com · REASONING MADE EASY 6 Geetika, Jasmine, Hema, Brinda, Ipsita and veena, once again not following any order. The following are

REASONING MADE EASY www.BankExamsToday.com

116

Only I follows.

14. Option A

None follows

15. Option E

Page 117: Reasoning Made Easy - pdfstores.files.wordpress.com · REASONING MADE EASY 6 Geetika, Jasmine, Hema, Brinda, Ipsita and veena, once again not following any order. The following are

REASONING MADE EASY www.BankExamsToday.com

117

Only I and III follow.

16. Option E

Either I or II and III follow

17. Option C

Page 118: Reasoning Made Easy - pdfstores.files.wordpress.com · REASONING MADE EASY 6 Geetika, Jasmine, Hema, Brinda, Ipsita and veena, once again not following any order. The following are

REASONING MADE EASY www.BankExamsToday.com

118

So, only II follows

18. Option D

Only III follows.

19. Option C

Page 119: Reasoning Made Easy - pdfstores.files.wordpress.com · REASONING MADE EASY 6 Geetika, Jasmine, Hema, Brinda, Ipsita and veena, once again not following any order. The following are

REASONING MADE EASY www.BankExamsToday.com

119

So, either I or III follows

20. Option A

So, None follows

21. Option D

Page 120: Reasoning Made Easy - pdfstores.files.wordpress.com · REASONING MADE EASY 6 Geetika, Jasmine, Hema, Brinda, Ipsita and veena, once again not following any order. The following are

REASONING MADE EASY www.BankExamsToday.com

120

So, all I, II and III follow

22. Option D

Only II and III follow.

Page 121: Reasoning Made Easy - pdfstores.files.wordpress.com · REASONING MADE EASY 6 Geetika, Jasmine, Hema, Brinda, Ipsita and veena, once again not following any order. The following are

REASONING MADE EASY www.BankExamsToday.com

121

23. Option B

Only I follows.

24. Option C

Either I or III follows

25. Option B

Page 122: Reasoning Made Easy - pdfstores.files.wordpress.com · REASONING MADE EASY 6 Geetika, Jasmine, Hema, Brinda, Ipsita and veena, once again not following any order. The following are

REASONING MADE EASY www.BankExamsToday.com

122

Only II follows

26. Option D

So, All I, II and III follow

27. Option E

Page 123: Reasoning Made Easy - pdfstores.files.wordpress.com · REASONING MADE EASY 6 Geetika, Jasmine, Hema, Brinda, Ipsita and veena, once again not following any order. The following are

REASONING MADE EASY www.BankExamsToday.com

123

Only II and III follow

28. Option A

None follows

29. Option E30. Option B31. Option D32. Option C33. Option B34. Option A

Page 124: Reasoning Made Easy - pdfstores.files.wordpress.com · REASONING MADE EASY 6 Geetika, Jasmine, Hema, Brinda, Ipsita and veena, once again not following any order. The following are

REASONING MADE EASY www.BankExamsToday.com

124

Chapter - 6

Blood Relations

1. R is sister of M who is brother of H. D is mother of K who is brother of M.How is R related to D?a) Sister b) Daughter c) Motherd) Data inadequate e) None of these

2. K is brother of T. M is mother of K. W is brother of M. How is W relatedto T?a) Maternal uncleb) Paternal unclec) Grandfatherd) Data inadequatee) None of these

3. Pointing to a woman, Nirmal said, “She is the daughter of my wife’sgrandfather’s only child”. How is the women releated to Nirmal?a) Wife b) Sister-in-law c) Sisterd) Data inadequate e) None of these

4. D is brother of K. M is sister of K. T is father of R who is brother of M. Fis mother of K. At least how many sons does T and F have?a) Two b) Three c) Fourd) Data inadequate e) None of these

Directions (Q. 5-6) Study the following information carefully and answer thequestions given below

S has two daughters R and A. G is married to R. G is father of N. L is sonof A.

5. How L is related to S?a) Nephew b) Brother c) Grandsond) Great Grandson e) None of these

6. Which of the following statement is/are true on the basis of informationgiven above?a) G is son-in-law of Sb) R is wife of Gc) A is mother of Ld) L is cousin of Ne) All are true

Page 125: Reasoning Made Easy - pdfstores.files.wordpress.com · REASONING MADE EASY 6 Geetika, Jasmine, Hema, Brinda, Ipsita and veena, once again not following any order. The following are

REASONING MADE EASY www.BankExamsToday.com

125

7. M is sister of K. D is brother of K. F is mother of M. How is K related toF?a) Son b) Daughter c) Son or Daughterd) Data inadequate e) None of the above

Directions (Q. 8-10) Read the following information carefully and answer thequestions which follow:

‘A × B’ means ‘A is wife of B’‘A + B’ means ‘A is brother of B’‘A ÷ B’ means ‘A is daughter of B’‘A ⎯B’ means ‘A is son of B’

8. How is Q related to P if ‘P ÷ R × T ⎯Q’?a) Granddaughterb) Mother-in-law/Father-in-lawc) Grandmotherd) Grandmother/Grandfathere) None of the above

9. How is R related to Q, if ‘P ⎯Q + R ÷ T’?a) Brother/Sister b) Niece c) Sisterd) Nephew/Niece e) None of these

10. How is T related to P if ‘P × Q ⎯T + R’?a) Motherb) Father-in-lawc) Mother-in-lawd) Mother-in-law/Father-in-lawe) None of the above

Directions (Q. 11-15) Read the information carefully and answer the followingquestions.

If A + B means A is the father of B.If A × B means A is the sister of B.If A $ B means A is the wife of B.If A % B means A is the mother of B.If A ÷ B means A is t he son of B.

11. What should come in place of the question mark, to establish that J is thebrother of T in the expression?J ÷ P % H ? T % La) x b) ÷ c) $d) Either ÷ or x e) Either + or ÷

12. Which among the given expressions indicate that M is the daughter of D?

Page 126: Reasoning Made Easy - pdfstores.files.wordpress.com · REASONING MADE EASY 6 Geetika, Jasmine, Hema, Brinda, Ipsita and veena, once again not following any order. The following are

REASONING MADE EASY www.BankExamsToday.com

126

a) L % R $ D + T × Mb) L + R $ D + M × Tc) L % R % D + T ÷ Md) D + L $ R + M × Te) L $ D ÷ R % M ÷ T

13. Which among the f ollowing options is true, if thue expression ‘I + T % J × L ÷K’ is definitely true?a) L is the daughter of Tb) K is the son-in-law of Ic) I is the grandmother of Ld) T is the father of Je) J is the brother of L

14. Which among the following expressions is true, if Y is the son of X is definitelyfalse?a) W % L × T × Y ÷ Xb) W + L × T × Y ÷ Xc) X + L × T × Y ÷ Wd) W $ X + L + Y + Te) W % X + T × Y ÷ L

15. What should come in place of the question mark, to establish that T is the sister-in-law of Q I n the expression?R % T × P ? Q + Va) ÷ b) % c) ×d) $ e) None of these

Directions (Q. 16-20) Study the following information carefully and answer thegiven questions:

There are eight family members- I, J, K, L, M, N, P and Q. J has only twochildren K and L. I is mother of K, who is sister of L. N is son of L. Q is brother ofN. M is mother of Q. P is granddaughter of I.

16. How is Q related to L?a) Mother b) Son c) Daughterd) Sister e) None of these

17. Who among the following is father of P?a) J b) L c) Nd) Cannot be determined e) None of these

18. How is M related to I?a) Son b) Son-in-law c) Daughterd) Cannot be determined e) None of these

Page 127: Reasoning Made Easy - pdfstores.files.wordpress.com · REASONING MADE EASY 6 Geetika, Jasmine, Hema, Brinda, Ipsita and veena, once again not following any order. The following are

REASONING MADE EASY www.BankExamsToday.com

127

19. How many children does M have?a) One b) Two c) Threed) Either b or c e) None of these

20. How is K related to M?a) Sister b) Daughter c) Sister-in-lawd) Mother-in-law e) None of these

Directions (Q. 21-23) Read the following information carefully and answer thequestions which follow.

If ‘A × B’ means ‘A is father of B’.If ‘A + B’ means ‘A is wife of B’.If ‘A ÷ B’ means ‘A is daughter of B’.If ‘A ⎯B’ means ‘A is son of B.

21. How is L related to Q in the expression ‘L ÷ M × O ⎯P ÷ Q’?a) Granddaughter b) Niece c) Daughter-in-lawd) Daughter e) cannot be determined

22. What will come in the place of the question mark, to establish that Q is thenephew of T in the expression ‘Q ? R ÷ S × T’?

a) + b) × c)⎯d) ÷ e) Either ⎯or ÷

23. Which of the following relations are true based upon the relations given inthe equation:‘A ⎯B × C + D ⎯E’?

a) C is mother of Ab) E is wife of Bc) D is brother of Ad) E is mother-in-law of Ce) None is true

Directions (Q. 24-26) Study the following information carefully and answer thequestions below.

(i) ‘P × Q’ means ‘P is brother of Q.(ii) ‘P ⎯Q’ means ‘P is mother of Q’.(iii) ‘P + Q’ means ‘P is father of Q’.(iv) ‘P ÷ Q’ means ‘P is sister of Q’.

24. Which of the following means ‘M is niece of N ‘?a) M × R ⎯N b) N ÷ J + M ÷ D c) N ÷ J + Md) N × J ⎯M e) None of these

Page 128: Reasoning Made Easy - pdfstores.files.wordpress.com · REASONING MADE EASY 6 Geetika, Jasmine, Hema, Brinda, Ipsita and veena, once again not following any order. The following are

REASONING MADE EASY www.BankExamsToday.com

128

25. Which of the following means ‘B is grandfather of F’?a) B + J ⎯F b) B ⎯J + F c) B × T ⎯Fd) B ÷ T + F e) None of these

26. How is M related to K in the expression ‘B + K ÷ T × M’?a) Son b) Daughter c) Son or daughterd) Data inadequate e) None of these

Directions (Q. 27-32) Read the following information carefully and answer thequestions which follow:

(i) ‘A × B’ means ‘A is father of B’(ii) ‘A + B’ means ‘A is daughter of B’(iii) ‘A ÷ B’ means ‘A is mother of B’(iv) ‘A ⎯B’ means ‘A is brother of B’

27. If ‘P ÷ R ⎯Q × T’ how is P related to T?a) Grandmother b) Mother-in-law c) Sisterd) Grandfather e) None of these

28. If ‘P ÷ Q + R × T’ how is T related to Q’?a) Aunt b) Sister c) Brotherd) Grandson e) None of these

29. Which of the following means that R is wife of P?a) P × R ⎯Q ⎯Tb) P ÷ T + R ⎯Qc) P ÷ R ⎯Q + Td) P × T ⎯Q + Re) None of these

30. If ‘R ⎯P ÷ J × Q’ how is J related to R?a) Son/daughter b) Nephew c) Nieced) Grandson e) None of these

31. If ‘P + Q ⎯R ÷ T’, how is T related to P?a) Aunt b) Aunt/Uncle c) Fatherd) Grandmother e) None of these

32. If ‘P × T ÷ Q + R’ how is R related to P?a) Daughter b) Husband c) Son-in-lawd) Son-in-law/Daughter-in-law e) None of the above

Answers:

Page 129: Reasoning Made Easy - pdfstores.files.wordpress.com · REASONING MADE EASY 6 Geetika, Jasmine, Hema, Brinda, Ipsita and veena, once again not following any order. The following are

REASONING MADE EASY www.BankExamsToday.com

129

1. Option B2. Option A3. Option A

Woman = daughter of Nirmal’s wife’s grandfather’s only child = daughterof Nirmal’s wife’s father = Nirmal’s wife

4. Option A5. Option C

A is daughter of S. L is son of A. So, L is grandson of S.

6. Option E

All the statements are true. R is wife of G. R is daughter of S. So, G is son-in-law of S. L is son of A and A is daughter of S. So, A is mother of L. Rand A are sisters. N is child of R and L is son of A. So, L is cousin of N.

7. Option C8. Option D9. Option E10. Option B11. Option A12. Option B13. Option B14. Option D15. Option D16. Option B17. Option B18. Option E19. Option C20. Option C21. Option A22. Option B23. Option E24. Option B25. Option A26. Option D27. Option A28. Option E29. Option D30. Option B31. Option E32. Option C

Page 130: Reasoning Made Easy - pdfstores.files.wordpress.com · REASONING MADE EASY 6 Geetika, Jasmine, Hema, Brinda, Ipsita and veena, once again not following any order. The following are

REASONING MADE EASY www.BankExamsToday.com

130

Chapter – 7

Analogy

Directions (Q.1-9) In each of the following questions, select the relatedletter/word/number from the given alternatives.

1. Lion : Forest : : Fish : ?a) Cage b) Nest c) Skyd) Water e) None of these

2. Polio : Virus : : Anthrax : ?a) Fungus b) Bacteria c) Virusd) Insect e) None of these

3. Money : Yenom : : Right : ?a) HTIRG b) THGIR c) GIRHTd) IRGHT e) None of these

4. NIIOLUSL : ILLUSION : : TBERVARE : ?a) STRANGE b) VERTEBRA c) VABTEREd) ERAVEBT e) None of these

5. RUST : 9687 : : TSUR : ?a) 7896 b) 7869 c) 7689d) 6789 e) None of these

6. 13 : 169 : : ?a) 12 : 140 b) 3 : 6 c) 11 : 111d) 9 : 81 e) None of these

7. 23 : 29 : : 41 : ?a) 43 b) 45 c) 47d) 49 e) None of these

8. 6 : 10 : : 9 : ?a) 10 b) 12 c) 15d) 18 e) None of these

9. River : Tributary : : Tree : ?a) Stem b) Root c) Branchd) Flower e) None of these

Page 131: Reasoning Made Easy - pdfstores.files.wordpress.com · REASONING MADE EASY 6 Geetika, Jasmine, Hema, Brinda, Ipsita and veena, once again not following any order. The following are

REASONING MADE EASY www.BankExamsToday.com

131

Answers:

1. Option DForest is the habitat of Lion. Similarly, water body is the habitat of fish.

2. Option BThe causative organism of polio is virus. Similarly, the causative organismof anthrax is bacteria.

3. Option B

4. Option BFrom the jumbled letters NIIOLUSL, we can form the meaningful wordILLUSION. Similarly, we can form VERTEBRA from the jumbled lettersTBERVARE.

5. Option BR U S T9 6 8 7Similarly,T S U R7 8 6 9

6. Option DThe relation is x : x square13 × 13 = 169Similarly,9 × 9 = 81

7. Option ATwo consecutive prime numbers are given:23 ⟶ 29Similarly,41 ⟶ 43

8. Option C2 × 3 = 62 × 5 = 10Similarly,3 × 3 = 93 × 5 = 15

9. Option CTributary is a part of river. Similarly, branch is a part of tree and is analogusto tributary in the case of river.

Page 132: Reasoning Made Easy - pdfstores.files.wordpress.com · REASONING MADE EASY 6 Geetika, Jasmine, Hema, Brinda, Ipsita and veena, once again not following any order. The following are

REASONING MADE EASY www.BankExamsToday.com

132

Chapter - 8

Coded Inequality

Directions (Q. 1-5) In the following questions, the symbols @, ©, $, % and *are used with the following meanings as illustrated below.‘P © Q’ means ‘P is not greater than Q’.‘P % Q’ means ‘P is not smaller than Q’.‘P * Q’ means ‘P is neither smaller than nor equal to Q’.‘P @ Q’ means ‘P is neither greater than nor equal to Q’.‘P $ Q’ means ‘P is neither greater than nor smaller than Q’.

Now in each of the following questions, assuming the given statements to be true,find which of the two conclusions I and II given below them is/are definitely true.

Give answer:a) If only conclusion I is true.b) If only conclusion II is true.c) If either conclusion I or II is true.d) If neither conclusion I nor II is true.e) If both conclusions I and II are true.

1. Statements: K @ V, V © N, N % FConclusions: I. F @ V

II. K @ N

2. Statements: H © W, W $ M, M @ BConclusions: I. B * H

II. M % H

3. Statements: D % B, B * T, T $ MConclusions: I. T © D

II. M © D

4. Statements: M * T, T @ K, K © NConclusions: I. N * T

II. N * M

5. Statements: R $ J, J % D, D * FConclusions: I. D $ R

II. D @ R

Directions (Q. 6-11) In the following questions, the symbols ©, @, $, % and *are used with following meanings as illustrated below.‘P % Q’ means ‘P is not greater than Q’

Page 133: Reasoning Made Easy - pdfstores.files.wordpress.com · REASONING MADE EASY 6 Geetika, Jasmine, Hema, Brinda, Ipsita and veena, once again not following any order. The following are

REASONING MADE EASY www.BankExamsToday.com

133

‘P % Q’ means ‘P is not smaller than Q’‘P * Q’ means ‘P is neither smaller than nor equal to Q’‘P © Q’ means ‘P is neither greater than nor equal to Q’‘P $ Q’ means ‘P is neither greater than nor smaller than Q’

In each question four statements showing relationship have been given, which arefollowed by four conclusions I, II, III and IV. Assuming that the given statementsare true, find out which conclusion (s) is/are definitely true.

6. Statements: M © D , D * K, K @ R, R * FConclusions: I. F © K

II. D * FIII. M © RIV. D * R

a) None is trueb) Only I is truec) Only II is trued) Only III is truee) Only IV is true

7. Statements: B % K, K $ T, T * F, H © FConclusions: I. B $ T

II. T © BIII. H © KIV. F © B

a) Only either I or II is trueb) Only III is truec) Only IV is trued) Only III and IV are truee) Only either I or II and III and IV are true

8. Statements: W * B, B @ F, F © R, R $ MConclusions: I. W * F

II. M * BIII. R * BIV. M * W

a) Only I and IV are trueb) Only II and III are truec) Only I and III are trued) Only II and IV are truee) None of the above

9. Statements: E @ K, K $ T, T © N, B % NConclusions: I. T % E

II. K © NIII. B * T

Page 134: Reasoning Made Easy - pdfstores.files.wordpress.com · REASONING MADE EASY 6 Geetika, Jasmine, Hema, Brinda, Ipsita and veena, once again not following any order. The following are

REASONING MADE EASY www.BankExamsToday.com

134

IV. B * Ea) Only I, II and III are trueb) Only II, III and IV are truec) Only I, III and IV are trued) All are truee) None of the above

10. Statements: Z $ B, B % M, M © F, F @ RConclusions: I. Z * M

II. F * BIII. R * MIV. M @ Z

a) Only I and II are trueb) Only I, III and IV are truec) Only III and IV are trued) Only either I or IV and III are truee) None of the above

11. Statements: H @ T, T $ N, F © N, B % FConclusions: I. F @ H

II. F © TIII. B * TIV. B % H

a) None is trueb) Only I is truec) Only II is trued) Only II and IV are truee) Only II and III are true

Directions (Q. 12-16) In the following questions, the symbols ^, #, %, @ and *are used with the following meaning as illustrated below:

‘P # Q’ means ‘P is neither greater nor smaller than Q’.‘P ^ Q’ means ‘P is not smaller than Q’.‘P @ Q’ means ‘P is neither smaller than nor equal to Q’.‘P * Q’ means ‘P is not greater than Q’.‘P % Q’ means ‘P is neither greater than nor equal to Q’.

Now in each of the following questions assuming the given statements to be true,find which of the three conclusions I, II and III given below them is/are definitelytrue and give your answer accordingly.

12. Statements: D % F, F @ H, H * NConclusions: I. N @ F

II. D % NIII. H % D

a) None is true

Page 135: Reasoning Made Easy - pdfstores.files.wordpress.com · REASONING MADE EASY 6 Geetika, Jasmine, Hema, Brinda, Ipsita and veena, once again not following any order. The following are

REASONING MADE EASY www.BankExamsToday.com

135

b) Only I is truec) Only II is trued) Only III is truee) Only I and II are true

13. Statements: B ^ D, D % T, T * MConclusions: I. B @ T

II. M @ DIII. B @ M

a) Only I is trueb) Only II is truec) Only III is trued) Only II and III are truee) None of these

14. Statements: K # W, M @ W, R ^ MConclusions: I. K % M

II. W % RIII. R @ K

a) Only I and II are trueb) Only I and III are truec) Only II and III are trued) All I, II and III are truee) None of the above

15. Statements: M @ K, K ^ T, T # JConclusions: I. J # K

II. M @ JIII. J % K

a) Only I is trueb) Only II is truec) Only III is trued) Only either I or III is truee) Only either I or III and II are true

16. Statements: R * N, N % B, B # TConclusions: I. B @ R

II. T @ NIII. R % T

a) Only I and II are trueb) Only I and III are truec) Only II and III are trued) All I, II and III are truee) None of the above

Page 136: Reasoning Made Easy - pdfstores.files.wordpress.com · REASONING MADE EASY 6 Geetika, Jasmine, Hema, Brinda, Ipsita and veena, once again not following any order. The following are

REASONING MADE EASY www.BankExamsToday.com

136

Directions (Q. 17-21) In the following questions, the symbols @, ©, $, % and *are uses with the following meaning as illustrated below.

‘P © Q’ means ‘P is not smaller than Q’‘P % Q’ means ‘P is not greater than Q’‘P * Q’ means ‘P is neither smaller than nor equal to Q’.‘P @ Q’ means ‘P is neither greater than nor smaller than Q’‘P $ Q’ means ‘P is neither greater than nor equal to Q’

Now in each of the following questions assuming the given statements to be true,find which of the conclusions I, II and III given below them is/are definitely true:

17. Statements: F % T, T @ J, J * WConclusions: I. J @ F

II. J * FIII. W $ T

a) Only I is trueb) Only II is truec) Only III is trued) Only either I or II is truee) Only either I or II and III are true

18. Statements: R * D, D © K, K $ MConclusions: I. M * R

II. K $ RIII. D * M

a) None is trueb) Only I is truec) Only II is trued) Only III is truee) Only II and III are true

19. Statements: Z © F, F $ M, M % KConclusions: I. K * F

II. Z * MIII. K * Z

a) Only I is trueb) Only II is truec) Only III is trued) Only II and III are truee) None of the above

20. Statements: H @ B, B © R, A $ RConclusions: I. B * A

II. R % HIII. A $ H

a) Only I and II are true

Page 137: Reasoning Made Easy - pdfstores.files.wordpress.com · REASONING MADE EASY 6 Geetika, Jasmine, Hema, Brinda, Ipsita and veena, once again not following any order. The following are

REASONING MADE EASY www.BankExamsToday.com

137

b) Only I and III are truec) Only II and III are trued) All I, II and III are truee) None of the above

21. Statements: M $ J, J * T, K © TConclusions: I. K * J

II. M $ TIII. M $ K

a) None is trueb) Only I is truec) Only II is trued) Only III is truee) Only II and III are true

Directions (Q. 22-27) In the following questions, the symbols @, ©, #, $ and *are used with the following meaning illustrated.‘P © Q’ means “P is not smaller than Q’.‘P * Q’ means ‘P is neither greater than nor smaller than Q’.‘P @ Q’ means ‘P is neither greater than nor equal to Q’.‘P $ Q’ means ‘P is neither smaller than nor equal to Q’.‘P # Q’ means ‘P is neither smaller than nor equal to Q’.

In each of the following questions assuming the given statements to be true, find outwhich of the three conclusions I, II and III given below them is/are definitely true.

22. Statements: M @ T, T $ R, R © JConclusions: I. J # M

II. R # MIII. J * T

a) Only I is trueb) Only II is truec) Only III is trued) Only I and II are truee) None of the above

23. Statements: D © B, B # H, H * FConclusions: I. F @ B

II. F @ DIII. H @ D

a) Only I is trueb) Only II is truec) Only III is trued) Only I and II are truee) All are true

Page 138: Reasoning Made Easy - pdfstores.files.wordpress.com · REASONING MADE EASY 6 Geetika, Jasmine, Hema, Brinda, Ipsita and veena, once again not following any order. The following are

REASONING MADE EASY www.BankExamsToday.com

138

24. Statements: H * M, M @ T, T $ KConclusions: I. K # M

II. T # HIII. H @ K

a) Only I is trueb) Only I and II are truec) Only II and III are trued) Only I and III are truee) All are true

25. Statements: N $ A, A # J , J © DConclusions: I. N @ J

II. A © DIII. D @ A

a) Only I is trueb) Only II is truec) Only III is trued) Only II and III are truee) None of the above

26. Statements: R * T, T @ M, M $ KConclusions: I. K @ R

II. M # RIII. K # T

a) Only I and II are trueb) Only II and III are truec) Only I and III are trued) All are truee) None of the above

27. Statements: F # W, W $ M, M © RConclusions: I. R $ W

II. F # RIII. W * R

a) None is trueb) Only I is truec) Only II is trued) Only III is truee) Only II and III are true

Directions (Q. 28-31) Read each statement carefully and answer the followingquestions.

28. Which of the following expressions will be true, if the expression R > O = A >S < T is definitely true?a) O > T b) S < R c) T > A

Page 139: Reasoning Made Easy - pdfstores.files.wordpress.com · REASONING MADE EASY 6 Geetika, Jasmine, Hema, Brinda, Ipsita and veena, once again not following any order. The following are

REASONING MADE EASY www.BankExamsToday.com

139

d) S = O e) None of these

29. Which of the following symbols should replace the question mark (?) in thegiven expression in order to make the expressions ‘P > A’ as well as ‘T < L’definitely true?a) ≤ b) > c) <d) ≥ e) None of these

30. Which of the following should be placed in the blank spaces respectively (in thesame order from left to right) in order to complete the given expression in sucha manner that makes the expression ‘A < P’ definitely false?__ ≤ __ < __ > __a) L, N, P, A b) L, A, P, N c) A, L, P, Nd) N, A, P, L e) P, N, A, L

31. Which of the following symbols should be placed in the blank spacesrespectively (in the same order from left to right) in order to complete the givenexpression in such a manner that makes t he expression ‘F > N’ and ‘U > D’definitely false?F __ O __ U __ N __ Da) <, <, >, = b) <, =, =, > c) <, =, =, <d) ≥, =, =, ≥ e) >, >, =, <

Answers:

1. Option BK @ V ⟶ K < VV © N ⟶ V ≤ NN % F ⟶ N ≥ FK < V ≤ N ≥ F

Conclusions: I. F @ V ⟶ F < V (F)II . K @ N ⟶ K < N (T)

Only II is true.

2. Optoin EH © W ⟶ H ≤ WW $ M ⟶ W = MM @ B ⟶ M < BH ≤ W = M < B

Conclusions: I. B * H ⟶ B > H (T)II. M % H ⟶ M ≥ H (T)

So, both I and II are true.

3. Option DD % B ⟶ D ≥ B

Page 140: Reasoning Made Easy - pdfstores.files.wordpress.com · REASONING MADE EASY 6 Geetika, Jasmine, Hema, Brinda, Ipsita and veena, once again not following any order. The following are

REASONING MADE EASY www.BankExamsToday.com

140

B * T ⟶ B > TT $ M ⟶ T = MD ≥ B > T = M

Conclusions: I. T © D ⟶ T ≤ D (F)II. M © D ⟶ M ≤ D (F)

Neither I nor II is true.

4. Option AM * T ⟶ M > TT @ K ⟶ T < KK © N ⟶ K ≤ NM > T < K ≤ N

Conclusions: I. N * T ⟶ N > T (T)II. N * M ⟶ N > M (F)

Only I is true.

5. Option CR $ J ⟶ R = JJ % D ⟶ J ≥ DD * F ⟶ D > FR = J ≥ D > F

Conclusions: I. D $ R ⟶ D = RII. D @ R ⟶ D < R

Either I or II is true.

6. Option AM © D ⟶ M < DD * K ⟶ D > KK @ R ⟶ K ≤ RR * F ⟶ R > F

Conclusions : I. F © K ⟶ F < K (False)II. D * F ⟶ D > F (False)III. M © R ⟶ M < R (False)IV. D * R ⟶ D > R (False)

None is true.

7. Option EB % K ⟶ B ≥ KK $ T ⟶ K = TT * F ⟶ T > FH © F ⟶ H < F

So, B ≥ K = T > F > HConclusions: I. B $ T ⟶ B = T

II. T © B ⟶ T < BIII. H © K ⟶ H < K (True)

Page 141: Reasoning Made Easy - pdfstores.files.wordpress.com · REASONING MADE EASY 6 Geetika, Jasmine, Hema, Brinda, Ipsita and veena, once again not following any order. The following are

REASONING MADE EASY www.BankExamsToday.com

141

IV. F © B ⟶ F < B (True)Either T is smaller than B or equal to.So, only III, IV and either I or II are true.

8. Option BW * B ⟶ W > BB @ F ⟶ B ≤ FF © R ⟶ F < RR $ M ⟶ R = M

So, W > B ≤ F < R = MConclusions: I. W * F ⟶ W > F (False)

II. M * B ⟶ M > B (True)III. R * B ⟶ R > B (True)IV. M * W ⟶M > W (False)

So, Only II and III are true.

9. Option CE @ K ⟶ E ≤ KK $ T ⟶ K = TT © N ⟶ T < NB % N ⟶ B ≥ N

Conclusions: I. T % E ⟶ T ≥ E (True)II. K © N ⟶ K < N (False)III. B * T ⟶ B > T (True)IV. B * E ⟶ B > E (True)

Only I, III and IV are true.

10. Option CZ $ B ⟶ Z = BB % M ⟶ B ≥ MM © F ⟶ F ≤ RF @ R ⟶ F ≤ R

Conclusions: I. Z * M ⟶ Z > M (False)II. F * B ⟶ F > B (False)III. R * M ⟶ R > M (True)IV. M @ Z ⟶ M ≤ Z (True)

Only III and IV are true.

11. Option CH @ T ⟶ H ≤ TT $ N ⟶ T = NF © N ⟶ F < NB % F ⟶ B ≥ F

So, H ≤ T = N > F ≤ BConclusions: I. F @ H ⟶ F ≤ H (False)

Page 142: Reasoning Made Easy - pdfstores.files.wordpress.com · REASONING MADE EASY 6 Geetika, Jasmine, Hema, Brinda, Ipsita and veena, once again not following any order. The following are

REASONING MADE EASY www.BankExamsToday.com

142

II. F © T ⟶ F < T (True)III. B * T ⟶ B > T (False)IV. B % H ⟶ B ≥ H (False)

Only II is true.

12. Option A13. Option B14. Option D15. Option E16. Option D17. Option E

F % T ⟶ F ≤ TT @ J ⟶ T = JJ * W ⟶ J > WFrom all the statements together,F ≤ T = J > WConclusions: I. J @ F ⟶ J = F

II. J * F ⟶ J > FIII. W $ T ⟶ W < T

J is either greater than or equal to F. So either I or II and III are true.

18. Option CR * D ⟶ R > DD © K ⟶ D ≥ KK $ M ⟶ K < MFrom all the statements together,R > D ≥ K < MConclusions: I. M * R ⟶ M > R (False)

II. K $ R ⟶ K < R (True)III. D * M ⟶ D > M (False)

So, only II is true.

19. Option AZ © F ⟶ Z ≥ FF $ M ⟶ F < MM % K ⟶ M ≤ KFrom all the statements together,Z ≥ F < M ≤ KConclusions: I. K * F ⟶ K > F (True)

II. Z * M ⟶ Z > M (False)III. K * Z ⟶ K > Z (False)

So, only I is true.

20. Option DH @ B ⟶ H = B

Page 143: Reasoning Made Easy - pdfstores.files.wordpress.com · REASONING MADE EASY 6 Geetika, Jasmine, Hema, Brinda, Ipsita and veena, once again not following any order. The following are

REASONING MADE EASY www.BankExamsToday.com

143

B © R ⟶ B ≥ RA $ R ⟶ A < RFrom all the three statements together,H = B ≥ R > AConclusions: I. B * A ⟶ B > A (True)

II. R % H ⟶ R ≤ H (True)III. A $ H ⟶ A < H (True)

So, all I, II and III are true.

21. Option AM $ J ⟶ M < JJ * T ⟶ J > TK © T ⟶ K ≥ TFrom all the three statements together,M < J < T ≤ KConclusions: I. K * J ⟶ K > J (False)

II. M $ T ⟶ M < T (False)III. M $ K ⟶ M < K (False)

None is true.

22. Option B23. Option E24. Option E25. Option C26. Option B27. Option A28. Option B29. Option B30. Option E31. Option C

Page 144: Reasoning Made Easy - pdfstores.files.wordpress.com · REASONING MADE EASY 6 Geetika, Jasmine, Hema, Brinda, Ipsita and veena, once again not following any order. The following are

REASONING MADE EASY www.BankExamsToday.com

144

Chapter – 9

Coding Decoding

Directions (Q. 1-3) Study the information given below carefully to answer thefollowing questions.

In a certain code language the following lines written as ‘lop eop aop fop’means ‘Traders are above laws’‘fop cop bop gop’ means ‘Developers were above profitable’‘aop bop uop qop’ means ‘Developers stopped following traders’‘cop jop eop uop’ means ‘Following maps were laws’

1. ‘Developers are following laws’ would be correctly written asa) ‘bop cop uop eop’b) ‘lop bop eop uop’c) ‘oup cop lop aop’d) ‘gop cop uop qop’e) None of these

2. ‘qop gop cop eop’ would correctly meana) profitable laws were stoppedb) developers stopped following lawsc) traders were above profitabled) were laws profitable traderse) None of these

3. ‘aop qop bop’ would correctly meana) following were aboveb) traders stopped developersc) developers are lawsd) traders above stoppede) laws are stopped

Directions (Q. 4-8) Study the information and answer the following questions.In a certain code language.‘Economics is not money’ is writen as ‘ka la ho ga’‘Demand and supply economies’ is written as ‘mo ta pa ka’‘Money makes only part’ is written as ‘zi la ne ki’‘Demand makes supply economics’ is written as ‘zi mo ka ta’

4. What is the code for ‘money’ in the given code language?a) ga b) mo c) pad) ta e) la

5. What is the code for ‘supply’ in the given code language?

Page 145: Reasoning Made Easy - pdfstores.files.wordpress.com · REASONING MADE EASY 6 Geetika, Jasmine, Hema, Brinda, Ipsita and veena, once again not following any order. The following are

REASONING MADE EASY www.BankExamsToday.com

145

a) Only ta b) Only mo c) Either pa or mod) Only pa e) Either mo or ta

6. What may be the possible code for ‘demand only more’ in the given codelanguage?a) xi ne mo b) mo zi ne c) ki ne mod) mo zi ki e) xi ka ta

7. What may be the possible code for ‘work and money’ in the given codelanguage?

a) pa ga la b) pa la lu c) mo la pad) tu la ga e) pa la ne

8. What is the code for ‘makes’ in the given code language?a) mo b) pa c) ned) ho e) None of these

Directions (Q. 9-13) Study the following information and answer the givenquestions.

In a certain code language ‘driving is not easy’ is written as ‘jo ro ho go’,‘rough and tough driving’ is written as ‘no sa jo da’, ‘looks easy but dangerous’ iswritten as ‘ai ro to po’ and ‘is rough tough dangerous driving’ is written as ‘ho jo noai da’

9. What is the code for ‘easy’ in the given code language?a) go b) no c) tod) ro e) so

10. What is the code for ‘tough’ in the given code language?a) no b) da c) sad) Either sa or da e) Either no or da

11. What may be the possible code for ‘rough but nice’?a) po bi no b) no po ai c) no to pod) no to ai e) ta jo bi

12. What may be the possible code for ‘easy and solved’ in the given codelanguage?a) sa go ro b) sa ro cu c) no ro sud) cu ro go e) sa ro to

13. What is the code for ‘dangerous’ in the given code language?a) no b) sa c) tod) ai e) ho

Page 146: Reasoning Made Easy - pdfstores.files.wordpress.com · REASONING MADE EASY 6 Geetika, Jasmine, Hema, Brinda, Ipsita and veena, once again not following any order. The following are

REASONING MADE EASY www.BankExamsToday.com

146

Directions (Q. 14-18) Study the following information carefully and answer thequestions.

In a certain code language, ‘newspaper enhance the knowledge’ is writtenas ‘yo no co po’, ‘new enhance good version’ is written as ‘co so ro jo’, ‘goodknowledge is necessary’ is written as ‘bo t o ro po’, ‘the new brand book’ is writtenas ‘qo jo do yo’, and ‘book is necessary’ is written as ‘bo do to’.

14. What is the code for ‘version’?a) so b) ro c) jod) Can’t be determined e) None of these

15. Which of the following is the code for ‘brand enhance version’?a) do qo co b) so qo co c) do ro cod) Can’t say e) None of these

16. What is the code for ‘new’?a) so b) ro c) jod) yo e) None of these

17. What is the code for ‘necessary’?a) bo b) to c) rod) Can’t say e) None of these

18. ‘necessary is tal ent’ willl be coded asa) mo bo to b) mo lo to c) bo to nod) can’t say e) None of these

Directions (Q. 19-22) Study the following information to answer the givenquestions.

In a certain code, ‘ze lo ka gi’ is a code for ‘must save some money’, ‘fe kaso ni’ is a code for ‘he made good money’, ni lo da so’ is a code for ‘he must begood’ and ‘we so ze da’ is a code for ‘be good save grace’.

19. Which of the following is the code of ‘must’?a) so b) da c) lod) ni e) None of these

20. What does the code ‘ze’ stand for?a) some b) must c) bed) grave e) save

21. Which of the following is the code of ‘good’?a) so b) we c) zed) lo e) fe

Page 147: Reasoning Made Easy - pdfstores.files.wordpress.com · REASONING MADE EASY 6 Geetika, Jasmine, Hema, Brinda, Ipsita and veena, once again not following any order. The following are

REASONING MADE EASY www.BankExamsToday.com

147

22. ‘grace of money’ may be coded asa) ka da fe b) we ka so c) ja da wed) ka we yo e) ja ka ze

Directions (Q. 23-27) In each question below is given a group of letters followedby four combinations of digits/symbols numbered a, b, c, and d. You have tofind out which of the combinations correctly represents the group of lettersbased on the following coding system and the conditions and mark the numberof that combination as your answer. If none of the combinations correctlyrepresents the group of letter, mark e) i.e., None of these as your answer.

Letter R A T K F Q E P J I M U D HDigit/Symbol Code 3 @ 2 1 9 4 © % 5 8 $ 6 7 @

Conditions:(i) If the first letter is a consonant and the last letter is a vowel, their codes

are to be interchanged.(ii) If both the first and the last letters are vowels, both are to be coded as *.(iii) If both the first and the last letters are consonants, both are to be coded

as the code for the last letter.

23. DPEHQAa) 7%©#4@ b) @%©#47 c) 7%©#47d) @%©#4@ e) None of these

24. KEMRDFa) *©$379 b) 1©$379 c) 1©$37*d) *©$37* e) None of these

25. AHDUQKa) 1#764@ b) 1#7641 c) *#764*d) @#764@ e) None of these

26. IDQJPTa) 8745%2 b) 2745%8 c) *745%*d) 2745%2 e) None of these

27. UDKFMEa) *719$© b) 6719$© c) *719$*d) ©719#$6 e) None of these

28. If ‘white’ means ‘black’, ‘black’ means ‘red’, ‘red’ means ‘blue’. ‘blue’means ‘yellow’ and ‘yellow’ means ‘grey’ then which of the followingrepresents the colour of clear sky?

Page 148: Reasoning Made Easy - pdfstores.files.wordpress.com · REASONING MADE EASY 6 Geetika, Jasmine, Hema, Brinda, Ipsita and veena, once again not following any order. The following are

REASONING MADE EASY www.BankExamsToday.com

148

a) Blue b) Red c) Yellowd) Can’t be determined e) None of these

29. In a certain code, MODEL is written as ‘513#2’ and DEAR is written as‘3#%8’. How is LOAD written in that code?a) 21%3 b) 23%1 c) 25%3d) 21#3 e) None of these

Directions (Q. 30-34) In each question below is given a group of letters followedby four combinations of digits/symbols numbered a, b, c and d. You have tofind out which of the combinations correctly represents the group of lettersbased on the following coding system and the conditions and mark the numberof that combination as your answer. If none of the four combinations correctlyrepresents the group of letters, give e) i.e. ‘None of these’ as the answer.

Letters: M D R P A T W E I F H U K ZDigit/Symbold Code: 5 6 # 7 8 1 @ $ 2 % 3 © 4 9

Conditions:(i) If the first letter is a coinsonant and the last letter is a vowel, their

codes are to be interchanged.(ii) If both the first and the last letters are vowels, both are to be coded as *.(iii) If the first letter is a vowel and the last letter is a consonant, both are to

be coded as the code for the consonant.

30. TUKDIPa) 1©4627 b) 1©4621 c) 7©4621d) 1©6427 e) None of these

31. EFDMKAa) $%6548 b) $%654$ c) *%654*d) 8%6548 e) None of these

32. APWTUHa) *7@1© * b) 87@1©3 c) 37@@5d) 87@1©8 e) None of the above

33. MARTWEa) 58#1@$ b) 58#1@5 c) $8#1@5d) $8#1@$ e) None of these

34. HEMKZIa) 2$5493 b) 3$5492 c) 3$5493d) 2$5492 e) None of these

Page 149: Reasoning Made Easy - pdfstores.files.wordpress.com · REASONING MADE EASY 6 Geetika, Jasmine, Hema, Brinda, Ipsita and veena, once again not following any order. The following are

REASONING MADE EASY www.BankExamsToday.com

149

35. In a certain code SAFER is written as 5@3#2 and RIDE is written as2©%#. How is FEDS written in that code?a) 3#©5 b) 3@%5 c) 3#%5d) 3#%2 e) None of these

36. In a certain code CONFUSED is written as EMBEFTV. How isSECLUDED written in that code?a) RDBKEFEV b) KBDRCDCT c) KBDREFEVd) MDFTCDCT e) None of these

37. In a certain code language ‘bring the white board’ is written as ‘ka na di pa’and ‘white and black board’ is written as ‘na di sa ra’. How is ‘the’ writtenin that code?a) ka b) pa c) ka or pad) Data inadequate e) None of these

Directions (Q. 38-43) In each question below is given a group of letters followedby four combinations of digits/symbols numbered a, b, c, and d. You have tofind out which of the combinations correctly represents the group of letter,based on the following coding system and the conditions that follow and markthe number of thatcombination as your answer. If none of the combinationcorrectly represents the group of letter, mark e) i.e. ‘None of these’ as youranswer.

Letter: R D A E J M K T B U I P W H FDigit/Symbol Code: 4 8 5 $ * 1 2 6 % © 7@ 3 9 #

Conditions:(i) If the first letter is a consonant and the last letter is a vowel both are to

be coded as ^.(ii) If both the first and the last letters are consonants both are to be coded

as the code for the last letter.(iii) If the first letter is a vowel and the last letter is a consonant their codes

are to be interchanged.

38. HIFMJUa) ©7#1*9 b) 97#1*9 c) ©7#1*©d) ^7#1*^ e) None of these

39. AKTRBWa) 3264%5 b) 3264%3 c) 324%65d) 5264%3 e) None of these

40. EBPDRIa) 7%@847 b) 7%@84$ c) $%@847d) $%8@47 e) None of these

Page 150: Reasoning Made Easy - pdfstores.files.wordpress.com · REASONING MADE EASY 6 Geetika, Jasmine, Hema, Brinda, Ipsita and veena, once again not following any order. The following are

REASONING MADE EASY www.BankExamsToday.com

150

41. BKAJIMa) %25*71 b) 125*7% c) %25*7%d) 15*271 e) None of these

42. UKPDMAa) 52@815 b) ©2@815 c) ©2@81©d) ©@2815 e) None of these

43. METUFBa) %$6©#1 b) 1$6©#1 c) %$6©#%d) 1$6©#% e) None of these

Directions (Q. 44-48) In each question below is given a group of letters followedby four combinations of digits/symbols numbered a, b, c and d. You have tofind out whi ch of the combinations correctly represents the group of lettersbased on the following conditions and the sub-conditions and mark the numberof that combination as your answer. If none of the four combinations correctlyrepresents the group of letters, mark e) i.e. ‘None of these’ as your answer.

Letter: A M P R D H J T W E I K N F UDigit/Symbol: 3 % 4 9 1 @ 2 5 © 6 * 7 8 $ ^

Conditions:(i) If the first letter is a consonant and the last letter is a vowel, their codes

are to be interchanged.(ii) If both the first as well as last letters are consonants, both are to be

coded as the code for the last letter.(iii) If the first letter is a vowel and the last letter is a consonant, both are to

be coded as ‘#’.

44. APIWKNa) 34*©73 b) #4*©7# c) 84*©78d) 84*©73 e) None of these

45. HPEIUMa) @46*^% b) %46*^© c) @46*^@d) #46*^# e) None of these

46. RDEMKHa) @16%79 b) 916%7@ c) @16%7@d) 916%79 e) None of these

47. FWHRKEa) 6©@976 b) 6$ ©@97 c) 6©@97$d) $©@976 e) None of these

Page 151: Reasoning Made Easy - pdfstores.files.wordpress.com · REASONING MADE EASY 6 Geetika, Jasmine, Hema, Brinda, Ipsita and veena, once again not following any order. The following are

REASONING MADE EASY www.BankExamsToday.com

151

48. IDAPRUa) *1349^ b) ^*1349 c) ^1349*d) ^1349^ e) None of these

49. In a certain code CROWNED is written as PSDVEFO. How is STREAMSwritten in that code?a) SUTDBNT b) TUSDTNB c) SUTFTNBd) QSRDTNB e) None of these

50. ‘DEAN’ is related to ‘NDAE’ and ‘ROAD’ is related to ‘DRAO’ in thesame way as ‘SOME’ is related toa) ESMO b) EOMS c) EMOSd) MSEO e) None of these

51. In a certain code TRAIN is written as ‘39*7%’ and MEAL is written as‘4$*@’. How is ITEM written in that code?a) 7$34 b) 73$4 c) 79$4d) 73*4 e) None of the above

52. In a certain code language ‘over and above’ is written as ‘da pa ta’ and ‘oldand beautiful’ is written as ‘sa na pa’. How is ‘over’ written in that codelanguage?a) da b) ta c) nad) da or ta e) None of these

Directions (Q. 53-57) Study the following information carefully and answer thequestions given below

In a certain code language, “exercise for your health” is written as “ja ka na se”“Health and exercise must” is written as “ka se re tu”“Must to exercise regularly” is written as “az ka dk tu”“To your good life” is written as “mo ja fu az”

53. Which of the following may represent “exercise regularly good for health”?a) Ja m o re dk azb) fu ja re tu dkc) ka se mo na dkd) ka re az tu see) na se tu az dk

54. What is the code for “life”?a) mo b) fu c) jad) az e) Either mo or fu

55. What is the code for “must exercise”?

Page 152: Reasoning Made Easy - pdfstores.files.wordpress.com · REASONING MADE EASY 6 Geetika, Jasmine, Hema, Brinda, Ipsita and veena, once again not following any order. The following are

REASONING MADE EASY www.BankExamsToday.com

152

a) tu se b) ka tu c) ka azd) fu tu e) na ka

56. Find the odd one out.a) January, Mayb) April, Junec) July, Augustd) January, Decembere) None of these

57. Which one of the given responses would be a meaningful order of thefollowing?1. Earth2. Jupiter3. Venus4. Mars5. Mercurya) 5, 3, 1, 2, 4 b) 5, 3, 4, 1, 2 c) 5, 3, 1, 4, 2d) 5, 3, 2, 4, 1 e) None of these

58. If ‘NEWS’ is written as ‘WENS’ then how ‘MATE’ will be written in thiscode?a) TAME b) META c) EATMd) AMET e) None of these

59. If REASON is coded as 5 and BELIEVED as 7, what is the code numberfor GOVERNMENT?a) 6 b) 8 c) 9d) 10 e) None of these

Directions (Q. 60-65) In each question below is given a group of letters followedby four combinations of digits/symbols numbered a, b, c and d. you have tofind out which of the combinations correctly represents the group of lettersbased on the following digits/symbols coding system and the condition thosefollow and mark the number of that combination as the answer. If none of thecombinations correctly represents the group of letters mark e) i.e. None ofthese as the answer.

Letter: P M A I D E J K F N Q B U W TDigit/Symbol Code 6 9 5 # 7 $ 1 % 2 @ 8 © 3 * 4

Conditions:(i) If the first letter is a consonant and the last letter is a vowel the codes

are to be interchanged.(ii) If the first letter is a vowel and the last letter is a consonant both are to

be coded as the code for the vowel.

Page 153: Reasoning Made Easy - pdfstores.files.wordpress.com · REASONING MADE EASY 6 Geetika, Jasmine, Hema, Brinda, Ipsita and veena, once again not following any order. The following are

REASONING MADE EASY www.BankExamsToday.com

153

(iii) If both the first and the last letters are consonants both are to be codedas the code for the last letter.

60. MKJIDEa) 9%1#7$ b) $%1#79 c) 91%#7$d) $%17#9 e) None of these

61. INQBWUa) #@8©*3 b) 3@8©*# c) #8@*©3d) 3#@8©* e) None of these

62. KFBPAWa) *2©65% b) % 2©65% c) *2©65*d) %2©65* e) None of these

63. EFDJTPa) 62714$ b) $27146 c) $27416d) $2714$ e) None of these

64. NWANUDa) @*5@37 b) 7*5@3@ c) @5*@37d) @*5@3@ e) None of these

65. APFTQIa) #62485 b ) #6248# c) 562485d) 56248# e) None of these

66. In a certain code if LOUD is written as JQSF then which of the followingEnglish words shall b e coded as PKQG?a) RISE b) ROPE c) ROADd) RICE e) ROLE

Directions (Q. 67-72) In each question below is given a group of letters followedby four combinations of digits/symbols numbered a, b, c, d, you have to findout which of the combinations correctly represents the group of letters basedon the following letter coding system and mark the number of thatcombination as the answer. If none of the digits/symbols combinationscorrectly represents the group of letters, mark e) i.e., None of these as theanswer.

Letter: D L E G Z K R U B W F H I A PDigit/Symbol: 4 8 $ 1 # 5 7 © 2 6 % * 3 9 @

Conditions:(i) If the first letter is a vowel and the last is a consonant, both are to be

coded as the code of the consonant.

Page 154: Reasoning Made Easy - pdfstores.files.wordpress.com · REASONING MADE EASY 6 Geetika, Jasmine, Hema, Brinda, Ipsita and veena, once again not following any order. The following are

REASONING MADE EASY www.BankExamsToday.com

154

(ii) If both first and the last letters are consonants, both are to be coded as ^.(iii) If first letter is a consonant and last is a vowel, the codes for first and

last letters are to be interchanged.

67. ABWUPFa) %26©@% b) %26©@9 c) 926©@%d) %26©9@ e) None of these

68. BFIDWEa) 2%346$ b) %3426$ c) $%3426d) $2%634 e) None of these

69. WKGLBAa) 951862 b) 651829 c) 651892d) 951826 e) None of these

70. ZEFHIRa) #$%*37 b) ^$%*3^ c) 7$%*3#d) #%$*37 e) None of these

71. ELBGPUa) $821@© b) ©821@$ c) $821©@d) $812@© e) None of these

72. UHRKLZa) ©*758© b) ©*758# c) #*758#d) #*758© e) None of these

73. In a certain code, LAWN is written as JCUP. How will SLIT be coded inthat code?a) QNGV b) QJGV c) ONVGd) NJGV e) NJVG

74. In a certain code PRAISE is written as #@$27% and RESPIRE is written as@7#2@%. How is REPAIR written in that code?a) @%#2$@ b) @%$#2@ c) @%#$2@d) @%$2#@ e) None of these

75. In a certain code SATELLITE is written as FUBTLDSHK. How isLAUNCHING written in that code?a) DOUBFGMHOb) OVBMCFMHGc) OVMBCFMGHe) DOUBCFMHGe) None of these

Page 155: Reasoning Made Easy - pdfstores.files.wordpress.com · REASONING MADE EASY 6 Geetika, Jasmine, Hema, Brinda, Ipsita and veena, once again not following any order. The following are

REASONING MADE EASY www.BankExamsToday.com

155

Directions (Q. 76-81) In each of these questions, a group of letters is givenfollowed by four combinations of digit/symbol numbered a, b, c, d. Letter areto be coded as per the scheme and conditions given below. You have to findout the serial number of the combination, which represents the letter group. Serial number of that combination is your answer. If none of the combinations iscorrect, your answer is e) i.e., None of these

LettersF

G A KM

E S PL

Q B U R I T

Digit/SymbolCode

9 % 2 $ * 7 @ 8 1 6 5 © 4 # 3

Conditions:i) If the first letter is a vowel and the last a consonant, both are to be coded as

the code for the vowel.ii) If the first letter is consonant and the last a vowel, the codes for the first and

the last letters are to be interchanged.iii) If both the first and the last letters are consonant both are to be coded as ^iv) If more than two vowels are there in the group of letters, all vowels are to

coded as --

76. BSQEGUa) 5@67%© b) ^@67%^ c) ©@67%©d) 5@67©% e) None of these

77. MGSELIa) *%@71# b) *%@17# c) #%@71*d) %#@17* e) None of these

78. IPKUSRa) #8$©@# b) 48$©@# c) #8$©@4d) #8$©4@ e) None of these

79. TEFSUMa) ^79@©^ b) 379@©* c) *79@©3d) ^79@^© e) None of these

80. KAGFUBa) $2%9©5 b) 52%9©$ c) $2%©95d) ^2%9©^ e) None of these

81. AQMMTEa) 76©*32 b) –6--*3-- c) 26©*32d) 26©*23 e) None of these

Page 156: Reasoning Made Easy - pdfstores.files.wordpress.com · REASONING MADE EASY 6 Geetika, Jasmine, Hema, Brinda, Ipsita and veena, once again not following any order. The following are

REASONING MADE EASY www.BankExamsToday.com

156

82. In a certain code JUST is written as #@%$ and LATE is written as ©^$*.How is TASTE written in that code?a) *^%$* b) $^%$* c) $^%*$d) $%^%* e) None of these

83. If Blue is called Green, Green is called Orange, Orange is called Yellow,Yellow is called Black, Black is called Red and Red is called White. Whatis the colour of turmeric?a) Orange b) Green c) Whited) Black e) None of these

84. In a certain code ROSE is wrtitten as #43$ and FIRST is written as 5*#37.How is STORE written in that code?a) 473$# b) 473#$ c) 374#$d) 347#$ e) None of these

85. In a certain code, DESPAIR is written as TFEQSJB. How is NUMERALwritten in that code?a) OVNFMBS b) NVOFSBN c) NVOMFBSd) NVOFMBS e) None of these

86. If ‘yellow’ is called ‘red’, ‘red’ is called ‘blue’, ‘blue’ is called ‘white’,‘white’ is called ‘black’, ‘black’ is called ‘green’ and ‘green is called‘violet’, what is the colour of clear sky?a) Green b) Violet c) Yellowd) Red e) None of these

87. In a certain code, STAR is written as 5$*2 and TORE is written as [email protected] is OATS written in that code?a) 3*5$ b) 3*$5 c) 3$*5d) 35*$ e) None of these

88. In a certain code language, ‘food is good’ is written as ‘ho na ta’, ‘eat foodregularly’ is written as ‘sa ta la’ and ‘keep good health’ is written as ‘da naja’. How is ‘eat’ written in that code language?a) sa b) la c) sa or lad) Data inadequate e) None of these

89. In a certain code, CORDIAL is written as ‘SPDCMBJ’. How isSOMEDAY written in that code?a) NPTDEBZ b) NPTFZBE c) TPNDZBEd) NPTDZBE e) None of these

90. In a certain code DOES is written as ‘5$3%’ and SITE is written as‘%4#3’. How is EDIT written in that code?a) 354# b) 3#54 c) 3$4#

Page 157: Reasoning Made Easy - pdfstores.files.wordpress.com · REASONING MADE EASY 6 Geetika, Jasmine, Hema, Brinda, Ipsita and veena, once again not following any order. The following are

REASONING MADE EASY www.BankExamsToday.com

157

d) 35$# e) None of these

Answers:

1. Option B2. Option A3. Option B4. Option E5. Option E6. Option A7. Option B8. Option D9. Option D10. Option E11. Option A12. Option B13. Option D14. Option A15. Option B16. Option C17. Option D18. Option A19. Option C20. Option E21. Option A22. Option D23. Option B

D P E H Q A@ % © # 4 7

24. Option EK E M R D F9 © $ 3 7 9

Condition (iii) follows

25. Option EA H D U Q K@ # 7 6 4 1

No condition follows.

26. Option AI D Q J P T8 7 4 5 % 2No condition follows.

Page 158: Reasoning Made Easy - pdfstores.files.wordpress.com · REASONING MADE EASY 6 Geetika, Jasmine, Hema, Brinda, Ipsita and veena, once again not following any order. The following are

REASONING MADE EASY www.BankExamsToday.com

158

27. Option C

U D K F M E* 7 1 9 $ *Condition (ii) follows

28. Option CColour of clear sky is blue and according to question, blue means yellow sothe colour of clear sky will be ‘yellow’.

29. Option AM O D E L5 1 3 # 2

D E A R3 # % 8

L O A D2 1 % 3

30. Option AT U K D I P1 © 4 6 2 7Condition (i) follows.

31. Option CE F D M K A* % 6 5 4 *Condition (ii) follows.

32. Option EA P W T U H3 7 @ 1 © 3Condition (iii) follows.

33. Option CM A R T W E$ 8 # 1 @ 5Condition (i) follows.

34. Option AH E M K Z I2 $ 5 4 9 3Condition (i) follows.

35. Option C

Page 159: Reasoning Made Easy - pdfstores.files.wordpress.com · REASONING MADE EASY 6 Geetika, Jasmine, Hema, Brinda, Ipsita and veena, once again not following any order. The following are

REASONING MADE EASY www.BankExamsToday.com

159

S A F E R5 @ 3 # 2

R I D E2 © % #

F E D S3 # % 5

36. Option C

37. Option CBring the white board ⟶ ka na di pa (i)White and black board ⟶ na di sa ra (ii)From equations (i) and (ii),White board = na diPut the code of white board in equation (i)Bring the = ka or pa

38. Option DH I F M J U^ 7 # 1 * ^Condition (i) follows.

39. Option AA K T R B W3 2 6 4 % 5Condition (iii) follows.

40. Option CE B P D R I$ % @ 8 4 7No condition follows.

41. Option EB K A J I M1 2 5 * 7 1Condition (ii) follows.

42. Option BU K P D M A© 2 @ 8 1 5

43. Option CM E T U F B% $ 6 © # %

Page 160: Reasoning Made Easy - pdfstores.files.wordpress.com · REASONING MADE EASY 6 Geetika, Jasmine, Hema, Brinda, Ipsita and veena, once again not following any order. The following are

REASONING MADE EASY www.BankExamsToday.com

160

Condition (ii) follows.

44. Option BA P I W K N# 4 * © 7 #Condition (iii) follows.

45. Option EH P E I U M% 4 6 * ^ %Condition (ii) follows.

46. Option CR D E M K H@ 1 6 % 7 @Condition (ii) follows.

47. Option CF W H R K E6 © @ 9 7 $Condition (i) follows.

48. Option AI D A P R U* 1 3 4 9 ^

49. Option E50. Option A

1 2 3 4 4 1 3 2D E A N N D A E

And 1 2 3 4 4 1 3 2R O A D D R A O

So, 1 2 3 4 4 1 3 2S O M E E S M O

51. Option BT R A I N3 9 * 7 %

And M E A L4 $ * @

So, I T E M7 3 $ 4

52. Option D

Page 161: Reasoning Made Easy - pdfstores.files.wordpress.com · REASONING MADE EASY 6 Geetika, Jasmine, Hema, Brinda, Ipsita and veena, once again not following any order. The following are

REASONING MADE EASY www.BankExamsToday.com

161

Over and above da pa taOld and beautiful sa na paSo, the code of over will be da or ta.

53. Option CExercise ⟶ kaRegularly ⟶ dkGood ⟶ mo or fuFor ⟶ naHealth ⟶ se

54. Option EThe code for “life” may be “mo” or “fu”.

55. Option BMust ⟶ tuExercise ⟶ ka

56. Optoin BExcept the months of April and June, all other months have 31 days each.

57. Option B

58. Option A1 2 3 4 3 2 1 4N E W S W E N SSimilarly,1 2 3 4 3 2 1 4M A T E T A M E

59. Option CREASON ⟶ 5Number of letters ⎯1BELIEVED ⟶ 8 ⎯1 = 7Similarly,GOVERNMENT ⟶ 10 ⎯1 = 9

60. Option B61. Option A62. Option C63. Option D64. Option E65. Option D66. Option C67. Option A

Page 162: Reasoning Made Easy - pdfstores.files.wordpress.com · REASONING MADE EASY 6 Geetika, Jasmine, Hema, Brinda, Ipsita and veena, once again not following any order. The following are

REASONING MADE EASY www.BankExamsToday.com

162

68. Option E69. Option D70. Option B71. Option A72. Option C73. Option A74. Option C75. Option B76. Option E77. Option C78. Option D79. Option A80. Option D81. Option B82. Option B83. Option D84. Option C85. Option D86. Option E87. Option B88. Option C89. Option D90. Option A

Page 163: Reasoning Made Easy - pdfstores.files.wordpress.com · REASONING MADE EASY 6 Geetika, Jasmine, Hema, Brinda, Ipsita and veena, once again not following any order. The following are

REASONING MADE EASY www.BankExamsToday.com

163

Chapter – 10

Decision Making

Directions (Q. 1-5) Read the following information carefully and answer thequestions given below:

Following are the conditions for admitting students in an EngineeringCollege:

(i) Have secured at least 75% agreegate marks in XII Std. exam in sciencestream.

(ii) Have secured an average of at least 85% marks in Physics, Chemistryand Mathematics in XII Std. exam.

(iii) Pay Rs.50, 000 at the time of admission.(iv) Be at least 17 years old as on 01st May 2014

However, in the case of a candidate who fulfils all the criteria except(a) (ii) Above, but has secured 90% aggregate marks, he/she should be referred

to the Principal.(b) (iii) Above, but pays at least Rs.40000 he/she should be referred to the

Chairman, Admission.

Based on the above criteria and information provided below, make a decision ineach case. You are not to assume anything. If the data given are not enough to takea decision mark your answer as ‘data I nadequate’. These cases are given to you ason 01.05.2014. Give answer

a) If the case is to be referred to the Principalb) If the student is to be admittedc) If the data are inadequate to take a decisiond) If the student is not to be admittede) If the case is to be referred to the Chairman Admission.

1. Nisha Malhotra has secured aggregate 80% marks I n XII Std. exam inscience stream. She can pay Rs.40,000 at the time of admission. She hassecured 86% average marks in Physics, Chemistry and Mathematics.a) If the case is to be referred to the Principalb) If the student is to be admittedc) If the data are inadequate to take a decisiond) If the student is not to be admittede) If the case is to be referred to the Chairman Admission.

2. Arindam Ghosh was born on 25 August 1995. He can pay Rs.60,000 at thetime of admission. He has secured 80% aggregate marks in XII Std. exam

Page 164: Reasoning Made Easy - pdfstores.files.wordpress.com · REASONING MADE EASY 6 Geetika, Jasmine, Hema, Brinda, Ipsita and veena, once again not following any order. The following are

REASONING MADE EASY www.BankExamsToday.com

164

in science stream and also 85% average marks in Physics, Chemistry andMathematics.a) If the case is to be referred to the Principalb) If the student is to be admittedc) If the data are inadequate to take a decisiond) If the student is not to be admittede) If the case is to be referred to the Chairman Admission.

3. Arun Khanna has secured 90% aggregate marks in XII Std. exam in sciencestream and he can also pay Rs.50,000 at the time of admission. He was bornon Feb. 20, 1996. He has secured 89% average marks in Physics, Chemistryand Mathematics.a) If the case is to be referred to the Principalb) If the student is to be admittedc) If the data are inadequate to take a decisiond) If the student is not to be admittede) If the case is to be referred to the Chairman Admission.

4. Kiran Thapar was born on July 15, 1994. He can pay Rs.40,000 at the timeof admission. He has secured 79% average in Physics, Chemistry andMathematics. He has secured 86% aggregate marks in XII Std. exam inscience stream.a) If the case is to be referred to the Principalb) If the student is to be admittedc) If the data are inadequate to take a decisiond) If the student is not to be admittede) If the case is to be referred to the Chairman Admission.

5. Vimal Upadhyaya was born on September 20, 1996. He has secured 78%aggregate marks in XII Std. exam in science stream. He can pay Rs.42,000at the time of admission. He has secured 88% marks in Physics, Chemistryand Mathematics.a) If the case is to be referred to the Principalb) If the student is to be admittedc) If the data are inadequate to take a decisiond) If the student is not to be admittede) If the case is to be referred to the Chairman Admission.

Directions (Q. 6-10) Study the following information carefully and answer thequestions given below.

Following are the conditions for selecting Marketing Manager in an organization:

The candidate must ____(i) Be at least 30 years old as on 01.03.2009(ii) Have secured at least 55% marks in graduation

Page 165: Reasoning Made Easy - pdfstores.files.wordpress.com · REASONING MADE EASY 6 Geetika, Jasmine, Hema, Brinda, Ipsita and veena, once again not following any order. The following are

REASONING MADE EASY www.BankExamsToday.com

165

(iii) Have secured at least 60% marks in Post Graduate4 Degree/Diploma inMarketing.

(iv) Have post qualification work experience of at least five years in th eMarketing Division of an organization

(v) Have secured at least 45% marks in the selection process.

In the case of a candidates who satisfies all other conditions except(a) At (iv) above, but has post qualification work experience of at least two

years as Deputy Marketing Manager, the case is to be referred to GM-Marketing.

(b) At (ii) above, but has secured at least 65% marks in Post Graduate Degree/Diploma in Marketing Management, the case is to be referred to VicePresident-Marketing.

In each question below is given details of one candidate. You have to take one ofthe following courses of action based on the information provided and theconditions and sub-conditions given above and mark your answer accordingly. Youare not to assume anything other than the information provided in each question. Allthese cases are given to you as on 01.03.2009.

Give answer:a) If the candidate is not to be selected.b) If the candidate is to be selected.c) If the data are inadequate to take a decision.d) If the case is to be referred to Vice President-Marketing.e) If the case is to be referred to GM-Marketing.

6. Suresh Mehta has secured 58% marks in Graduation. He was born on 19th May1975. He has secured 50% marks in the selection process. He h as been workingfor the past seven years in the Marketing devision of an organization aftercompleting his Post Graduation with 62% marks.

7. Sudha Gopalan has secured 50% marks in both selection process andGraduation. She has been working for the past six years in the marketingdivision of an organization after completing her Post Graduate Diploma inMarketing with 70% marks. She was born on 14th October 1978.

8. Divya Kohli has been working for the past five years in Marketing Division ofan organization after completing her Post Graduate Diploma in Marketing with65% marks. She has secured 55% marks in Graduation and 50% marks in theselection process. She was born on 02nd April 1979.

9. Navin Marathe was born on 08th April 1975. He has secured 60% marks in bothGraduation and Post Graduate Degree in Marketing. He has been working forthe past six years in the Marketing division of an organization after completing

Page 166: Reasoning Made Easy - pdfstores.files.wordpress.com · REASONING MADE EASY 6 Geetika, Jasmine, Hema, Brinda, Ipsita and veena, once again not following any order. The following are

REASONING MADE EASY www.BankExamsToday.com

166

his PG Degree in Marketing. He has secured 50% marks in the selectionprocess.

10. Varun Malhotra was born on 03rd July 1976. He has been working as DeputyMarketing Manager in an organization for the past three years after completinghis Post Graduate Degree in Marketing with 65% marks. He secured 55%marks in both Graduation and selection process.

Directions (Q. 11-20) Study the following information carefully and answer thequestions given below.

Following are the conditions for selecting Personnel Manager in an organization:

The candidate must:(i) Be a graduate with at least 50% marks.(ii) Have a Post Graduate Degree/Diploma in personnel Management /HR

with at least 60% marks.(iii) Not be more than 35 years as on 01.06.2009(iv) Have post qualification work experience of at least five years in the

Personnel/HR Division of an organisation.(v) Have secured at least 45% marks in the selection process.

In the case of a candidate who satisfies all the conditions except-(a) (iii) Above, but has post-qualification work experience of at least ten years,

the case is to be referred to the Director-Personnel.(b) At (iv) above, but has post-qualification work experience as Deputy

Manager of at least three years, th e case is to be referred to President-Personnel.

In each question below are given details of one candidate. You have to take one ofthe following courses of action based on the information provided and theconditions and sub-conditions given above and mark the number of that course ofaction as your answer. You are not to assume anything other than the informationprovided in each question. All these cases are given to you as on 01.06.2009.

Give answer:a) If the candidate is to be selected.b) If the candidate is not to be selected.c) If the information provided is inadequate to take a decision.d) If the case is to be referred to the Director-Personnel.e) If the case is to be referred to the President-Personnel.

11. Meena Srivastava was born on 06th March 1978. She has been working asDeputy Personnel Manager in an organization for the past four years aftercompleting her Post Graduate Diploma in HR with 68% marks. She hassecured 50% marks in both graduation and selection process.

Page 167: Reasoning Made Easy - pdfstores.files.wordpress.com · REASONING MADE EASY 6 Geetika, Jasmine, Hema, Brinda, Ipsita and veena, once again not following any order. The following are

REASONING MADE EASY www.BankExamsToday.com

167

12. Ketan Desai was born on 05th January 1979. He has been working for the pastfive years in the personnel department of an organisation after completing hisPost Graduate Diploma in Personnel Management with 64 % marks. He hassecured 40% marks in the selection process and 52% marks in Graduation.

13. Anant Joshi has been working in the Personnel Department of an organisationfor the past six years. He was born on 07th November 1977. He has secured60% marks in post Graduate Degree in Personnel Management. He has alsosecured 55% marks in both Graduation and selection process.

14. Mohan Bajpai was born on 10th April 1975. He has secured 55% marks inGraduation and 65% marks in Post Graduate Diploma in PersonnelManagement. He has been working in the HR Department of an organisationfor the past six years after completing his Post Graduate Diploma.

15. Gopal Sharma has been working for the past five years in the HR Department ofan organisation after completing his Post Graduate Diploma in HR with 62%marks. He has secured 50% marks in both Graduate and selection process. Hewas born on 29th May 1974.

16. Arun Vohra has secured 55% marks in Graduation. He has been working in thePersonnel Department of an organisation for the past eleven years aftercompleting his post Graduate Degree in Personnel Management with 65%marks. He has secured 50% marks in the selection process. He was born on 12th

August 1972.

17. Asha Dhar has secured 52% marks in Graduation and 62% marks in PostGraduate Degree in Personnel Management. She has also secured 48% marks inthe selection process. She has been working for the past seven years in thePersonnel Department of an organisation after completing her Post GraduateDegree. She was born on 08th June 1974.

18. Sudha Ghosal was born on 20th October 1976. She has been working as DeputyPersonnel Manager for the past four years in an organisation after completingher Post Graduate Degree in HR with 67% marks. She has secured 60% marksin Graduation and 45% marks in the selection process.

19. Amit Saxena was born on 25th July 1973. He has been working in the PersonnelDepartment of an organisation for the past eleven years after completing hisPost Graduate Diploma in HR with 70% marks. He has secured 60% ma rks inboth Graduation and selection process.

20. Navin Das was born on 14th April 1978. He has been working in the PersonnelDepartment of an organisation for the past six years after completing his Post

Page 168: Reasoning Made Easy - pdfstores.files.wordpress.com · REASONING MADE EASY 6 Geetika, Jasmine, Hema, Brinda, Ipsita and veena, once again not following any order. The following are

REASONING MADE EASY www.BankExamsToday.com

168

Graduate Diploma in HR with 65% marks. He has secured 45% marks in bothGraduation and selection process.

Directions (Q. 21-25) Study the following information carefully and answer thequestions given below

Following are the conditions for selecting Systems Manager in an organization:

The candidate must(i) Be a graduate engineer in IT, Computer Science, Electronics with at

least 60% marks(ii) Be at least 30 years and not more than 40 years as on 01.09.2009(iii) Have secured at least 40% marks in the written examination(iv) Have secured at least 50% marks in the selection interview(v) Have post qualification work experience of at least 10 years in the

Systems Department of an organization.

In the case of a candidate who satisfies all the conditions except(a) At (i) above, but secure at least 60% marks in MEIT or Computer Science,

the case as to be referred to DGM-Systems.(b) At (v) above, but has post qualification experince of at least five years as

Deputy Systems Manager the case is to be referred to the GM-Systems.

In each question below, details of one candidate are given. You have to take one ofthe following courses of actions based on the information provided and theconditions and sub-conditions given above and mark the number of that course ofaction as your answer. You are not to assume anything else other than theinformation provided for each candidate. All these cases are given to you as on01.09.2009.

Give answer:(a) If the candidate is to be selected(b) If the candidate is not to be selected(c) If the case is to be referred to DGM-Systems

(d) If the case is to be referred to GM-Systems(e) If the data provided are not adequate to take a decision.

21. Samir Ghosh was born on 25th May 1978. He has secured 65% marks in BE-ITin the year 1999. Since then he is working in the systems department of anorganization. He has secured 50% marks in both written examination andselection interview.

22. Navin Prakash has secured 62% marks in 65% marks in BE-Computer Science.He has been working in the systems department of an organization since July1999 after completion of BE. He was born on 4 April 1974. He has secured55% marks in selection interview and 45% marks in the written examination.

Page 169: Reasoning Made Easy - pdfstores.files.wordpress.com · REASONING MADE EASY 6 Geetika, Jasmine, Hema, Brinda, Ipsita and veena, once again not following any order. The following are

REASONING MADE EASY www.BankExamsToday.com

169

23. Neeta Pathak has been working as Deputy Systems Manager in an organizationfor the past 7 years after completing her B.E. in IT with 70% marks. She hassecured 45% marks in selection interview and 55% marks in the writtenexamination. She was born on 12th November 1978.

24. Ashok Malhotra was born on 19th March 1977. He has secured 56% marks I nboth selection interview and written examination. He has secured 58% marks inBE-IT and 72% marks in ME-IT. He has been working in the systemsdepartment of an organization for the past 11 years after copleting ME-IT.

25. Geema D’Souza was born on 15th December 1972. She has secured 60% marksin both written examination and selection interview. She has been working asDeputy Systems Manager for the last 6 years in an organization aftercompleting her BE-Electronics with 75% marks.

Directions (Q. 26-35) Study the following information carefully and answer thequestions given below.

Following are the conditions for selecting Accounts Manager in an organization.

The candidate must(i) Be a graduate in Commerce with minimum 55% marks(ii) Be a post graduate in Commerce with minimum 50 % marks(iii) Have post qualification work experience of at least three years in the

Accounts department in an organization.(iv) Not be less than 25 years and not more than 35 years old as on

01.05.2008.(v) Have secured at least 40% marks in the selection process.

In the case if candidate satisfies all other criteria except-A. At (ii) above but h as work experience of at least four years as Assistant

Accounts Manager in an organization. His/her case is to be referred to GM-Accounts of the organization.

B. At (iii) above but has successfully completed CA/ICWA, the case is to bereferred to Director Finance.

In each question below details of one candidate is given. You have to take oneof the following courses of action in each case based on the informationprovided and the conditions and sub-conditions given above and mark thenumber of that course of action as your answer. You are not to assumeanything other the information provided in each case. All t he cases are givento you as on 01.05.2008.

Give answer:a) If the data provided are not adequate to take a decision.

Page 170: Reasoning Made Easy - pdfstores.files.wordpress.com · REASONING MADE EASY 6 Geetika, Jasmine, Hema, Brinda, Ipsita and veena, once again not following any order. The following are

REASONING MADE EASY www.BankExamsToday.com

170

b) If the case is t o be referred to GM- Accounts.c) If the case is to be referred to Director-Finance.d) If the candidate is not to be selected.e) If the candidate is to be selected.

26. Abhinav Chaturvedi is a Commerce graduate with 60% marks. He has beenworking in the Accounts Department of an organization for the past four yearsafter completing his Post Graduation in Commerce with 55% marks. He wasborn on 08th July 1980. He has secured 45% marks in the selection process.

27. Vaibhav Sinha was born on 12th April 1979. He has been working as AssistantAccounts Manager in an organization for the past five years after completinghis post Graduation in Commerce with 40% marks. He has secured 55% marksin both the selection process and Graduation in Commerce.

28. Seema Bhasin has secured 60% marks in Graduation in Commerce and 55%marks in post Graduation in Commerce. She has secured 55% marks in theselection process. She was born on 20th August 1978. She has also successfullycompleted her CA. She does not have any work experience.

29. Nirmala Sawant was born on 04th July 1981. She has been working in theAccounts department in an organization after completing her post graduationCommerce with 65% marks. She has secured 50% marks in the selectionprocess.

30. Ashok Pradhan was born on 03rd May 1979. He has secured 42% marks in theselection process. He has been working in the Accounts Department of anorganization for the past four years after completing his post Graduation inCommerce with 62% marks. He has secured 54% marks in his Graduation inCommerce.

31. Prabir Mazumdar has secured 58% marks in his Graduation in Commerce and53% marks in post Graduation in Commerce. He has been working as AssistantAccounts Manager in an organization for the past four years after completinghis post Graduation. He has secured 44% marks in the selection process. Hewas born on 14th November 1977.

32. Neha Dev was born on 08th February 1976. She has been working in theAccounts Department of an organization for the past three years aftercompleting her post Graduation in Commerce with 53% marks. She has secured57% marks in Graduation in Commerce. She appeared in selection process.

33. Sudha Goswami was born on 19th October 1982. She has been working asAssistant Accounts Manager in an organization for the past five years after

Page 171: Reasoning Made Easy - pdfstores.files.wordpress.com · REASONING MADE EASY 6 Geetika, Jasmine, Hema, Brinda, Ipsita and veena, once again not following any order. The following are

REASONING MADE EASY www.BankExamsToday.com

171

completing her Graduation in Commerce with 60% marks. She has secured50% marks in the selection process.

34. Francis D’costa has secured 60% marks in both graduation and Post Graduationin Commerce. He has successfully completed ICWA after his Graduation. Hehas been working in the Account Department of an organization for last oneyear.

35. Prathama Sengupta has secured 55% marks in Post Graduation in Commerceand 45% marks in the selection process. She was born on 11th April 1981. Shehas been working in the Accounts Department of an organization for the pastfour years after completing her Post Graduation. She has secured 50% marks inGraduation in Commerce.

Answers:

1. Option C2. Option B3. Option B4. Option D5. Option E6. Option C

Suresh Mehta’s condition (iii) is not given so data is insufficient to takedecision.

7. Option DSudha Gopalan fulfils condition (b) instead of condition (ii) so, her case isto be referred to Vice President-Marketing.

8. Option ADivya Kohli does not fulfil condition (i) so, she is not to be selected.

9. Option BNavin Marathe fulfils all conditions so, he is to be selected.

10. Option EVarun Malhotra fulfils condition (a) instead of (iv) so, his case is to bereferred to GM-Marketing.

11. Option EMeena Srivastava fulfils conditions (b) instead of condition (iv) so, her caseis to be referred to the President-Personnel.

12. Option BKetan Desai does not fulfil condition (v) so, he is not to be selected.

Page 172: Reasoning Made Easy - pdfstores.files.wordpress.com · REASONING MADE EASY 6 Geetika, Jasmine, Hema, Brinda, Ipsita and veena, once again not following any order. The following are

REASONING MADE EASY www.BankExamsToday.com

172

13. Option CPost qualification work of Anant Joshi is not cleared (which is least 5years) so, data is insufficient.

14. Option CMarks of selection process (i.e. 45%) is not cleared so, data is Insufficient.

15. Option BGopal Sharma is overaged.

16. Option DArun Vohra fulfils condition (a) instead of (iii) so, his case is to be referredto the Director-Personnel.

17. Option AAsha Dhar fulfils all conditions so, she is to be selected.

18. Option ESudha Ghosal fulfils condition (b) instead of (iv) so, her case is to bereferred to President-Personnel.

19. Option DAmit Saxena fulfils condition (a) instead of (iii) so, his case is to be referredto the Director-Personnel.

20. Option BNavin Das does not fulfil condition (i) so, he is not to be selected.

21. Option ESameer’s working experience is not cleared in years. So, data isinsufficient.

22. Naveen Prakash fulfils all conditions so, he is to be selected.

23. Option BNeeta Pathak does not fulfil condition (iv) so, she is not to be selected.

24. Optoin CAshok Malhotra fulfils condition (a) so, his case is to be referred to DGM-systems.

25. Option DGeema D’Souza fulfils condition (b) so, her case is to be referred to GM-systems.

Page 173: Reasoning Made Easy - pdfstores.files.wordpress.com · REASONING MADE EASY 6 Geetika, Jasmine, Hema, Brinda, Ipsita and veena, once again not following any order. The following are

REASONING MADE EASY www.BankExamsToday.com

173

26. Option E27. Option B28. Option C29. Option A30. Option D31. Option E32. Option D33. Option B34. Option C35. Option D

Page 174: Reasoning Made Easy - pdfstores.files.wordpress.com · REASONING MADE EASY 6 Geetika, Jasmine, Hema, Brinda, Ipsita and veena, once again not following any order. The following are

REASONING MADE EASY www.BankExamsToday.com

174

Chapter – 11

Direction Sense

1. P walked 20 m towards North, took a left turn and walked 10 m, then took aright turn and walked 20 m, again took a right turn and walked 10 m. Howfar is he from his starting point?a) 50 m b) 60 m c) 40 md) Can’t be determined e) None of these

2. Kunal walked 10 m, towards the East, turned right and walked another 15m. He then turns left and walks 3 m. He finally takes a left turn afterwalking 6 m. Which direction is he facing now?a) East b) North c) Westd) South e) South-West

3. Town D is to the West of town M. Town R is to the South of town D. TownK is to the East of town R. Town K is towards which direction of town D?a) South b) East c) North-Eastd) South-East e) None of these

4. Ram walks 12 kms to the North, then 10 kms. To West, 12 kms. to South.How far is Ram from the starting point?a) 9 kms. b) 13 kms. c) 8 kms.d) 10 kms. e) None of these

5. One morning Udai and Vishal were talking to each other face to face at acrossing. If Vishal’s shadow was exactly to the left of Udai, which directionwas Udai facing?a) East b) West c) Northd) South e) None of these

6. Two cars start from the opposite places of a main road, 150 km apart. Firstcar runs for 25 km and takes a right turn and then runs 15 km. It then turnsleft and then runs for another 25 km and then takes the direction back toreach the main road. In the mean time, due to minor break down the othercar has run only 35 km along the main road. What would be the distancebetween two cars at this point?a) 65 km b) 75 km c) 80 kmd) 85 km e) None of these

7. The length and breadth of a room are 8 m and 6 m respectively. A cat runsalong all the four walls and finally along a diagonal order to catch a rat.How much total distance is covered by the cat?a) 10 b) 14 c) 38

Page 175: Reasoning Made Easy - pdfstores.files.wordpress.com · REASONING MADE EASY 6 Geetika, Jasmine, Hema, Brinda, Ipsita and veena, once again not following any order. The following are

REASONING MADE EASY www.BankExamsToday.com

175

d) 48 e) None of these

8. X started to walk straight towards south. After walking 5 m he turned to theleft and walked 3 m. After this he turned to right and walked 5 m now towhich direction X is facing?a) North-East b) South c) Northd) South-West e) None of these

9. If A × B means A is to the South of B, A + B means is to the North of B, A% B means A is to the east of B, A ⎯B means A is to the West of B, then inP % Q + R ⎯S, S is in which direction with respect to Q?a) South-West b) South-East c) North-Eastd) North-West e) None of these

10. One morning after sunrise, Suresh was standing facing a pole. The shadowof the pole fell exactly to his right. To which direction was he facing?a) East b) South c) Westd) Data is inadequate e) None of these

11. Rohit walked 25 m towards South. Then he turned to his left and walked 20m. He then turned to his left and walked 25 m. He again turned to his rightand walked 15 m. At what distance is he from the starting point and inwhich direction?a) 35 m East b) 35 m North c) 30 m Westd) 45 m East e) None of these

12. Sachin walks 20 km towards North. He turns left and walks 40 km. Heagain turns left and walks 20 km. Finally he moves 20 km after turning tothe left. How far is he from his starting position?a) 20 km b) 30 km c) 50 kmd) 60 km e) None of these

Dirrections (Q. 13-16) Each of the following is based on the followinginformation:

(i) Six flats on a floor in two rows facing North and South are allotted to P, Q,R, S, T and U.

(ii) Q gets a North facing flat and is not next to S.(iii) S and U get diagonally opposite flats.(iv) R next to U gets a South facing flat and T gets North facing flat.

13. If the flats of P and T are interchanged then whose flat will be next to thatof U?a) P b) Q c) Rd) T e) None of these

Page 176: Reasoning Made Easy - pdfstores.files.wordpress.com · REASONING MADE EASY 6 Geetika, Jasmine, Hema, Brinda, Ipsita and veena, once again not following any order. The following are

REASONING MADE EASY www.BankExamsToday.com

176

14. Which of the following combination get South facing flats?a) QTS b) UPT c) URPd) Data is inadequate e) None of these

15. The flats of which of the other pair than SU, is diagonally opposite to eachother?a) QP b) QR c) PTd) TS e) None of these

16. Whose flat is between Q and S?a) T b) U c) Rd) P e) None of these

17. Starting from a point, Raju walked 12 m North, he turned right and walked10 km, he again turned right and walked 12 m, then he turned left andwalked 5 m. How far is he now and in which direction from the startingpoint?a) 27 m towards Eastb) 5 m towards Eastc) 10 m towards Westd) 15 m towards Easte) None of these

18. Ravi travelled 4 km straight towards South. He turned left and travelled 6km straight, then turned right and travelled 4 km straiht. How far is he fromthe starting point?a) 8 km b) 10 km c) 12 kmd) 18 km e) None of these

19. From point P, Akshay starts walking towards East. After walking 30 m, heturns to his right and walks 10 m. He then turns to his right and walks for30 m. He again turns to his right and walks 30 m. How far is he from pointP and in which direction?a) Point P itself b) 10 m, North c) 20 m,Westd) 20 m, North e) None of these

20. Facing towards South, Ram started walking and turned left after walking 30m, he walked 25 m and turned left and walked 30 m. How far is he from hisstarting position and in which direction?a) At the starting point onlyb) 25 m, Westc) 25 m, Eastd) 30 m, Easte) None of these

Page 177: Reasoning Made Easy - pdfstores.files.wordpress.com · REASONING MADE EASY 6 Geetika, Jasmine, Hema, Brinda, Ipsita and veena, once again not following any order. The following are

REASONING MADE EASY www.BankExamsToday.com

177

21. The town of Paranda is located on Gree lake. The town of Akram is West ofParanda. Tokhada is East of Akram but West of Paranda. Kokran is East ofBopri but West of Tokhada and Akram. If they are all in the same district,which town is the farthest West?a) Paranda b) Kokran c) Akramd) Tokhada e) Bopri

22. B is to the South-West of A, C is to the East of B and South-East of A andD is to the North of C in line with B and A. In which direction of A is Dlocated?a) North b) East c) South-Eastd) North-East e) None of these

23. A man is facing West. He turns 45° in the clockwise direction and thenanother 180° in the same direction and then 270° in the anticlockwisedirection. Find which direction he is facing now?a) South-West b) West c) Southd) East-South e) None of these

24. A dog runs 20 metre towards East and turns Right, runs 10 metre and turnsto right, runs 9 metre and again turns to left, runs 5 metre and then turns toleft, runs 12 metre and finally turns to left and runs 6 metre. Now whichdirection dog is facing?a) East b) North c) Westd) South e) None of these

25. Rahul put his timepiece on the table in such a way that at 06:00 p.m. hourhand points to North. In which direction the minute hand will point at 09:15p.m.?a) South-East b) South c) Northd) West e) None of these

26. K is 40 m South-West of L. If M is 40 m South-East of L, then M is inwhich direction of K?a) East b) West c) North-Eastd) South e) None of these

27. A child went 90 m in the East to look for his father, then he turned right andwent 20 m. After this he turned right and after going 30 m he reached to hisuncle’s house. His father was not there. From there he went 100 m to hisNorth and met his father. How far did he meet his father from the startingpoint?a) 80 m b) 100 m c) 140 md) 260 m e) None of these

Page 178: Reasoning Made Easy - pdfstores.files.wordpress.com · REASONING MADE EASY 6 Geetika, Jasmine, Hema, Brinda, Ipsita and veena, once again not following any order. The following are

REASONING MADE EASY www.BankExamsToday.com

178

28. Ravi wants to go to the University. He starts from his home which is in theEast and comes to a crossing. The road to the left ends in a theatre, straightahead is the hospital. In which direction is the University?a) North b) South c) Eastd) West e) None of these

29. A river flows West to East and on the way turns left and goes in a semi-circle round a hillock, and then turns left at right angles. In which directionin the river finally flowing?a) West b) East c) Northd) South e) None of these

30. You go North, turn right, then right again and then go the left. In whichdirection are you now?a) North b) South c) Eastd) West e) None of these

Answers:

1. Option C2. Option B3. Option D4. Option D5. Option C6. Option A

Required distance = DF= 150 ⎯(25 + 25 + 35)= 150 ⎯85 = 65 km

7. Option CRequired distance = 8 + 6 + 8 + 6 + √8 + 6

= 28 + √100= 28 + 10 = 38 m

8. Option BX will face in the end towards South.

9. Option BS is in the South-East of Q.

10. Option BSun rises in the East in the morning. Since the shadow of Suresh falls to hisright. So, he is facing South.

Page 179: Reasoning Made Easy - pdfstores.files.wordpress.com · REASONING MADE EASY 6 Geetika, Jasmine, Hema, Brinda, Ipsita and veena, once again not following any order. The following are

REASONING MADE EASY www.BankExamsToday.com

179

11. Option ARequired distance = AE

= 20 + 15 = 35 m towards East

12. Option ARequired distance = 40 ⎯20 = 20 km

13. Option CFlat R will be next to U

14. Option CURP flat combination get South facing flats.

15. Option AQP is diagonally opposite to each other.

16. Flat T is between Q and S.

17. Option D

18. Option B

19. Option D

20. Option C

21. Option E

22. Option D

23. Option AThe man firstly faces the direction OA. On moving 45° clockwise, he facesthe direction OB.Now, again he moved 180° clockwise, now, he will be facing OC. Fromhere he moved 270° anticlockwise, Finally he is facing OD, which is South-West.

24. Option B

25. Option D

Page 180: Reasoning Made Easy - pdfstores.files.wordpress.com · REASONING MADE EASY 6 Geetika, Jasmine, Hema, Brinda, Ipsita and veena, once again not following any order. The following are

REASONING MADE EASY www.BankExamsToday.com

180

At 09:15 p.m., the minute hand will point towards West.

26. Option A

M is in the East of K.

27. Option B

28. Option A

Page 181: Reasoning Made Easy - pdfstores.files.wordpress.com · REASONING MADE EASY 6 Geetika, Jasmine, Hema, Brinda, Ipsita and veena, once again not following any order. The following are

REASONING MADE EASY www.BankExamsToday.com

181

Starting from his house in the East, Ravi moves westwards. Then thetheatre, which is to the left, will be in the South. The hospital, which isstraight ahead, will be to the West. So, the University will be to the North.

29. Option B

30. Option C

Page 182: Reasoning Made Easy - pdfstores.files.wordpress.com · REASONING MADE EASY 6 Geetika, Jasmine, Hema, Brinda, Ipsita and veena, once again not following any order. The following are

REASONING MADE EASY www.BankExamsToday.com

182

Chapter – 12

Letter and Number Series

1. If in the word ISOLATE, all the consonants are replaced by the previousletter in the alphabet and all the vowels are replaced by the next letter thenall the letters are arranged alphabetically, which letter will be third from theright end?a) P b) B c) Nd) Q e) None of these

2. If in the word EQUALITY, the positions of first and the fifth letters areinterchanged; similarly the positions of the second and the sixth letters areinterchanged and so on. Which letter will be third from the right end?a) Q b) U c) Id) T e) None of these

3. How many meaningful English words can be made from the letters AERT,using each letter only once in each word?a) None b) One c) Twod) Three e) Four

4. How many such pairs of letters are there in the word REFRESHING eachof which has as many letters between them in the word as they have in theEnglish alphabet?a) None b) One c) Twod) Three e) More than three

5. If in the number 38564927 first all the even digits are arranged in ascendingorder and then all the odd digits are arranged in ascending order, whichdigit will be fourth from the right end?a) 5 b) 3 c) 6d) 4 e) None of these

6. How many such pairs of letters are there in the word BOARDING each ofwhich has as many letters between them in the word as in the Englishalphabet?a) None b) One c) Twod) Three e) More than three

7. How many such digits are there in the number 284371 each of which is asfar away from the beginning of the number as when they are arranged indescending order?a) None b) One c) Twod) Three e) More than three

Page 183: Reasoning Made Easy - pdfstores.files.wordpress.com · REASONING MADE EASY 6 Geetika, Jasmine, Hema, Brinda, Ipsita and veena, once again not following any order. The following are

REASONING MADE EASY www.BankExamsToday.com

183

8. How many meaningful English words can be made with the letters onlyonce in each word?a) None b) One c) Twod) Three e) More than three

9. How many meaningful English words can be made with the letters ONDEusing each letter only once in each word?a) None b) One c) Twod) Three e) More than three

10. How many such pairs of letters are there in the word JUMPING each ofwhich has as many letters between them in the word as in the Englishalphabet?a) None b) One c) Twod) Three e) More than three

11. How many such digits are there in the number 6837941 each of which is asfar away from the beginning in the number as when the digits are arrangedin descending order within the number?a) None b) One c) Twod) Three e) More than three

12. What should come next in the following letter series?P P O P O N P O N M P O N M L P O N M La) P b) K c) Jd) I e) None of these

13. How many such pairs of letters are there in the word STAPLER each ofwhich has as m any letters between them in the word as in the Englishalphabet?a) None b) One c) Twod) Three e) More than three

14. How many such pairs of letters are there in the word OBJECTIVE each ofwhich has as many letters between them in the word as in the Englishalphabet?a) None b) One c) Twod) Three e) More than three

15. If each consonant in the word TOLERANT is replaced by the previousletter in the English alphabet and each vowel in the word is replaced by thenext letter in the English alphabet and the new set of letters are arrangedalphabetically, which of the following will be the 4th from the right endafter the replacement?a) M b) P c) Q

Page 184: Reasoning Made Easy - pdfstores.files.wordpress.com · REASONING MADE EASY 6 Geetika, Jasmine, Hema, Brinda, Ipsita and veena, once again not following any order. The following are

REASONING MADE EASY www.BankExamsToday.com

184

d) K e) None of these

16. How many such pairs of digits are there in the number 8429516 each ofwhich has as many digits between them in the number as when the digitsare arranged in ascending order within the number?a) None b) One c) Twod) Three e) More than three

17. The positions of the first and the second digits in the number 85341279 areinterchanged. Similarly the positions of 3rd and 4th digits are interchangedand so on till the position of 7th and 8th digits. Which of the following willbe 3rd to the right of three after the rearrangement?a) 9 b) 7 c) 8d) 2 e) None of these

18. How many meaningful English words can be formed with the letters MRTAusing each letter only once in each word?a) None b) One c) Twod) Three e) More than three

19. The positions of the first and the fifth digit in the number 53216894 areinterchanged. Similarly, the position of the second and the sixth digit are interchanged and so on. Which of the following will be the second from t heright end after the rearrangement?a) 3 b) 2 c) 1d) 9 e) None of these

20. The positions of how many digits in the number 53147926 will remainunchanged after the digits within the number are rearranged in descendingorder?a) None b) One c) Twod) Three e) More than three

21. How many such pairs of letters are there in the word HOARDINGS each ofwhich has as many letters between them in the word as in the Englishalphabet?a) None b) One c) Twod) Three e) More than three

22. How many such pairs of letters are there in the word DISCREDIT each ofwhich has as many letters between them in the word as in the Englishalphabet?a) None b) One c) Twod) Three e) More than three

Page 185: Reasoning Made Easy - pdfstores.files.wordpress.com · REASONING MADE EASY 6 Geetika, Jasmine, Hema, Brinda, Ipsita and veena, once again not following any order. The following are

REASONING MADE EASY www.BankExamsToday.com

185

23. If it is possible to make only one meaningful English word with 3rd, 4th, 6th

and 8th letters of the word SINGLETON, which of the following will be 3rd

letter of that word? If no such word can be made, g ive ‘X’ as the answerand I f more than one such word can be made, give ‘Y’ as the answer.a) N b) O c) Ed) X e) Y

24. What should come next in the following number series?9 8 7 6 5 4 3 2 1 1 2 3 4 5 6 7 8 9 9 8 7 6 5 4 3 2 2 3 4 5 6 7 8

a) 8 b) 1 c) 9d) 2 e) None of these

25. How many such digits are there in the number 764528 each of which is asfar away from the beginning of the number as when the digits are arrangedin descending order within the number?a) None b) One c) Twod) Three e) More than three

26. What should come next in the letter series given below?ABABCABCDABCDEABCDEFABCDEFGABCa) D b) E c) Fd) H e) None of these

27. How many meaningful English words can be made with the letters EPRYusing each letter only once in each word?a) None b) One c) Twod) Three e) More than three

Directions (Q. 28-34) In each of the following questions, a series is given withone term missing. Choose the correct alternative from the given ones that willcomplete the series.

28. AD, EI, JN, PS, ?a) WY b) XX c) WXd) WW e) None of these

29. PON, RQP, TSR, VUT, ?a) WUY b) YXZ c) XWVd) UVW e) None of these

30. ar, cs, et, ?a) wy b) gv c) vbd) gu e) None of these

31. 9, 15, 23, 33, ?a) 44 b) 36 c) 38

Page 186: Reasoning Made Easy - pdfstores.files.wordpress.com · REASONING MADE EASY 6 Geetika, Jasmine, Hema, Brinda, Ipsita and veena, once again not following any order. The following are

REASONING MADE EASY www.BankExamsToday.com

186

d) 45 e) None of these

32. 4, 7, 14, 24, 41, ?a) 71 b) 68 c) 72d) 51 e) None of these

33. 5, 16, 51, 158, ?a) 481 b) 465 c) 441d) 478 e) None of these

34. CBA, ABC, ABCD, DCBA, ABCDE, ?a) EDCBA b) DBAC c) CABDd) BACD e) None of these

35. How many meaningful English words can be formed with the letters LTSOusing each letter only once in each word?a) None b) One c) Twod) Three e) More than three

36. How many such digits are there in the number 928416375 each of which isas far away from th e beginning in the number as when the digits arearranged in descending order within the number?a) None b) One c) Twod) Three e) More than three

37. What should come next in the following letter series?C b a a c b a a b c b a a b c c b a a b c d c b a aa) a b) b c) cd) d e) None of these

Directions (Q. 38-43) Study the number series given below and answer thequestions which follow.

6 7 8 9 9 8 7 9 7 7 8 9 7 8 7 6 9 6 8 9 7 7 9 8 9 7 6 8 8 7

38. How many such numbers are there in the given series each of which whensubtracted from the following number has a difference of 2?a) Three b) Four c) Fived) Nine e) None of these

39. Which of the following nu mbers is obtained when 18th number from theleft of the number series is added to 19th from the right?a) 15 b) 20 c) 10d) 17 e) None of these

Page 187: Reasoning Made Easy - pdfstores.files.wordpress.com · REASONING MADE EASY 6 Geetika, Jasmine, Hema, Brinda, Ipsita and veena, once again not following any order. The following are

REASONING MADE EASY www.BankExamsToday.com

187

40. How many nines are there in the given series each of which is immediatelypreceded by an odd number?a) One b) Two c) Threed) Four e) More than four

41. How many such even numbers are there in the given series each of which isimmediately followed by an even number?a) None b) One c) Twod) Three e) More than three

42. How many such odd numbers are there in the given series each of which isimmediately preceded by an even number?a) Five b) Seven c) Nined) Eleven e) None of these

43. How many such sevens are there in the given series each of which isimmediately preceded by 9 and also immediately followed by 8?a) None b) One c) Twod) Three e) More than three

Answers:

1. Option A2. Option A3. Option C4. Option D5. Option B6. OptionC7. Option D8. Option C9. Option C

DONE and NODE10. Option C

J U M P I N G10 21 13 16 9 14 7

11. Option CNumber 6 8 3 7 9 4 1Decreasing Order 9 8 7 6 4 3 1There will be no change in the place of 8 and 1.

12. Option BSeries are increasing as follows:P PO PON PONM PONML PONMLK

Page 188: Reasoning Made Easy - pdfstores.files.wordpress.com · REASONING MADE EASY 6 Geetika, Jasmine, Hema, Brinda, Ipsita and veena, once again not following any order. The following are

REASONING MADE EASY www.BankExamsToday.com

188

13. Option CS T A P L E RS ⎯T and S ⎯P

14. Option CO B J E C T I V ETwo pairs formed – IO, TV

15. Option BT O L E R A N TAccording to question,S P K F Q B M SAlphabetically,B F K M P Q S SSo, P is 4th from the right.

16. Option C8 4 2 9 5 1 6In increasing order1 2 4 5 6 8 9

17. Option ANumber: 8 5 3 4 1 2 79Arrangement: 5 8 4 3 2 1 9 7

18. Option CThe meaningful words are: MART, TRAM

19. Option BNumber: 5 3 2 1 6 8 9 4After interchanging the digits6 8 9 4 5 3 2

20. Option BNumber: 5 3 1 4 7 92 6In decreasing order: 9 7 6 5 4

21. Option E22. Option E

D I S C R E D I TSo, CE, DI, DE, EI are pairs of letters.

23. Option A

1 2 3 4 5 6 7 8 9S I N G L E T O N

Page 189: Reasoning Made Easy - pdfstores.files.wordpress.com · REASONING MADE EASY 6 Geetika, Jasmine, Hema, Brinda, Ipsita and veena, once again not following any order. The following are

REASONING MADE EASY www.BankExamsToday.com

189

Meaningful word from NGEO is GONE.3rd letter in the word GONE is N

24. Option C987654321, 123456789, 98765432, 2345678

25. Option B

7 6 4 5 2 88 7 6 5 4 2

26. Option A

AB, ABC, ADCD, ABCDE, ABCDEF, ABCDEFG, ABC D

27. Option C

Meaningful words PREY, PYRE28. Option C29. Option C30. Option D31. Option D

9 + 6 = 1515 + 8 = 2323 + 10 = 3333 + 12 = 45

32. Option B4 + 3 = 77 + 4 + 3 = 1414 + 3 + 7 = 2424 + 7 + 10 = 4141 + 10 + 17 = 68

33. Option A5 × 3 + 1 = 1616 × 3 + 3 = 5151 × 3 + 5 = 158158 × 3 + 7 = 481

34. Option ACBA ⟶ ABCReverse orderABCD ⟶ DCBAABCDE ⟶ EDCBA

35. Option D36. Option C37. Option B38. Option E

Page 190: Reasoning Made Easy - pdfstores.files.wordpress.com · REASONING MADE EASY 6 Geetika, Jasmine, Hema, Brinda, Ipsita and veena, once again not following any order. The following are

REASONING MADE EASY www.BankExamsToday.com

190

39. Option A40. Option C41. Option D42. Option C43. Option B

Page 191: Reasoning Made Easy - pdfstores.files.wordpress.com · REASONING MADE EASY 6 Geetika, Jasmine, Hema, Brinda, Ipsita and veena, once again not following any order. The following are

REASONING MADE EASY www.BankExamsToday.com

191

Chapter – 13

Logical Arrangement

1. Which one of the given responses would be a meaningful order of thefollowing?

(i) Earth(ii) Jupiter(iii) Venus(iv) Mars(v) Mercury

a) v, iii, i, ii, iv b) v, iii, iv, i, ii c) v, iii, i, iv, iid) v, iii, ii, iv, i e) None of these

Arrange the words given below in a meaningful sequence.

2. (i) Key (ii) Door (iii) Lock (iv) Room (v) Switch ona) v, i, ii, iv, iii b) iv, ii, i, v, iii c) i, ii, iii, v, ivd) i, iii, ii, iv, v e) None of these

3. (i) Windows (ii) Walls (iii) Floor (iv) Foundation (v) Roof (vi) Rooma) iv, v, iii, ii, i, vib) iv, iii, v, vi, ii, ic) iv, ii, i, v, iii, vid) iv, i, v, vi, ii, iiie) None of these

4. (i) District (ii) Village (iii) State (iv) Blocka) ii, i, iv, iii b) ii, iii, iv, i c) ii, iv, i, iii

d) iii, ii, i, iv e) None of these

5. (i) Site (ii) Plan (iii) Rent (iv) Money (v) Buildinga) iv, i, ii, v, iii b) iii, iv, ii, v, i c) ii, iii, v, i, iv

d) i, ii, iii, v, iv e) None of these

6. (i) Cut (ii) Put on (iii) Mark (iv) Measure (v) Tailora) iv, iii, i, v, ii b) iii, i, v, iv, ii c) ii, iv, iii, i, v

d) i, iii, ii, iv, v e) None of these

7. (i) Mother (ii) Child (iii) Milk (iv) Cry (v) Smilea) i, v, ii, iv, iii b) ii, iv, i, iii, v c) ii, iv, iii, i, vd) iii, ii, i, v, iv e) None of these

8. (i) Word (ii) Paragraph (iii) Sentence (iv) Letters (v) Phrasea) iv, i, v, ii, iii b) iv, i, iii, v, ii c) iv, ii, v, i, iii

Page 192: Reasoning Made Easy - pdfstores.files.wordpress.com · REASONING MADE EASY 6 Geetika, Jasmine, Hema, Brinda, Ipsita and veena, once again not following any order. The following are

REASONING MADE EASY www.BankExamsToday.com

192

d) iv, i, v, iii, ii e) None of these

9. (i) Caste (ii) Family (iii) Newly married couple (iv) Clan (v) speciesa) ii, iii, i, iv, v b) iii, iv, v, i, ii c) iii, ii, i, iv, vd) iv, v, iii, ii, i e) None of these

10. (i) Elephant (ii) Cat (iii) Mosquito (iv) Tiger (v) Whalea) v, iii, i, ii, iv b) iii, ii, iv, i, v c) i, iii, v, iv, iid) ii, v, i, iv, iii e) None of these

11. (i) Protect (ii) Pressure (iii) Relief (iv) Rain (v) Flooda) ii, iv, iii, i, v b) ii, iv, v, i, iii c) ii, v, iv, i, iiid) iii, ii, iv, v, i e) None of these

12. (i) Rainbow (ii) Rain (iii) Sun (iv) Happy (v) Childa) iv, ii, iii, v, i b) ii, iii, i, v, iv c) iv, v, i, ii, iiid) ii, i, iv, v, iii e) None of these

13. (i) Patients (ii) Diagnosis (iii) Bill (iv) Doctor (v) Treatmenta) i, iv, iii, ii, v b) i, iv, ii, v, iii c) i, iv, ii, iii, vd) i, ii, iii, iv, v e) None of these

14. (i) Table (ii) Tree (iii) Wood (iv) Seed (v) Planta) i, ii, iii, iv, v b) iv, v, iii, ii, i c) I, iii, ii, iv, vd) iv, v, ii, iii, i e) None of these

15. (i) Doctor (ii) Fever (iii) Prescribe (iv) Diagnose (v) Medicinea) ii, iv, iii, v, i b) i, iv, iii, ii, v c) ii, i, iii, iv, vd) ii, i, iv, iii, v e) None of these

16. (i) Index (ii) Contents (iii) Title (iv) Chapters (v) Introductiona) iii, ii, v, i, iv b) ii, iii, iv, v, i c) iii, ii, v, iv, id) v, I, iv, ii, iii e) None of these

17. (i) Book (ii) Pulp (iii) Timber (iv) Jungle (v) Papea) iii, ii, v, i, iv b) iv, iii, ii, v, i c) ii, v, i, iv, iiid) v, iv, iii, i, ii e) None of these

18. (i) College (ii) Child (iii) Salary (iv) School (v) Employmenta) iv, i, iii, v, ii b) i, ii, iv, iii, v c) ii, iv, i, v, iiid) v, iii, ii, i, iv e) None of these

19. (i) Study (ii) Job (iii) Examination (iv) Earn (v) Applya) i, iii, v, ii, iv b) i, iii, ii, v, iv c) i, ii, iii, iv, vd) i, iii, v, iv, ii e) None of these

Page 193: Reasoning Made Easy - pdfstores.files.wordpress.com · REASONING MADE EASY 6 Geetika, Jasmine, Hema, Brinda, Ipsita and veena, once again not following any order. The following are

REASONING MADE EASY www.BankExamsToday.com

193

20. (i) Frog (ii) Eagle (iii) Grasshopper (iv) Snake (v) Grassa) v, iii, i, iv, ii b) i, iii, v, ii, iv c) v, iii, iv, ii, id) iii, iv, ii, v, i e) None of these

Answers:

1. Option B2. Option D3. Option C4. Option C5. Option D6. Option A7. Option C8. Option D9. Option C10. Option B11. Option B12. Option B13. Option B14. Option D15. Option D16. Option C17. Option B18. Option C19. Option A20. Option A

Page 194: Reasoning Made Easy - pdfstores.files.wordpress.com · REASONING MADE EASY 6 Geetika, Jasmine, Hema, Brinda, Ipsita and veena, once again not following any order. The following are

REASONING MADE EASY www.BankExamsToday.com

194

Chapter – 14

Mathematical Operations

1. If ‘Q’ means ‘+’, ‘T’ means ‘⎯’, ‘R’ means ‘÷’ and ‘K’ means ‘×’ then2 4 R 4 Q 8 K 6 T 10 = ?a) 48 b) 24 c) 44d) 2/3 e) None of these

2. If ‘÷’ denotes ‘⎯’, ‘×’ denotes ‘+’, ‘⎯’ denotes ‘×’ and ‘+’ denotes ‘÷’ then28 × 12 + 4 ÷ 6 ⎯4 = ?a) 4 b) 1 c) 6d) 5 e) None of these

3. If ‘+’ means ‘÷’, ‘⎯’ means ‘×’, ‘÷’ means ‘⎯’ and ‘×’ means ‘+’ then 42÷ 24 + 6 × 4 ⎯3 = ?a) 22-3/4 b) 50 c) 58d) 26 e) None of these

4. Which of the following interchange of signs would make the given equationcorrect?(6 + 3) + (4 × 7) = 29a) + and ⎯ b) ÷ and + c) × and +d) ÷ and × e) None of these

5. Change the sign to find the equation 28 ⎯7 + 2 × 2 = 0a) Change + into ×b) Change × into +c) Change ⎯into +

d) Change + into ⎯e) None of these

6. 9 ÷ 5 ÷ 4 ⎯3 × 2 = ?a) 2 b) ⎯9 c) ⎯3d) 3 e) None of these

7. If + means ÷ , × means ⎯, ÷ means × and ⎯means +, then 8 + 6 × 4 ÷ 3 ⎯4= ?a) ⎯12 b) ⎯20/3 c) 12d) 20/3 e) None of these

8. If × means ÷ , ⎯means ×, ÷ means + and + means ⎯, then 3 ⎯15 ÷ 19 × 8 +6 = ?a) 8 b) 4 c) 2d) ⎯1 e) None of these

Page 195: Reasoning Made Easy - pdfstores.files.wordpress.com · REASONING MADE EASY 6 Geetika, Jasmine, Hema, Brinda, Ipsita and veena, once again not following any order. The following are

REASONING MADE EASY www.BankExamsToday.com

195

9. When 1100010 is divided by 0101, what will be the decimal remainder?a) 2 b) 3 c) 4d) 6 e) None of these

10. How many basic binary subtraction combinations are possible?a) 4 b) 3 c) 2d) 1 e) None of these

11. When multiplying in binary the decimal values 13 × 11, what is the thirdpartial product?a) 100000 b) 100001 c) 0000d) 1011 e) None of these

12. If × stands for ‘addition’, ÷ stands for ‘subtraction’, + stands for‘multiplication’ and ⎯stands for ‘division’, then 20 × 8 ÷ 8 ⎯4 + 2 = ?a) 80 b) 25 c) 24d) 5 e) None of these

Answers:

1. Option C2. Option E

28 × 12 + 4 ÷ 6 ⎯4 = ?? = 28 + 12 ÷ 4 ⎯6 × 4? = 28 + 12/4 ⎯6 × 4? = 28 + 3 ⎯24? = 7

3. Option B? = 42 ÷ 24 + 6 × 4 ⎯3= 42 ⎯24 ÷ 6 + 4 × 3= 42 ⎯24/6 + 12= 42 ⎯4 + 12 = 50

4. Option C(6 + 3) + (4 × 7) = 29(6 × 3) + (4 + 7) = 2918 + 11 = 29

5. Option A28 ⎯7 + 2 × 2 = 028 ⎯7 × 2 × 2 = 028 ⎯28 = 0

6. Option E9 ⎯5 × 4 ÷ 3 ÷ 2 = 9 ⎯5 × 4 ÷ 3/2 = ⎯19/2

Page 196: Reasoning Made Easy - pdfstores.files.wordpress.com · REASONING MADE EASY 6 Geetika, Jasmine, Hema, Brinda, Ipsita and veena, once again not following any order. The following are

REASONING MADE EASY www.BankExamsToday.com

196

7. Option B

8. Option C

9. Option B

10. Option A

11. Option A

12. Option C20 + 8 ⎯8 ÷ 4 × 2 = 20 + 8 ⎯2 × 2 = 20 + 8 ⎯4 = 24

Page 197: Reasoning Made Easy - pdfstores.files.wordpress.com · REASONING MADE EASY 6 Geetika, Jasmine, Hema, Brinda, Ipsita and veena, once again not following any order. The following are

REASONING MADE EASY www.BankExamsToday.com

197

Chapter – 15

Odd Man Out Series

1. Four of the following five are alike in a certain way and so form a group.Which is the one that does not belong to the group?a) PS b) FI c) ADd) KN e) GD

2. Four of the following five are alike in a certain way and so form a group.Which is the one that does not belong to the group?a) 23 b) 29 c) 37d) 39 e) 31

3. Four of the following five are alike in a certain way and so form a group.Which is the one that does not belong to the group?a) Yellow b) Blue c) Pinkd) Green e) Red

4. Four of the following are alike in a certain way and so form a group. Whichis the one that does not belong to that group?a) Table b) Chair c) Benchd) Desk e) Wood

5. Four of the following are alike in a certain way and so form a group. Whichis the one that d oes not belong to that group?a) 27 b) 64 c) 125d) 216 e) 384

6. Four of the following five are alike in a certain way and so form a group.Which is the one that does not belong to that group?a) 35 b) 80 c) 45d) 60 e) 75

7. Four of the following five are alike in a certain way and so form a group.Which is the o ne that does not belong to that group?a) Sweet b) Cake c) Pastryd) Bread e) Biscuit

8. Four of the following five are alike in a certain way and so form a group.Which is the one that does not belong to that group?a) 39 b) 27 c) 48d) 42 e) 24

Page 198: Reasoning Made Easy - pdfstores.files.wordpress.com · REASONING MADE EASY 6 Geetika, Jasmine, Hema, Brinda, Ipsita and veena, once again not following any order. The following are

REASONING MADE EASY www.BankExamsToday.com

198

9. Four of the following five are a like in a certain way and so form a group.Which is the one that does not belong to that group?a) VT b) MK c) DBd) KH e) XV

10. Four of the following five are alike in a certain way and so form a group.Which is the one that does not belong to that group?a) 187 b) 323 c) 119d) 221 e) 289

Directions (Q. 11-19) In each of the following questions, select the one which isdifferent from the other three responses.

11. a) Aptitude b) Altitude c) Attituded) Behaviour e) None of these

12. a) SSA b) RMSA c) RUSAd) NASA e) None of these

13. a) 63 b) 69 c) 65d) 66 e) None of these

14. a) 108 b) 91 c) 144d) 225 e) None of these

15. 187 : 11 b) 194 : 12 c) 195 : 13d) 224 : 14 e) None of these

16. a) D b) G c) Hd) J e) None of these

17. a) SRT b) PON c) KJLd) VUW e) None of these

18. a) EVFU b) CXDW c) AZBXd) GTHS e) None of these

19. a) Squareb) Equilateral Trianglec) Rhombusd) Right Angle Triangle

20. Four of the following five are alike in a cretain way and so form a group.Which is the one that does not belong to that group?a) 21 b) 35 c) 42d) 56 e) 49

Page 199: Reasoning Made Easy - pdfstores.files.wordpress.com · REASONING MADE EASY 6 Geetika, Jasmine, Hema, Brinda, Ipsita and veena, once again not following any order. The following are

REASONING MADE EASY www.BankExamsToday.com

199

21. Four of the following five are alike in a certain way and so form a group.Which is the one that does not belong to that group?a) Table b) Desk c) Wardrobed) Computer e) Chair

22. Four of the following five are alike in a certain way and so form a group.Which is one that does not belong to that group?a) Orange b) Apple c) Guavad) Papaya e) Mango

23. Find the odd one out.a) January, Mayb) April, Junec) July, Augustd) January, Decembere) None of these

Answers:

1. Option E2. Option D3. Option C4. Option E5. Option E6. Option C

Apart from 45, sum of both digits of every number will be an even numberbut 45 = 4 + 5 = 9 which is an odd number.

7. Option A

Except ‘sweet’ all the other are baked items.8. Option B

27 is the cube of 39. Option D10. Option E

289 = 17 × 17So, 289 is a perfect square.

11. Option B

Altitude means the height above sea l evel. Except altitude, all other wordsimply human nature and mental status.

12. Option C

Page 200: Reasoning Made Easy - pdfstores.files.wordpress.com · REASONING MADE EASY 6 Geetika, Jasmine, Hema, Brinda, Ipsita and veena, once again not following any order. The following are

REASONING MADE EASY www.BankExamsToday.com

200

SSA ⟶ ASS (An Animal)RMSA ⟶ MARS (A Planet)NASA ⟶ American Space Agency

13. Option D

Except 66, all others are odd numbers.

14. Option BThe number 91 is a product of two prime numbers. 13 × 7 = 91

15. Option B

Except in the number pair 194-12, in all other number pairs the first numberis completely divisible by the second number.187/11 = 17; 195/13 = 15; 224/14 = 16But, 194/12 = 16.166

16. Option B

D ⟶ position number 4H ⟶ position number 8J ⟶ position number 10G ⟶ position number 7 (An odd number)

17. Option B18. Option C19. Option D

Except in the case of right angle triangle, in all other geometrical figures, allthe sides are equal.

20. Option EThe number of 49 is a perfect square of a natural number.

21. Option DComputer is a mechanical device. All others are wooden items.

22. Option AOrange is citrus fruit.

23. Optoin BExcept the months of April and June, all other months have 31 days each.

Page 201: Reasoning Made Easy - pdfstores.files.wordpress.com · REASONING MADE EASY 6 Geetika, Jasmine, Hema, Brinda, Ipsita and veena, once again not following any order. The following are

REASONING MADE EASY www.BankExamsToday.com

201

Chapter – 16

Ranking Based Problems

1. In a row of 35 children, M is 15th from the right end and there are 10children between M and R. What is R’s position from the left end of therow?a) 15th b) 5th c) 30th

d) Data inadequate e) None of the above

2. Among B, F, J, K and W each having a different weight, F is heavier thanonly J. B I s heavier than F and W but not as h eavy as K. Who amongthem is the third heaviest among them?a) B b) F c) Kd) W e) None of these

3. J, D, L, H and F each travelling to station, each one reaches at a differenttime. L reaches only after J and D reaches only before F. Who amongstthem is 3rd to reach?a) F b) L c) Dd) Cannot be determined e) None of these

4. In a queue, Amrita is 10th from the front while Mukul is 25th from behindand Mamta is just in the middle of the two. If there be 50 persons in thequeue. What position does Mamta occupy from the front?a) 20th b) 19th c) 18th

d) 17th e) None of these

5. Some boys are sitting in a row. P is sitting 14th from the left and Q is 7th

from the right. If there are four boys between P and Q. How many boys arethere in the row?a) 25 b) 23 c) 21d) 19 e) None of these

6. Standing on a platform, Amit told Sunita that Aligarh was more than 10kilometers but less than 15 kilometers from there. Sunita knew that it wasmore than 12 but less than 14 kilometers from there. If both of them werecorrect, which of the following could be the distance of Aligarh from theplatform?a) 11 km b) 12 km c) 13 kmd) 14 km e) 15 km

7. The train for Lucknow leaves every two and a half hours from New DelhiRailway Station. An announcement was made at the station that the train forLucknow had left 40 minutes ago and the next train will leave at 18.00 hrs.At what time was the announcement made?

Page 202: Reasoning Made Easy - pdfstores.files.wordpress.com · REASONING MADE EASY 6 Geetika, Jasmine, Hema, Brinda, Ipsita and veena, once again not following any order. The following are

REASONING MADE EASY www.BankExamsToday.com

202

a) 15.30 hrs. b) 17.10 hrs. c) 16.00hrs.d) 15.50 hrs. e) None of these

8. How many 6’s are there in the following series of numbers which arepreceded by 7 but not immediately followed by 9?6 7 9 5 6 9 7 6 8 7 6 7 8 6 9 4 6 7 7 6 9 5 7 6 3a) One b) Two c) Threed) Four e) None of these

9. How many 7’s are there in the following sequence which are preceded by 9and followed by 6?7 8 9 7 6 5 3 4 2 8 9 7 2 4 5 9 2 9 7 6 4 7a) 2 b) 3 c) 4d) 5 e) None of these

10. How many 6’s are there in the following number sequence which areimmediately preceded by 9 but not immediately followed by 4?5 6 4 3 2 9 6 3 1 6 4 9 6 4 21 5 9 6 7 2 1 4 7 4 9 6 4 2a) One b) Two c) Threed) Four e) More than four

11. 5 1 4 7 3 9 8 5 7 2 6 3 1 5 8 6 3 8 5 2 2 4 3 4 9 6How many odd numbers are there in the sequence which are immediatelypreceded and also immediately followed by an even number?a) 1 b) 2 c) 3d) 4 e) More than 4

12. The letters L, M, N, O, P, Q, R, S and T in their order are substituted bynine integers 1 to 9 but not in that order. 4 is assigned to P. The differencebetween P and T is 5. The difference between N and T is 3. What is theinteger asigned to N?a) 4 b) 5 c) 6d) 7 e) None of these

Answers:

1. Option D2. Option D

K > B > W > F > JSo, W is the third heaviest.

3. Option EJ > L > H > D > FH is 3rd to reach.

4. Option C

Page 203: Reasoning Made Easy - pdfstores.files.wordpress.com · REASONING MADE EASY 6 Geetika, Jasmine, Hema, Brinda, Ipsita and veena, once again not following any order. The following are

REASONING MADE EASY www.BankExamsToday.com

203

5. Option A

6. Option C

7. Option E

8. Option C

9. Option A

10. Option B

11. Option D

12. Option C

Page 204: Reasoning Made Easy - pdfstores.files.wordpress.com · REASONING MADE EASY 6 Geetika, Jasmine, Hema, Brinda, Ipsita and veena, once again not following any order. The following are

REASONING MADE EASY www.BankExamsToday.com

204

Chapter – 17

Arithmatic Reasoning

1. The total of the ages of Amar, Akbar and Anthony is 80 years. What wasthe total of their ages three years ago?a) 71 years b) 72 years c) 74 yearsd) 77 years e) None of these

2. A woman says, “If you reverse my own age, the figures represent myhusband’s age. He is, of course, senior to me and the difference betweenour ages is one-eleventh of their sum.” The woman’s age isa) 23 years b) 34 years c) 45 yearsd) Can’t be determined e) None of these

3. In a garden, there are 10 rows and 12 coloumns of mango trees. Thedistance between the two trees is 2 metres and a distance of one metre is leftfrom all sides of the boundary of the garden. The length of the garden isa) 20 m b) 22 m c) 24 md) 26 m e) None of these

4. A motorist knows four different routes from Bristol to Birmingham. FromBirmingham to Shefield he knows three different routes and from Sheffieldto Carlisle he knows two different routes. How many routes does he knowfrom Bristol to Carlisle?a) 4 b) 8 c) 12d) 24 e) None of these

5. If 100 cats kill 100 mice in 100 days, then 4 cats would kill 4 mice in howmany days?a) 1 day b) 4 days c) 40 daysd) 100 days e) None of these

6. Five bells begin to toll together and toll respectively at intervals of 6, 5, 7,10 and 12 seconds. How many times will they toll togethe in one hourexcluding the one at the start?a) 7 times b) 8 times c) 9 timesd) 11 times e) None of these

7. Today is Varun’s birthday. One year, from today he will be twice as old ashe was 12 years ago. How old is Varun today?a) 20 years b) 22 years c) 25yearsd) 27 years e) None of these

Page 205: Reasoning Made Easy - pdfstores.files.wordpress.com · REASONING MADE EASY 6 Geetika, Jasmine, Hema, Brinda, Ipsita and veena, once again not following any order. The following are

REASONING MADE EASY www.BankExamsToday.com

205

8. When Rahul was born, his father was 32 years older than his brother and hismother was 25 years older than his sister. If Rahul’s brother is 6 yearsoldern than him and his mother is 3 years younger than his father, how oldwas Rahul’s sister when he was born?a) 7 years b) 10 years c) 14yearsd) 19 years e) None of these

9. A shepherd had 17 sheep. All but nine died. How many was he left witha) Nil b) 8 c) 9d) 17 e) None of these

10. The total number of digits used in numbering the pages of a book having366 pages isa) 732 b) 990 c) 1098d) 1305 e) None of these

11. A student got twice as many sums wrong as he got right. If he attempted 48sums in all, how did he solve correctly?a) 12 b) 16 c) 18d) 24 e) None of these

12. A father tells his son, “I was of your present age when you were born.” Ifthe father is 36 now, how old was the boy five years back?a) 13 b) 15 c) 17d) 20 e) None of these

13. Find the number which when added to itself 13 times, gives 112a) 7 b) 8 c) 9d) 11 e) None of these

14. At the end of a business conference the ten people present all shake handswith each other once. How many handshakes will there be altogether?a) 20 b) 45 c) 55d) 90 e) None of these

15. A group of 1200 persons consisting of captains and soldiers is travelling ina train. For every 15 soldiers there is one captain. The number of captains inthe group isa) 85 b) 80 c) 75d) 70 e) None of these

16. A placed three sheets with two carbons to get two extra copies of theoriginal. Then he decided to get more carbon copies and folded the paper insuch a way that the upper half of the sheets were on top of the lower half.Then he typed. How many carbon copies did he get?

Page 206: Reasoning Made Easy - pdfstores.files.wordpress.com · REASONING MADE EASY 6 Geetika, Jasmine, Hema, Brinda, Ipsita and veena, once again not following any order. The following are

REASONING MADE EASY www.BankExamsToday.com

206

a) 1 b) 2 c) 3d) 4 e) None of these

17. Aruna cut a cake into two halves and cuts one half into smaller pieces ofequal size. Each of the small pieces is 20 grams in weight. If she has sevenpieces of the cake in all with her, how heavy was the original cake?a) 240 gm b) 220 gm c) 225 gmd) 250 gm e) None of these

18. The number of boys in a class is three times the number of girls. Which oneof the following numbers cannot represent the total number of children inthe class?a) 48 b) 44 c) 42d) 40 e) None of these

19. In a group of 15 people, 7 read French, 8 read English while 3 of them readnone of these two. How many of them read French and English both?a) 2 b) 3 c) 4d) 5 e) None of these

20. A tailor had a number of shirt pieces to cut from a roll of fabric. He cuteach roll of equal length into 10 pieces. He cut at the rate of 45 cuts aminute. How many rolls would be cut in 24 minutes?a) 30 rolls b) 58 rolls c) 120rollsd) 150 rolls e) None of these

21. There are deer and peacocks in a zoo. By counting heads they are 80. Thenumber of their legs is 200. How many peacocks are there?a) 10 b) 20 c) 40d) 60 e) None of these

22. A farmer built a fence around his square plot. He used 27 fence poles oneach side of the square. How many poles did he need altogether?a) 54 b) 84 c) 104d) 108 e) None of these

23. I have a few sweets to be distributed. If I keep 2, 3 or 4 in pack, I am leftwith one sweet. If I keep 5 in a pack, I am left with none. What is theminimum number of sweets I have to pack and distribute?a) 25 b) 35 c) 56d) 65 e) None of these

24. A man has Rs.480 in the denominations of one rupee notes, five rupee notesand ten rupee notes. The number of each denomination is equal. What is thetotal number of notes that he has?

Page 207: Reasoning Made Easy - pdfstores.files.wordpress.com · REASONING MADE EASY 6 Geetika, Jasmine, Hema, Brinda, Ipsita and veena, once again not following any order. The following are

REASONING MADE EASY www.BankExamsToday.com

207

a) 45 b) 75 c) 90d) 120 e) None of these

25. A is 3 years older to B and 3 years younger to C, while B and D are twins.How many years older is C and D?a) 3 b) 6 c) 9d) 12 e) None of these

Answers:

1. Required sum = (80 ⎯3 × 3) years = (80 ⎯9) years = 71 years

2. Let x and y be the ten’s and unit’s digits respectively of the numeraldenoting the woman’s ageThen, woman’s age = (10x + y) years, husband’s age = (10y + x) yearsTherefore (10y + x) ⎯(10x + y) = (1/11) (10y + x + 10x + y)(9y ⎯9x) = (1/11) (11y + 11x) = (x + y)10x = 8yx = 4/5yClearly, y should be a single digit multiple of 5, which is 5.Hence, woman’s age = 10x + y = 45 years

3. Each row contains 12 plants.There are 11 gapes between two corner trees (11 × 2) metres and 1 metre oneach side is left.Therefore length = (22 + 2)m = 24 m

4. Total number of routes from Bristol to Carlisle = (4 × 3 × 2) = 24

5. Less cats, more days (indirect proportion)Less mice, less days (direct proportion)Let the required number of days be x.Cat 4 : 100 :: x : 100Mice 100 : 4100 × 4 × x = 4 × 100 × 100 or x = 4 × 100 × 100 / 100 × 4 = 100

6. L.C.M. of 6, 5, 7, 10 and 12 is 420So, the bells will toll together after every 420 seconds i.e. 7 minutes.Now, 7 × 8 = 56 and 7 × 9 = 63Thus, in 1 hour (or 60 minutes), the bells will toll together 8 times,excluding the one at the start.

7. Let Varun’s age today = x yearsThen, Varun’s age after 1 year = x + 1 yearsTherefore, x + 1 = 2 (x ⎯12)x + 1 = 2x ⎯24

Page 208: Reasoning Made Easy - pdfstores.files.wordpress.com · REASONING MADE EASY 6 Geetika, Jasmine, Hema, Brinda, Ipsita and veena, once again not following any order. The following are

REASONING MADE EASY www.BankExamsToday.com

208

x = 25

8. When Rahul was born, his brother’s age = 6 years, his father’s age = 6 + 32years = 38 years, his mother’s age = 38 ⎯3 = 35 years, his sister’s age = 35⎯25 = 10 years

9. ‘All but nine died’ means ‘All except nine died’ i.e. 9 sheep remained alive.

10. Total number of digits = (Number of digits in 1 ⎯digit page numbers +number of digits in 2 ⎯digit page numbers + number of digits in 3 ⎯digitpage numbers= (1 × 9 + 2 × 90 + 3 × 267) = (9 + 180 + 801) = 990

11. Suppose the boy got x sums right and 2x sums wrong.Then, x + 2x = 483x = 48x = 16

12. Let the father’s age be x and the son’s age be y.Then, x ⎯y = y or x = 2yNow, x = 36, So, 2y = 36 or y = 18Therefore Son’s present age = 18 yearsSo, son’s age 5 years ago = 13 years

13. Let the number be x. Then x + 13x = 11214x = 112x = 8

14. Total number of handshakes = 9 + 8 + 7 + 6 + 5 + 4 + 3 + 2 + 1 = 45

15. Out of every 16 persons, there is one captain, so, number of captains1200/16 = 75

16. Since the number of carbons is 2, only two copies can be obtained.

17. Seven pieces consist of 6 smaller equal pieces and one half cake piece.Weight of each small piece = 20 gmSo, total weight of the cake = 2 (20 × 6) = 240 gm

18. Let number of girls = x and number of boys = 3xThen, 3x + x = 4x = total number of studentsThus, to find exact value of x, the total number of students must be divisibleby 4

19. Option B

Page 209: Reasoning Made Easy - pdfstores.files.wordpress.com · REASONING MADE EASY 6 Geetika, Jasmine, Hema, Brinda, Ipsita and veena, once again not following any order. The following are

REASONING MADE EASY www.BankExamsToday.com

209

20. Option C21. Option D22. Option C23. Option A24. Option C25. Option B

Page 210: Reasoning Made Easy - pdfstores.files.wordpress.com · REASONING MADE EASY 6 Geetika, Jasmine, Hema, Brinda, Ipsita and veena, once again not following any order. The following are

REASONING MADE EASY www.BankExamsToday.com

210

Chapter – 18

Classification

Directions (Q. 1-5): In each of the following questions, five words have been givenout of which four are alike in some manne, while 5th one is different. Choose theword which is different from the rest.

1. Choose the word which is different from the resta) Chicken b) Snake c) Swand) Crocodile e) Frog

2. Choose the word which is different from the resta) Cap b) Turban c) Helmetd) Veil e) Hat

3. Choose the word which is different from the resta) Kiwi b) Eagle c) Emud) Ostrich

4. Choose the word which is different from the resta) Rigveda b) Yajurveda c) Atharvavedad) Ayurveda e) Samveda

5. Choose the word which is different from the resta) Curd b) Butter c) Oild) Cheese e) Cream

Directions (Q. 6-10): Out of the five figures marked (1), (2), (3), (4) and (5), fourare similar in a certain manner. However, one figure is not like the other four.Choose the figure which is different from the rest.

6. Choose the figure which is different from the rest.

(1) (2) (3) (4) (5)a) 1 b) 2 c) 3d) 4 e) 5

7. Choose the figure which is different from the rest.

(1) (2) (3) (4) (5)a) 1 b) 2 c) 3

REASONING MADE EASY www.BankExamsToday.com

210

Chapter – 18

Classification

Directions (Q. 1-5): In each of the following questions, five words have been givenout of which four are alike in some manne, while 5th one is different. Choose theword which is different from the rest.

1. Choose the word which is different from the resta) Chicken b) Snake c) Swand) Crocodile e) Frog

2. Choose the word which is different from the resta) Cap b) Turban c) Helmetd) Veil e) Hat

3. Choose the word which is different from the resta) Kiwi b) Eagle c) Emud) Ostrich

4. Choose the word which is different from the resta) Rigveda b) Yajurveda c) Atharvavedad) Ayurveda e) Samveda

5. Choose the word which is different from the resta) Curd b) Butter c) Oild) Cheese e) Cream

Directions (Q. 6-10): Out of the five figures marked (1), (2), (3), (4) and (5), fourare similar in a certain manner. However, one figure is not like the other four.Choose the figure which is different from the rest.

6. Choose the figure which is different from the rest.

(1) (2) (3) (4) (5)a) 1 b) 2 c) 3d) 4 e) 5

7. Choose the figure which is different from the rest.

(1) (2) (3) (4) (5)a) 1 b) 2 c) 3

REASONING MADE EASY www.BankExamsToday.com

210

Chapter – 18

Classification

Directions (Q. 1-5): In each of the following questions, five words have been givenout of which four are alike in some manne, while 5th one is different. Choose theword which is different from the rest.

1. Choose the word which is different from the resta) Chicken b) Snake c) Swand) Crocodile e) Frog

2. Choose the word which is different from the resta) Cap b) Turban c) Helmetd) Veil e) Hat

3. Choose the word which is different from the resta) Kiwi b) Eagle c) Emud) Ostrich

4. Choose the word which is different from the resta) Rigveda b) Yajurveda c) Atharvavedad) Ayurveda e) Samveda

5. Choose the word which is different from the resta) Curd b) Butter c) Oild) Cheese e) Cream

Directions (Q. 6-10): Out of the five figures marked (1), (2), (3), (4) and (5), fourare similar in a certain manner. However, one figure is not like the other four.Choose the figure which is different from the rest.

6. Choose the figure which is different from the rest.

(1) (2) (3) (4) (5)a) 1 b) 2 c) 3d) 4 e) 5

7. Choose the figure which is different from the rest.

(1) (2) (3) (4) (5)a) 1 b) 2 c) 3

Page 211: Reasoning Made Easy - pdfstores.files.wordpress.com · REASONING MADE EASY 6 Geetika, Jasmine, Hema, Brinda, Ipsita and veena, once again not following any order. The following are

REASONING MADE EASY www.BankExamsToday.com

211

d) 4 e) 5

8. Choose the figure which is different from the rest.

(1) (2) (3) (4) (5)a) 1 b) 2 c) 3d) 4 e) 5

9. Choose the figure which is different from the rest.

(1) (2) (3) (4) (5)a) 1 b) 2 c) 3d) 4 e) 5

10. Choose the figure which is different from the rest.

(1) (2) (3) (4) (5)a) 1 b) 2 c) 3d) 4 e) 5

Directions (Q. 11-15): In each of the following questions, five words have beengiven out of which four are alike in some manner, while 5th one is different. Choosethe word which is different from the rest.

11. Choose the word which is different from the resta) Pear b) Apple c) Litchid) Guava e) Orange

12. Choose the word which is different from the resta) Dagger b) Hammer c) Knifed) Sword e) Blade

13. Choose the word which is different from the resta) Kanpur b) Allahabad c) Varanasid) Mathura

14. Choose the word which is different from the resta) Oyster b) Clam c) Scallopd) Mussel

15. Choose the word which is different from the rest

REASONING MADE EASY www.BankExamsToday.com

211

d) 4 e) 5

8. Choose the figure which is different from the rest.

(1) (2) (3) (4) (5)a) 1 b) 2 c) 3d) 4 e) 5

9. Choose the figure which is different from the rest.

(1) (2) (3) (4) (5)a) 1 b) 2 c) 3d) 4 e) 5

10. Choose the figure which is different from the rest.

(1) (2) (3) (4) (5)a) 1 b) 2 c) 3d) 4 e) 5

Directions (Q. 11-15): In each of the following questions, five words have beengiven out of which four are alike in some manner, while 5th one is different. Choosethe word which is different from the rest.

11. Choose the word which is different from the resta) Pear b) Apple c) Litchid) Guava e) Orange

12. Choose the word which is different from the resta) Dagger b) Hammer c) Knifed) Sword e) Blade

13. Choose the word which is different from the resta) Kanpur b) Allahabad c) Varanasid) Mathura

14. Choose the word which is different from the resta) Oyster b) Clam c) Scallopd) Mussel

15. Choose the word which is different from the rest

REASONING MADE EASY www.BankExamsToday.com

211

d) 4 e) 5

8. Choose the figure which is different from the rest.

(1) (2) (3) (4) (5)a) 1 b) 2 c) 3d) 4 e) 5

9. Choose the figure which is different from the rest.

(1) (2) (3) (4) (5)a) 1 b) 2 c) 3d) 4 e) 5

10. Choose the figure which is different from the rest.

(1) (2) (3) (4) (5)a) 1 b) 2 c) 3d) 4 e) 5

Directions (Q. 11-15): In each of the following questions, five words have beengiven out of which four are alike in some manner, while 5th one is different. Choosethe word which is different from the rest.

11. Choose the word which is different from the resta) Pear b) Apple c) Litchid) Guava e) Orange

12. Choose the word which is different from the resta) Dagger b) Hammer c) Knifed) Sword e) Blade

13. Choose the word which is different from the resta) Kanpur b) Allahabad c) Varanasid) Mathura

14. Choose the word which is different from the resta) Oyster b) Clam c) Scallopd) Mussel

15. Choose the word which is different from the rest

Page 212: Reasoning Made Easy - pdfstores.files.wordpress.com · REASONING MADE EASY 6 Geetika, Jasmine, Hema, Brinda, Ipsita and veena, once again not following any order. The following are

REASONING MADE EASY www.BankExamsToday.com

212

a) Deck b) Quay c) Sternd) Bow e) Mast

Directions (Q. 16-20): Out of the five figures marked (1), (2), (3), (4) and (5), fourare similar in a certain manner. However, one figure is not like the other four.Choose the figure which is different from the rest.

16. Choose the figure which is different from the rest.

(1) (2) (3) (4) (5)a) 1 b) 2 c) 3d) 4 e) 5

17. Choose the figure which is different from the rest.

(1) (2) (3) (4) (5)a) 1 b) 2 c) 3d) 4 e) 5

18. Choose the figure which is different from the rest.

(1) (2) (3) (4) (5)a) 1 b) 2 c) 3d) 4 e) 5

19. Choose the figure which is different from the rest.

(1) (2) (3) (4) (5)a) 1 b) 2 c) 3d) 4 e) 5

20. Choose the figure which is different from the rest.

(1) (2) (3) (4) (5)a) 1 b) 2 c) 3d) 4 e) 5

REASONING MADE EASY www.BankExamsToday.com

212

a) Deck b) Quay c) Sternd) Bow e) Mast

Directions (Q. 16-20): Out of the five figures marked (1), (2), (3), (4) and (5), fourare similar in a certain manner. However, one figure is not like the other four.Choose the figure which is different from the rest.

16. Choose the figure which is different from the rest.

(1) (2) (3) (4) (5)a) 1 b) 2 c) 3d) 4 e) 5

17. Choose the figure which is different from the rest.

(1) (2) (3) (4) (5)a) 1 b) 2 c) 3d) 4 e) 5

18. Choose the figure which is different from the rest.

(1) (2) (3) (4) (5)a) 1 b) 2 c) 3d) 4 e) 5

19. Choose the figure which is different from the rest.

(1) (2) (3) (4) (5)a) 1 b) 2 c) 3d) 4 e) 5

20. Choose the figure which is different from the rest.

(1) (2) (3) (4) (5)a) 1 b) 2 c) 3d) 4 e) 5

REASONING MADE EASY www.BankExamsToday.com

212

a) Deck b) Quay c) Sternd) Bow e) Mast

Directions (Q. 16-20): Out of the five figures marked (1), (2), (3), (4) and (5), fourare similar in a certain manner. However, one figure is not like the other four.Choose the figure which is different from the rest.

16. Choose the figure which is different from the rest.

(1) (2) (3) (4) (5)a) 1 b) 2 c) 3d) 4 e) 5

17. Choose the figure which is different from the rest.

(1) (2) (3) (4) (5)a) 1 b) 2 c) 3d) 4 e) 5

18. Choose the figure which is different from the rest.

(1) (2) (3) (4) (5)a) 1 b) 2 c) 3d) 4 e) 5

19. Choose the figure which is different from the rest.

(1) (2) (3) (4) (5)a) 1 b) 2 c) 3d) 4 e) 5

20. Choose the figure which is different from the rest.

(1) (2) (3) (4) (5)a) 1 b) 2 c) 3d) 4 e) 5

Page 213: Reasoning Made Easy - pdfstores.files.wordpress.com · REASONING MADE EASY 6 Geetika, Jasmine, Hema, Brinda, Ipsita and veena, once again not following any order. The following are

REASONING MADE EASY www.BankExamsToday.com

213

Answers:

1. Option A2. Option D3. Option B4. Option D5. Option C6. Option A7. Option E8. Option D9. Option B10. Option B11. Option E12. Option B13. Option D14. Option C15. Option B16. Option D17. Option C18. Option C19. Option D20. Option C

Page 214: Reasoning Made Easy - pdfstores.files.wordpress.com · REASONING MADE EASY 6 Geetika, Jasmine, Hema, Brinda, Ipsita and veena, once again not following any order. The following are

REASONING MADE EASY www.BankExamsToday.com

214

Chapter – 19

Dice Problem

1. Which symbol will be on the face opposite to the face with symbol *

a) @ b) $ c) 8d) + e) None of these

2. Two positions of a dice are shown below. When number ‘1’ on the top.What number will be at the bottom?a) 3 b) 5 c) 2d) 6 e) None of these

3. From the four positions of a dice given below, find the colour which isopposite to yellow?a) Violet b) Red c) Rosed) Blue e) None of these

4. From the positions of a cube are shown below, which letter will be on theface opposite to face with ‘A’?a) D b) B c) Cd) F e) None of these

5. Which digit will appear on the face opposite to the face with number 4?

a) 3 b) 5 c) 6d) 2/3 e) None of these

6. Which number is on the face opposite to 6?

a) 4 b) 1 c) 2d) 3 e) None of these

Page 215: Reasoning Made Easy - pdfstores.files.wordpress.com · REASONING MADE EASY 6 Geetika, Jasmine, Hema, Brinda, Ipsita and veena, once again not following any order. The following are

REASONING MADE EASY www.BankExamsToday.com

215

7. When the digit 5 is on the bottom then which number will be on its uppersurface?

a) 1 b) 3 c) 4d) 6 e) None of these

8. Observe the dots on the dice (one to six dots) in the following figures. Howmany dots are contained on the face opposite to the containing four dots?

a) 2 b) 3 c) 5d) 6 e) None of these

9. Four usual dice are thrown on the ground. The total of number on the topfaces of these four dice is 13 as the top faces showed 4, 3, 1 and 5respectively. What is the total of the faces touching the ground?a) 12 b) 13 c) 15d) Can’t be determined e) None of these

10. A dice is numbered from 1 to 6 in different ways.If 1 is opposite to 5 and 2 is opposite to 3, thena) 4 is adjacent to 3 and 6b) 2 is adjacent to 4 and 6c) 4 is adjacent to 5 and 6d) 6 is adjacent to 3 and 4

11. Which symbol will be on the face opposite to the face with symbol C?

a) D b) E c) B or Dd) F e) None of these

12. Which number is on the face opposite to 56?

Page 216: Reasoning Made Easy - pdfstores.files.wordpress.com · REASONING MADE EASY 6 Geetika, Jasmine, Hema, Brinda, Ipsita and veena, once again not following any order. The following are

REASONING MADE EASY www.BankExamsToday.com

216

a) 54 b) 55 c) 50d) Can’t be determined e) None of these

13. If there are five dots at the bottom, then how many dots will be on its uppersurface?

a) 1 b) 2 c) 3d) 4 e) None of these

14. Two positions of dice are shown below. What will be on the top when ‘B’is at bottom?

a) F b) D c) Ad) E e) None of these

15. Two positions of dice are shown below. What number will appear on theopposite to the face containing ‘5’?

a) 2 b) 4 c) 1d) 6 e) None of these

Page 217: Reasoning Made Easy - pdfstores.files.wordpress.com · REASONING MADE EASY 6 Geetika, Jasmine, Hema, Brinda, Ipsita and veena, once again not following any order. The following are

REASONING MADE EASY www.BankExamsToday.com

217

16. Two positions of a cube with surfaces are shown below. When the surface‘D’ touch the bottom, what surface will be on top?

a) D b) A c) Bd) F e) None of these

17. Two positions of a dice are shown below. When ‘C’ is at the bottom thenwhat is on top?

a) A b) B c) Cd) D e) None of these

18. Which digit will appear on the face opposite to the face with alphabet ‘d’?

a) a b) g c) cd) f e) None of these

19. Two positions of a cube are shown below. When the surface ‘D’ touch thebottom, what surface will be on the top?

Page 218: Reasoning Made Easy - pdfstores.files.wordpress.com · REASONING MADE EASY 6 Geetika, Jasmine, Hema, Brinda, Ipsita and veena, once again not following any order. The following are

REASONING MADE EASY www.BankExamsToday.com

218

a) B b) C c) Ed) Can’t be determined e) None of these

20. How many dot/dots will appear on the face opposite to the face with ‘FourDots’?

a) 3 dots b) 2 dots c) 1 dotd) 5 dots e) None of these

Answers:

1. Option CThe symbols of the adjacent faces to the face with symbol * are @, -, + and$. Hence the required symbol is 8.

2. Option BAccording to the rule (2) when ‘one’ is at the top, then 5 will be at thebottom.

3. Option AThe colours adjacent to yellow are orange, blue, red and rose. Hence violetwill be opposite to yellow.

4. Option AThe letters of the adjacent faces to the face with letter A, are B, F, C and E.Hence D is the letter of the face opposite to the face with letters (A).

5. Option AHere the common faces with number 3, are in same positions. Hence 6 isopposite to 2 and 5 is opposite to 1. Therefore 4 is opposite to 3.

6. Option BAs the numbers 2, 3, 4 and 5 are adjacent to 6. Hence the number on theface opposite to 6 is 1.

7. Option AAccording to the rule No.3, common faces with number 2 are in samepositions. Hence when the digit 5 is on the bottom then 1 will on the uppersurface.

Page 219: Reasoning Made Easy - pdfstores.files.wordpress.com · REASONING MADE EASY 6 Geetika, Jasmine, Hema, Brinda, Ipsita and veena, once again not following any order. The following are

REASONING MADE EASY www.BankExamsToday.com

219

8. Option A9. Option C

In a usual dice, the sum of the numbers on any two opposite faces is always7. Thus, 1 is opposite 6, 2 is opposite 5 and 3 is opposite 4. Consequently,when 4, 3, 1 and 5 are the numbers on the top faces, then 3, 4, 6 and 2respectively are the numbers on the face touching the ground. The total ofthese numbers = 3 + 4 + 6 + 2 = 15

10. Option BIf 1 is opposite to 5 and 2 is opposite to 3, then 4 definitely lies opposite to6. Therefore, 2 cannot lie opposite to any of the two numbers – 4 or 6.Hence, 2 necessarily lies adjacent to both 4 and 6.

11. Option D12. Option E13. Option A14. Option B15. Option B16. Option B17. Option E18. Option C19. Option E20. Option A

Page 220: Reasoning Made Easy - pdfstores.files.wordpress.com · REASONING MADE EASY 6 Geetika, Jasmine, Hema, Brinda, Ipsita and veena, once again not following any order. The following are

REASONING MADE EASY www.BankExamsToday.com

220

Chapter – 20

Statement and Arguments

Directions (Q. 1-5): Each question given below consists of a statement, followedby two arguments numbered I and II. You have to decide which of the arguments isa strong argument and which is a weak argument.

Give answer:a) If only argument I is strongb) If only argument II is strongc) If either I or II is strongd) If neither I nor II is stronge) If both I and II are strong

1. Statement: Should India encourage exports, when most things areinsufficient for internal use itself?Arguments:I. Yes, we have to earn foreignexchange to pay for our imports.II. No, even selective encouragement would lead to shortages.a) Only argument I is strongb) Only argument II is strongc) Either I or II is strongd) Neither I nor II is stronge) Both I and II are strong

2. Statement: Should all the drugs patented and manufactured in Westerncountries be first tried out on sample basis before giving licence for sale togeneral public in India?Arguments:I. Yes, many such drugs require different doses and duration for

Indian population and hence it is necessary.II. No, this is just not feasible and hence cannot be implemented.

a) Only argument I is strongb) Only argument II is strongc) Either I or II is strongd) Neither I nor II is stronge) Both I and II are strong

3. Statement: Should India make efforts to harness solar energy to fulfil itsenergy requirements?Arguments:I. Yes, most of the energy sources used at present is exhaustible.II. No, harnessing solar energy requires a lot of capital, which India

lacks in.

Page 221: Reasoning Made Easy - pdfstores.files.wordpress.com · REASONING MADE EASY 6 Geetika, Jasmine, Hema, Brinda, Ipsita and veena, once again not following any order. The following are

REASONING MADE EASY www.BankExamsToday.com

221

a) Only argument I is strongb) Only argument II is strongc) Either I or II is strongd) Neither I nor II is stronge) Both I and II are strong

4. Statement: Should there be students union in college/university?Arguments:I. No, this will create a political atmosphere in the campus.II. Yes, it is very necessary students are future political leaders.

a) Only argument I is strongb) Only argument II is strongc) Either I or II is strongd) Neither I nor II is stronge) Both I and II are strong

5. Statement: Should India give away Kashmir to Pakistan?Arguments:I. No, Kashmir is a beautiful State. It earns a lot of foreign exchange

for India.II. Yes, this would help settle conflicts.

a) Only argument I is strongb) Only argument II is strongc) Either I or II is strongd) Neither I nor II is stronge) Both I and II are strong

Directions (Q. 6-10): Each question given below consists of a statement, followedby two arguments numbered I and II. You have to decide which of the arguments isa strong argument and which is a weak argument.

Give answer:a) If only argument I is strongb) If only argument II is strongc) If either I or II is strongd) If neither I nor II is stronge) If both I and II are strong

6. Statement: Should there be an upper age limit of 65 years for contestingParliamentary/Legislative Assembly elections?Arguments:I. Yes, generally people above the age of 65 lose their dynamism and

will power.II. No, the life span is so increased that people remain physically and

mentally active even upto the age of 80.a) Only argument I is strong

Page 222: Reasoning Made Easy - pdfstores.files.wordpress.com · REASONING MADE EASY 6 Geetika, Jasmine, Hema, Brinda, Ipsita and veena, once again not following any order. The following are

REASONING MADE EASY www.BankExamsToday.com

222

b) Only argument II is strongc) Either I or II is strongd) Neither I nor II is stronge) Both I and II are strong

7. Statement: Should new big industries be started in Mumbai?Arguments:I. Yes, it will create job opportunities.II. No, it will further add to pollution of the city.

a) Only argument I is strongb) Only argument II is strongc) Either I or II is strongd) Neither I nor II is stronge) Both I and II are strong

8. Statement: Should high chimneys be installed in industries?Arguments:I. Yes, it reduces pollution at ground level.II. No, it increases pollution in upper atmosphere.

a) Only argument I is strongb) Only argument II is strongc) Either I or II is strongd) Neither I nor II is stronge) Both I and II are strong

9. Statement: Does India need so many plans for development?Arguments:I. Yes, nothing can be achieved without proper planning.II. No, too much time, money and energy is wasted on planning.

a) Only argument I is strongb) Only argument II is strongc) Either I or II is strongd) Neither I nor II is stronge) Both I and II are strong

10. Statement: Should articles of only deserving authors be allowed to bepublished?Arguments:I. Yes, it will save a lot of paper which is in short reply.II. No, it is not possible to draw a line between the deserving and the

undeserving.a) Only argument I is strongb) Only argument II is strongc) Either I or II is strongd) Neither I nor II is stronge) Both I and II are strong

Page 223: Reasoning Made Easy - pdfstores.files.wordpress.com · REASONING MADE EASY 6 Geetika, Jasmine, Hema, Brinda, Ipsita and veena, once again not following any order. The following are

REASONING MADE EASY www.BankExamsToday.com

223

Directions (Q. 11-15): Each question given below consists of a statement, followedby two arguments numbered I and II. You have to decide which of the arguments isa strong argument and which is a wweak argument.

Give answer:a) If only argument I is strongb) If only argument II is strongc) If either I or II is strongd) If neither I nor II is stronge) If both I and II are strong

11. Statement: Should all the unauthorized structures in the city bedemolished?Arguments:I. No, where will the people residing in such houses live?II. Yes, this will give a clear message to general public and they will

refrain from constructing unauthorized buildings.a) Only argument I is strongb) Only argument II is strongc) Either I or II is strongd) Neither I nor II is stronge) Both I and II are strong

12. Statement: Should there be a maximum limit for the number of ministersin the Central Government?Arguments:I. No, the political party in power should have the freedom to decide

the number of ministers to be appointed.II. Yes, the number of ministers should be restricted to a certain

percentage of the total number of seats in the Parliament to avoidunncessary expenditure.a) Only argument I is strongb) Only argument II is strongc) Either I or II is strongd) Neither I nor II is stronge) Both I and II are strong

13. Statement: Should foreign films be banned in India?Arguments:I. Yes, they depict an alien culture which adversely affects our values.II. No, foreign films are of a high artistic standard.

a) Only argument I is strongb) Only argument II is strongc) Either I or II is strongd) Neither I nor II is strong

Page 224: Reasoning Made Easy - pdfstores.files.wordpress.com · REASONING MADE EASY 6 Geetika, Jasmine, Hema, Brinda, Ipsita and veena, once again not following any order. The following are

REASONING MADE EASY www.BankExamsToday.com

224

e) Both I and II are strong

14. Statement: Is buying things on installments profitable to the customer?Arguments:I. Yes, he has to pay less.II. No, paying installments upsets the family budget.

a) Only argument I is strongb) Only argument II is strongc) Either I or II is strongd) Neither I nor II is stronge) Both I and II are strong

15. Statement: Should Doordarshan be given autonomous status?Arguments:I. Yes, it will help Doordarshan to have fair and impartial coverage of

all important events.II. No, the coverage of events will be decided by a few who may not

have healthy outlook.a) Only argument I is strongb) Only argument II is strongc) Either I or II is strongd) Neither I nor II is stronge) Both I and II are strong

Directions (Q. 16-20): Each question given below consists of a statement, followedby two arguments numbered I and II. You have to decide which of the arguments isa strong argument and which is a weak argument.

Give answer:a) If only argument I is strongb) If only argument II is strongc) If either I or II is strongd) If neither I nor II is stronge) If both I and II are strong

16. Statement: Should Government jobs in rural areas have more incentives?Arguments:I. Yes, incentives are essential for attracting government servants

there.II. No, rural areas are already cheaper, healthier and less complex than

big cities. So? Why offer extra incentives.a) Only argument I is strongb) Only argument II is strongc) Either I or II is strongd) Neither I nor II is stronge) Both I and II are strong

Page 225: Reasoning Made Easy - pdfstores.files.wordpress.com · REASONING MADE EASY 6 Geetika, Jasmine, Hema, Brinda, Ipsita and veena, once again not following any order. The following are

REASONING MADE EASY www.BankExamsToday.com

225

17. Statement: Should there be a cap on maximum number of contestants forParliamentary elections in any constituency?Arguments:I. Yes, this will make the Parliamentary elections more meaningful as

the voters can make a considered judgment for casting their vote.II. No, in a democracy any person fulfilling the eligibility criteria can

contest Parliamentary elections and there should be no restrictions.a) Only argument I is strongb) Only argument II is strongc) Either I or II is strongd) Neither I nor II is stronge) Both I and II are strong

18. Statement: Should so much money be spent on advertisements?Arguments:I. Yes, it is an essential concomitant in a capitalist economy.II. No, it leads to wastage of resources.

a) Only argument I is strongb) Only argument II is strongc) Either I or II is strongd) Neither I nor II is stronge) Both I and II are strong

19. Statement: Should all the legislators be forced to resign from theirprofession?Arguments:I. Yes, they will be able to devote more time for the country.II. No, nobody will contest election.

a) Only argument I is strongb) Only argument II is strongc) Either I or II is strongd) Neither I nor II is stronge) Both I and II are strong

20. Statement: Should computer knowledge be made a compulsory subject forall the students at secondary school level?Arguments:I. No, our need is bread for everyone, we cannot follow western

models.II. Yes, we cannot compete in the international market without

equipping our children with computers.a) Only argument I is strongb) Only argument II is strongc) Either I or II is strongd) Neither I nor II is strong

Page 226: Reasoning Made Easy - pdfstores.files.wordpress.com · REASONING MADE EASY 6 Geetika, Jasmine, Hema, Brinda, Ipsita and veena, once again not following any order. The following are

REASONING MADE EASY www.BankExamsToday.com

226

e) Both I and II are strong

Directions (Q. 21-25): Each question given below consists of a statement, followedby two arguments numbered I and II. You have to decide which of the arguments isa strong argument and which is a weak argument.

Give answer:a) If only argument I is strongb) If only argument II is strongc) If either I or II is strongd) If neither I nor II is stronge) If both I and II are strong

21. Statement: Should religion be banned?Argument:I. Yes, it develops fanaticism in people.II. No, religion binds people together.

a) Only argument I is strongb) Only argument II is strongc) Either I or II is strongd) Neither I nor II is stronge) Both I and II are strong

22. Statement: Should India become a permanent member of UN’s SecurityCouncil?Arguments:I. Yes, India has emerged as a country which loves peace and amity.II. No, let us first solve problems of our own people like poverty,

malnutrition.a) Only argument I is strongb) Only argument II is strongc) Either I or II is strongd) Neither I nor II is stronge) Both I and II are strong

23. Statement: Should fashionable dresses be banned?Arguments:I. Yes, fashions keep changing and hence consumption of cloth

increases.II. No, fashionable clothes are a peson’s self expression and therefore

his/her fundamental right.a) Only argument I is strongb) Only argument II is strongc) Either I or II is strongd) Neither I nor II is stronge) Both I and II are strong

Page 227: Reasoning Made Easy - pdfstores.files.wordpress.com · REASONING MADE EASY 6 Geetika, Jasmine, Hema, Brinda, Ipsita and veena, once again not following any order. The following are

REASONING MADE EASY www.BankExamsToday.com

227

24. Statement: Should an organization like UNO be dissolved?Arguments:I. Yes, with cold war coming to an end, such organizations have no

role to play.II. No, in the absence of such organizations there may be a World

War.a) Only argument I is strongb) Only argument II is strongc) Either I or II is strongd) Neither I nor II is stronge) Both I and II are strong

25. Statement: Should there be no place of interview in selection?Arguments:I. Yes, it is very subjective in assessment.II. No, it is the only instrument to judge candidates motives and

personality.a) Only argument I is strongb) Only argument II is strongc) Either I or II is strongd) Neither I nor II is stronge) Both I and II are strong

Directions (Q. 26-30): Each question given below consists of a statement, followedby two arguments numbered I and II. You have to decide which of the arguments isa strong argument and which is a weak argument.

Give answer:a) If only argument I is strongb) If only argument II is strongc) If either I or II is strongd) If neither I nor II is stronge) If both I and II are strong

26. Statement: Should children be legally made responsible to take care oftheir parents during their old age?Arguments:I. Yes, such matter can only be solved by legal means.II. Yes, only this will bring some relief to poor parents.

a) Only argument I is strongb) Only argument II is strongc) Either I or II is strongd) Neither I nor II is stronge) Both I and II are strong

Page 228: Reasoning Made Easy - pdfstores.files.wordpress.com · REASONING MADE EASY 6 Geetika, Jasmine, Hema, Brinda, Ipsita and veena, once again not following any order. The following are

REASONING MADE EASY www.BankExamsToday.com

228

27. Statement: Should there be reservation in Government jobs for candidatesfrom single child family?Arguments:I. No, this is not advisable as the jobs should be offered to only

deserving candidates without any reservation for a particular group.II. Yes, this will help reduce the growing population in India as the

parents will be encouraged to adopt single child norm.a) Only argument I is strongb) Only argument II is strongc) Either I or II is strongd) Neither I nor II is stronge) Both I and II are strong

28. Statement: Should higher education be completely stopped for some time?Arguments:I. No, it will hamper the country’s future progress.II. Yes, it will reduce the educated unemployment.

a) Only argument I is strongb) Only argument II is strongc) Either I or II is strongd) Neither I nor II is stronge) Both I and II are strong

29. Statement: Should we scrap the Public Distribution System in India?Arguments:I. Yes, protectionism is over, everyone must get the bread on his/her

own.II. Yes, the poor do not get any benefit because of corruption.

a) Only argument I is strongb) Only argument II is strongc) Either I or II is strongd) Neither I nor II is stronge) Both I and II are strong

30. Statement: Should India have no military force at all?Arguments:I. No, other countries in the World do not believe in non-violence.II. Yes, many Indian believe in non-violence.

a) Only argument I is strongb) Only argument II is strongc) Either I or II is strongd) Neither I nor II is stronge) Both I and II are strong

Page 229: Reasoning Made Easy - pdfstores.files.wordpress.com · REASONING MADE EASY 6 Geetika, Jasmine, Hema, Brinda, Ipsita and veena, once again not following any order. The following are

REASONING MADE EASY www.BankExamsToday.com

229

Directions (Q. 31-35): Each question given below consists of a statement, followedby three or four arguments numbered I, II, III and IV. You have to decide which ofthe arguments is/are strong arguments and which is/are weak arguments andaccordingly choose your answer from the alternatives given below each question.

31. Statement: Should people with educational qualification higher than theoptimum requirements be debarred from seking jobs?Arguments:I. No, it will further aggrevate the problem of educated

unemployment.II. Yes, it creates complexes among employees and affects the work

adversely.III. No, this goes against the basic rights of the individuals.IV. Yes, this will increase productivity.

a) Only argument I is strongb) Only argument II is strongc) Either I or II is strongd) Neither I nor II is stronge) Both I and II are strong

32. Statement: Should India go in for computerization in all possible sectors?Arguments:I. Yes, it will bring efficiency and accuracy in the work.II. No, it will be an injustice to the monumental human resources

which are at present underutilized.III. No, computerization demands a lot of money. We should not waste

money on it.IV. Yes, when advanced countries are introducing computers in every

field, how can India afford to lag behind?a) Only argument I is strongb) Only argument II is strongc) Either I or II is strongd) Neither I nor II is stronge) Both I and II are strong

33. Statement: Should all the school teachers be debarred from giving privatetuitions?Arguments:I. No, the needy students will be deprived of the expertise of these

teachers.II. Yes, this is an injustice to the unemployed educated people who

can earn their living by giving tuitions.III. Yes, only then the quality of teaching in schools will improve.IV. Yes, now salary of these teachers is reasonable.

a) Only argument I is strongb) Only argument II is strong

Page 230: Reasoning Made Easy - pdfstores.files.wordpress.com · REASONING MADE EASY 6 Geetika, Jasmine, Hema, Brinda, Ipsita and veena, once again not following any order. The following are

REASONING MADE EASY www.BankExamsToday.com

230

c) Either I or II is strongd) Neither I nor II is stronge) Both I and II are strong

34. Statement: Should education be made compulsory for all children upto theage of 14?Arguments:I. Yes, this will help to eradicate the system of forced employment of

these children.II. Yes, this is an effective way to make the entire population

educated.III. No, we do not have adequate infrastructure to educate the entire

population.IV. Yes, this would increase the standard of living.

a) Only argument I is strongb) Only argument II is strongc) Either I or II is strongd) Neither I nor II is stronge) Both I and II are strong

35. Statement: Should trade unions be banned completely?Arguments:I. Yes, workers can concentrate on production.II. No, this is the only way through which employees can put their

demands before the management.III. Yes, employees get their illegal demands fulfilled through these

unions.IV. No, trade unions are not banned in other economically advanced

countries.a) Only argument I is strongb) Only argument II is strongc) Either I or II is strongd) Neither I nor II is stronge) Both I and II are strong

Answers:

1. Option AClearly, India can export only the surplus and that which can be saved afterfulfilling its own needs, to pay for its imports. Encouragement to exportcannot lead to shortages as it shall provide the resources for imports. So,only argument I holds.

2. Option A

Page 231: Reasoning Made Easy - pdfstores.files.wordpress.com · REASONING MADE EASY 6 Geetika, Jasmine, Hema, Brinda, Ipsita and veena, once again not following any order. The following are

REASONING MADE EASY www.BankExamsToday.com

231

Clearly, health of the citizens is an issue of major concern for theGovernment. So, a product like drugs, must be first studied and tested in theIndian context before giving licence for its sale. So, only argument I holdsstrong.

3. Option A

4. Option EThe students union formation shall be a step towards giving to students thebasic education in the field of politics. However, it shall create the samepolitical atmosphere in the campus. Thus, both the arguments hold strong.

5. Option AIndia cannot part with a State that is a major foreign exchange earner to it.So, argument I holds strong. Further, giving away a piece of landunconditionally and unreasonably is no solution to settle disputes. So,argument II is vague.

6. Option DThe age of a person is no criterion for judging his mental capabilities andadministrative qualities. So, none of the arguments holds strong.

7. Option COpening up of new industries is advantageous in opening more employmentavenues, and disadvantageous in that it adds to the pollution. So, either ofthe arguments holds strong.

8. Option APollution at ground level is the most hazardous in the way of beinginjurious to human and animal life. So, argument I alone holds.

9. Option ABefore indulging in new development programme it is much necessary toplan the exact target, policies and their implementation and the allocation offunds which shows the right direction to work. So, argument I holds strong.Also, planning ensures full utilization of available resources and funds andstepwise approach towards the target. So, spending a part of money on it isno wastage. Thus, argument II is not valid.

10. Option BI does not provide a strong reason in support of the statement. Also, it is notpossible to analyze the reaally deserving and not deserving. So, argument IIholds strong.

11. Option B

Page 232: Reasoning Made Easy - pdfstores.files.wordpress.com · REASONING MADE EASY 6 Geetika, Jasmine, Hema, Brinda, Ipsita and veena, once again not following any order. The following are

REASONING MADE EASY www.BankExamsToday.com

232

The demolition of unauthorized buildings would teach a lesson to theunscrupulous builders and also serve as a warning for the citizens not toindulge in such activities in the future. This is essential, as unauthorizedconstructions impose undue burden on the city’s infrastructure. So, onlyargument II holds strong.

12. Option BThere should be some norms regarding the number of ministers in theGovernment, as more number of ministers would unnecessarily add to theGovernment expenditure. So, argument II holds strong. Also, giving libertyto the party in power could promote extension of unreasonable favour tosome people at the cost of Government funds. So, argument I does not hold.

13. Option DForeign films depict the alien culture but this only helps in learning more.So, argument I does not hold. Also, the reason stated in argument II is notstrong enough in contradicting the ban. So, it also does not hold.

14. Option DIn buying things on installments, a customer has to pay more as the interestis also included. So, argument I does not hold. Moreover, one who buys anitem on installments maintains his future budget accordingly as he is wellacquainted with when and how much he has to pay, beforehand. So,argument II is also not valid.

15. Option A16. Option A

Government jobs in rural areas are underlined with several difficulties. Inlieu of these, extra incentives are needed. So, only argument I holds strong.

17. Option EIf there were less candidates, the voters would find it easy to make a choice.So, argument I holds. Also, every person satisfying the conditions laiddown by the Constitution must be given an opportunity and should not bedenied the same just to cut down the number of candidates. So, argument IIalso holds strong.

18. Option AThe advertisements are/the means to introduce people with the produce andits advantages. So, arguments I holds strong. But argument II is vaguebecause advertisements are an investment for better gain and not a wastage.

19. Option AThe legislators should surely not be engaged in any other professionbecause only then will they be able to work with devotion. So, argument Iholds. Also, if such a law is enforced, only those people will contest

Page 233: Reasoning Made Easy - pdfstores.files.wordpress.com · REASONING MADE EASY 6 Geetika, Jasmine, Hema, Brinda, Ipsita and veena, once again not following any order. The following are

REASONING MADE EASY www.BankExamsToday.com

233

elections who are really preparred to work for the country. So, argument IIis vague.

20. Option BNow a days, computers have entered all walks of life and children need tobe prepared for the same. So, argument II is strong. Argument I holds norelevance.

21. Option CReligion binds people togethe through the name of God and human values.But at the same time it may create differences and ill-will people, So, eitherof the arguments holds strong.

22. Option AA peace loving nation like India can well join an international forum whichseeks to bring different nations on friendly terms with each other. So,argument I holds strong. Argument II highlights a different aspect. Theinternal peoblems of a nation should not debar it from strengtheninginternational ties. So, argument II is vague.

23. Option BImposing ban on fashionable dresses will be a restriction on the pesonalchoice and hence the right to freedom of an individual. So, only argument IIis strong.

24. Option BAn organization like UNO is meant to maintain peace all over and willalways serve to prevent conflicts between countries. So, its role never ends.So, argument I does not hold. Also, lack of such an organization may infuture lead to increased mutual conflicts and international wars, on accountof lack of a common platform for mutual discussions. So, argument IIholds.

25. Option ABesides interview, there can be other modes of written examination to judgecandidates motives. So argument II is not strong enough. However, theinterview is a subjective assessment without doubt. So, argument I holds.

26. Option DTaking care of the parents is a moral duty of the children and cannot bethrust upon them legally, nor such a compulsion can ensure good care of theold people, So, none of the arguments holds strong.

27. Option DThe Government has already made provisions for reservation of jobs for theeconomically backward sections, which is a must. So, abolishing thepractice of reservation altogether has no meaning. Thus, argument I is

Page 234: Reasoning Made Easy - pdfstores.files.wordpress.com · REASONING MADE EASY 6 Geetika, Jasmine, Hema, Brinda, Ipsita and veena, once again not following any order. The following are

REASONING MADE EASY www.BankExamsToday.com

234

vague. Also, more reservations would lead to non-recruitment of manymore deserving candidates. Besides, such a reservation, if implemented,will cater to the job requirements of only a small section of population andnot a major part of it. So, argument II also does not hold strong.

28. Option AHigher education is not the cause of unemployment. Infact, it has createdgreater job opportunities. So, argument II is vague. Also, higher educationpromotes the country’s development. So, argument I holds.

29. Option DThe Public Distribution System is indeed necessary to provide basicamenities to the economically backward sections of population. So,argument I is vague. Also, if the objectives, of a system are not fulfilledbecause of corruption, then getting rid of the system is no solution. Instead,efforts should be made to end corruption and extend its benefits to thepeople for whom it is meant. So, argument II also does not hold.

30. Option DIndia needs to have military force to defend itself against the threat of othermilitary powers in the World. So, none of the arguments holds strong.

31. Option DThe issue discussed in the statement is nowhere related to increase inunemployment, as the number of vacancies filled in will remain the same.Also, in a working place, it is the peformance of the individual that mattersand that makes him more or less wanted, and not his educationalqualifications. So, neither I nor II holds strong. Besides, the needs of a jobare laid down in the desired qualifications for the job. So, recruitment ofmore qualified people cannot augment productivity. Thus, IV also does nothold strong. However, it is the right of an individual to get the post forwhich he fulfils the eligibility criteria, whatever be his extra merits. Hence,argument III holds strong.

32. Option AThe need of today is to put to better use the underutilized human resources.Computers with better and speedy efficiency can accomplish this. So,argument I holds, while II does not. Computerization is a much beneficialproject and investment in it is not at all a waste. So, III is not strong.Further, development in a new field is not a matter of merely following upother countries. So, IV also does not hold strong.

33. Option EOnly III is strong. The lure of earning private tuitions reduces the effortsand devotion of the teachers towards the students in schools. So, if tuitionsare banned, students can benefit from their teachers knowledge in the

Page 235: Reasoning Made Easy - pdfstores.files.wordpress.com · REASONING MADE EASY 6 Geetika, Jasmine, Hema, Brinda, Ipsita and veena, once again not following any order. The following are

REASONING MADE EASY www.BankExamsToday.com

235

school itself. So, argument III holds strong while I does not. However, aperson cannot be barred from earning more just because he already has agood salary. So, argument IV is vague. Further, the unemployed peoplethriving on tuitions can survive with the school teachers holding tuitionstoo, if they are capable enough to guide the students well. So, argument IIalso does not hold strong.

34. Option DToday’s children are to make up future citizens of the country and so it isabsolutely essential to make them learned, more responsible, moreinnovative and self-dependent by imparting them education. So, argumentII holds strong while I and IV do not. Besides, the goal of literacy cannot bedenied for want of infrastructure. So, argument III also does not hold.

35. Option BTrade unions provide a common platform for the workers to voice theirdemands and protests and thus ensure that they are not subdued orexploited. So, argument II holds strong, while I and III do not, besides, theidea of imitation of other countries in the implementation of a certain policyholds no relevance. So, argument IV also does not hold strong.

Page 236: Reasoning Made Easy - pdfstores.files.wordpress.com · REASONING MADE EASY 6 Geetika, Jasmine, Hema, Brinda, Ipsita and veena, once again not following any order. The following are

REASONING MADE EASY www.BankExamsToday.com

236

Chapter – 21

Statement and Assumptions

Directions (Q. 1-5): In each question below is given a statement followed by twoassumptions numbered I and II. You have to consider the statement and thefollowing assumptions and decide which of the assumptions is implicit in thestatement.

Give answer:a) If only assumption I is implicitb) If only assumption II is implicitc) If either I or II is implicitd) If neither I nor II is implicite) If both I and II are implicit

1. Statement: You are hereby appointed as a programmer with a probationperiod of one year and your performance will be reviewed at the end of theperiod for confirmation.Assumptions:I. The performance of an individual generally is not known at the

time of appointment offer.II. Generally an individual tries to prove his worth in the probation

period.a) Only assumption I is implicitb) Only assumption II is implicitc) Either I or II is implicitd) Neither I nor II is implicite) Both I and II are implicit

2. Statement: It is desirable to put the child in school at the age of 5 or so.Assumptions:I. At that age the child reaches appropriate level of development and

is ready to learn.II. The schools do not admit children after six years of age.

a) Only assumption I is implicitb) Only assumption II is implicitc) Either I or II is implicitd) Neither I nor II is implicita) Both I and II are implicit

3. Statement: In order to bring punctuality in our office, we must provideconveyance allowance to our employees. In charge of a company tellsPersonnel Manager.Assumptions:

Page 237: Reasoning Made Easy - pdfstores.files.wordpress.com · REASONING MADE EASY 6 Geetika, Jasmine, Hema, Brinda, Ipsita and veena, once again not following any order. The following are

REASONING MADE EASY www.BankExamsToday.com

237

I. Conveyance allowance will not help in bringing punctuality.II. Discipline and reward should always go hand in hand.

a) Only assumption I is implicitb) Only assumption II is implicitc) Either I or II is implicitd) Neither I nor II is implicita) Both I and II are implicit

4. Statement: Unemployment allowance should be given to all unemployedIndian youth above 18 years of age.Assumptions:I. There are unemployed youth in India who needs monetary support.II. The Government has sufficient funds to provide allowance to all

unemployed youth.a) Only assumption I is implicitb) Only assumption II is implicitc) Either I or II is implicitd) Neither I nor II is implicita) Both I and II are implicit

5. Statement: If you trouble me, I will slap you. A mother warns her child.Assumptions:I. With the warning, the child may stop troubling her.II. All children are basically naughty.

a) Only assumption I is implicitb) Only assumption II is implicitc) Either I or II is implicitd) Neither I nor II is implicita) Both I and II are implicit

Directions (Q. 6-10): In each question below is given a statement followed by twoassumptions numbered I and II. You have to consider the statement and thefollowing assumptions and decide which of the assumptions is implicit in thestatement.

Give answer:a) If only assumption I is implicitb) If only assumption II is implicitc) If either I or II is implicitd) If neither I nor II is implicite) If both I and II are implicit

6. Statement: The Government has decided to levy 2% on the tax amountpayable for funding drought relief programmes.Assumptions:

Page 238: Reasoning Made Easy - pdfstores.files.wordpress.com · REASONING MADE EASY 6 Geetika, Jasmine, Hema, Brinda, Ipsita and veena, once again not following any order. The following are

REASONING MADE EASY www.BankExamsToday.com

238

I. The Government does not have sufficient money to fund droughtrelief programmes.

II. The amount collected by way of surcharge may be adequate to fundthe drought relief programes.a) Only assumption I is implicitb) Only assumption II is implicitc) Either I or II is implicitd) Neither I nor II is implicite) Both I and II are implicit

7. Statement: Detergents should be used to clean clothes.Assumptions:I. Detergents form more lather.II. Detergents help to dislodge grease and dirt.

a) Only assumption I is implicitb) Only assumption II is implicitc) Either I or II is implicitd) Neither I nor II is implicite) Both I and II are implicit

8. Statement: It will be a substantial achievement in the field of education ifone provides one school for every village in our country and enforceattendance.Assumptions:I. Children in villages do not attend school regularly.II. Providing school to every village is desirable.

a) Only assumption I is implicitb) Only assumption II is implicitc) Either I or II is implicitd) Neither I nor II is implicite) Both I and II are implicit

9. Statement: The Government has decided to disinvest large chunk of itsequity in select public sector undertaking for a better fiscal management.Assumptions:I. The amount generated out of the disinvestment process may reduce

substantially the mounting fiscal deficits.II. There will be enough demand in the market for the shares of these

undertakings.a) Only assumption I is implicitb) Only assumption II is implicitc) Either I or II is implicitd) Neither I nor II is implicite) Both I and II are implicit

10. Statement: Never before such a lucid book was available on the topic.

Page 239: Reasoning Made Easy - pdfstores.files.wordpress.com · REASONING MADE EASY 6 Geetika, Jasmine, Hema, Brinda, Ipsita and veena, once again not following any order. The following are

REASONING MADE EASY www.BankExamsToday.com

239

Assumptions:I. Some other books were available onthis topic.II. You can write lucid books on very few topics.

a) Only assumption I is implicitb) Only assumption II is implicitc) Either I or II is implicitd) Neither I nor II is implicite) Both I and II are implicit

Directions (Q. 11-15): In each question below is given a statement followed by twoassumptions numbered I and II. You have to consider the statement and thefollowing assumptions and decide which of the assumptions is implicit in thestatement.

Give answer:a) If only assumption I is implicitb) If only assumption II is implicitc) If either I or II is implicitd) If neither I nor II is implicite) If both I and II are implicit

11. Statement: The Government has decided to pay compensation to the tuneof Rs.1 Lakh to the family membes of those who are killed in railwayaccidents.Assumptions:I. The Government has enough funds to meet the expenses due to

compensation.II. There may be reduction in incidents of railway accidents in near

future.a) Only assumption I is implicitb) Only assumption II is implicitc) Either I or II is implicitd) Neither I nor II is implicite) Both I and II are implicit

12. Statement: Films have become indispensable for the entertainment ofpeople.Assumptions:I. Films are the only media of entertainment.II. People enjoy films.

a) Only assumption I is implicitb) Only assumption II is implicitc) Either I or II is implicitd) Neither I nor II is implicite) Both I and II are implicit

Page 240: Reasoning Made Easy - pdfstores.files.wordpress.com · REASONING MADE EASY 6 Geetika, Jasmine, Hema, Brinda, Ipsita and veena, once again not following any order. The following are

REASONING MADE EASY www.BankExamsToday.com

240

13. Statement: Of all the newspapers published in Mumbai, readership of the“Times” is the largest in the Metropolis.Assumptions:I. ‘Times’ is not popular in mofussil areas.II. ‘Times’ has the popular feature of cartoons on burning social and

political issues.a) Only assumption I is implicitb) Only assumption II is implicitc) Either I or II is implicitd) Neither I nor II is implicite) Both I and II are implicit

14. Statement: Apart from the entertainment value of television, itseducational value cannot be ignored.Assumptions:I. People take television to be a means of entertainment only.II. The educational value of television is not realised properly.

a) Only assumption I is implicitb) Only assumption II is implicitc) Either I or II is implicitd) Neither I nor II is implicite) Both I and II are implicit

15. Statement: Children are influenced more by their teachers nowadays.Assumptions:I. The children consider teachers as their models.II. A large amount of children’s time is spent in school.

a) Only assumption I is implicitb) Only assumption II is implicitc) Either I or II is implicitd) Neither I nor II is implicite) Both I and II are implicit

Directions (Q. 16-20): In each question below is given a statement followed by twoassumptions numbered I and II. You have to consider the statement and thefollowing assumptions and decide which of the assumptions is implicit in thestatement.

Give answer:a) If only assumption I is implicitb) If only assumption II is implicitc) If either I or II is implicitd) If neither I nor II is implicite) If both I and II are implicit

Page 241: Reasoning Made Easy - pdfstores.files.wordpress.com · REASONING MADE EASY 6 Geetika, Jasmine, Hema, Brinda, Ipsita and veena, once again not following any order. The following are

REASONING MADE EASY www.BankExamsToday.com

241

16. Statement: A’s advice to B – “Go to Jammu via Amritsar – The shortestroute”.Assumptions:I. B wishes to go to Jammu.II. A gives advice to everybody.

a) Only assumption I is implicitb) Only assumption II is implicitc) Either I or II is implicitd) Neither I nor II is implicite) Both I and II are implicit

17. Statement: All existing inequalities can be reduced, if not utterlyeradicated, action of governments or by revolutionary change ofgovernment.Assumptions:I. Inequality is a manmade phenomenon.II. No person would voluntarily part with what he possesses.

a) Only assumption I is implicitb) Only assumption II is implicitc) Either I or II is implicitd) Neither I nor II is implicite) Both I and II are implicit

18. Statement: The campaign of ‘Keep Your City Clean’ started by the CivilCouncil did not evoke any response from the citizens.Assumptions:I. People do not desire to keep their city clean.II. The Civil Council has failed in its campaign.

a) Only assumption I is implicitb) Only assumption II is implicitc) Either I or II is implicitd) Neither I nor II is implicite) Both I and II are implicit

19. Statement: The district administration has issued a circular to all thefarmers under its jurisdiction advising them for not using pesticidesindiscriminately as it may pollute the ground water.Assumptions:I. People may stop using ground water if the farmers continue to use

pesticides indiscriminately.II. Farmers may refrain from using pesticides indiscriminately.

a) Only assumption I is implicitb) Only assumption II is implicitc) Either I or II is implicitd) Neither I nor II is implicite) Both I and II are implicit

Page 242: Reasoning Made Easy - pdfstores.files.wordpress.com · REASONING MADE EASY 6 Geetika, Jasmine, Hema, Brinda, Ipsita and veena, once again not following any order. The following are

REASONING MADE EASY www.BankExamsToday.com

242

20. Statement: The coffee powder of company X is quite better in taste thanthe much advertised coffee of company Y.Assumptions:I. If your product is not good, your spend more on advertisement.II. Some people are tempted to buy a product by the advertisement.

a) Only assumption I is implicitb) Only assumption II is implicitc) Either I or II is implicitd) Neither I nor II is implicite) Both I and II are implicit

Directions (Q. 21-25): In each question below is given a statement followed by twoassumptions numbered I and II. You have to consider the statement and thefollowing assumptions and decide which of the assumptions is implicit in thestatement.

Give answer:a) If only assumption I is implicitb) If only assumption II is implicitc) If either I or II is implicitd) If neither I nor II is implicite) If both I and II are implicit

21. Statement: Because of the large number of potholes in road X, reachingairport in time has become difficult.Assumptions:I. Reaching airport in time may not be always necessary.II. There is no other convenient road to the airport.

a) Only assumption I is implicitb) Only assumption II is implicitc) Either I or II is implicitd) Neither I nor II is implicite) Both I and II are implicit

22. Statement: Safety and health practices in many Indian companies are wellbelow the international standards.Assumptions:I. International standards of health and safety are ideal and unrealistic.II. Indian organizations do not consider safety and health management

as their prime social responsibility.a) Only assumption I is implicitb) Only assumption II is implicitc) Either I or II is implicitd) Neither I nor II is implicite) Both I and II are implicit

Page 243: Reasoning Made Easy - pdfstores.files.wordpress.com · REASONING MADE EASY 6 Geetika, Jasmine, Hema, Brinda, Ipsita and veena, once again not following any order. The following are

REASONING MADE EASY www.BankExamsToday.com

243

23. Statement: Greater public participation results in good civic governance.Statement of Municipal Commissioner of City A.Assumptions:I. The municipal office is not competent to effect good civic

administration.II. Good civic governance is a matter of collective will and effort of

the people and administration.a) Only assumption I is implicitb) Only assumption II is implicitc) Either I or II is implicitd) Neither I nor II is implicite) Both I and II are implicit

24. Statement: The regulatory authority has set up a review committee tofindout the reasons for unstable stock prices.Assumptions:I. The investors may regain confidence in stock market by this

decision.II. The review committee has the expertise to find out the causes for

volatility in the stock market.a) Only assumption I is implicitb) Only assumption II is implicitc) Either I or II is implicitd) Neither I nor II is implicite) Both I and II are implicit

25. Statement: Please note that the company will provide accommodation toonly outside candidates if selected. A condition in an advertisement.Assumptions:I. The local candidates would be having some other arrangement for

their stay.II. The company plans to select only local candidates.

a) Only assumption I is implicitb) Only assumption II is implicitc) Either I or II is implicitd) Neither I nor II is implicite) Both I and II are implicit

Directions (Q. 26-30): In each question below is given a statement followed by twoassumptions numbered I and II. You have to consider the statement and thefollowing assumptions and decide which of the assumptions is implicit in thestatement.

Give answer:a) If only assumption I is implicit

Page 244: Reasoning Made Easy - pdfstores.files.wordpress.com · REASONING MADE EASY 6 Geetika, Jasmine, Hema, Brinda, Ipsita and veena, once again not following any order. The following are

REASONING MADE EASY www.BankExamsToday.com

244

b) If only assumption II is implicitc) If either I or II is implicitd) If neither I nor II is implicite) If both I and II are implicit

26. Statement: Many historians have done more harm than good by distortingtruth.Assumptions:I. People believe what is reported by the historians.II. Historians are seldom expected to depict the truth.

In each question below is given a statement followed by two assumptions numberedI and II. You have to consider the statement and the following assumptions anddecide which of the assumptions is implicit in the statement.

Give answer:a) If only assumption I is implicitb) If only assumption II is implicitc) If either I or II is implicitd) If neither I nor II is implicite) If both I and II are implicit

27. Statement: As there is a great demand, every person seeking tickets of theprogramme will be given five tickets.Assumptions:I. The organizers are not keen on selling the tickets.II. No one is interested in getting more than five tickets.

a) Only assumption I is implicitb) Only assumption II is implicitc) Either I or II is implicitd) Neither I nor II is implicite) Both I and II are implicit

28. Statement” Computer education should start at schools itself.Assummptions:I. Learning computers is easy.II. Computer education fetches jobs easily.

a) Only assumption I is implicitb) Only assumption II is implicitc) Either I or II is implicitd) Neither I nor II is implicite) Both I and II are implicit

29. Statement: If he is intellegent, he will pass the examination.Assumptions:I. To pass, he must be intelligent.II. He will pass the examination.

Page 245: Reasoning Made Easy - pdfstores.files.wordpress.com · REASONING MADE EASY 6 Geetika, Jasmine, Hema, Brinda, Ipsita and veena, once again not following any order. The following are

REASONING MADE EASY www.BankExamsToday.com

245

a) Only assumption I is implicitb) Only assumption II is implicitc) Either I or II is implicitd) Neither I nor II is implicite) Both I and II are implicit

30. Statement: Today I must satisfy myself only by looking at a pink headedduck in an encyclopaedia.Assumptions:I. Pink headed ducks are as good as extinct now.II. People refer to encyclopaedia to know only about things which do

not exist now.a) Only assumption I is implicitb) Only assumption II is implicitc) Either I or II is implicitd) Neither I nor II is implicite) Both I and II are implicit

Answers:

1. Option EThe performance of the individual has to be tested over a span of time asthe statement mentions. So, I is implicit. The statement mentions that theindividual’s worth shall be reviewed before confirmation. So, II is alsoimplicit.

2. Option ASince the statement talks of putting the child in school at the age of 5, itmeans that the child is mentally prepared for the same at this age. So, I isimplicit, But nothing about admission after 6 years of age is mentioned inthe statement. So, II is not implicit.

3. Option BAssumption I goes against the statement. So, it is not implicit. Theallowance will serve as a reward to the employees and shall provoke themto come on time. So, II is implicit.

4. Option AI directly follows from the statement and so is implicit. Also, the statementis a suggestion and does not tell about a Government policy or its positionof funds. So, II is not implicit.

5. Option AThe mothe warns her child with the expectation that he would stoptroubling her. So, I is implicit. The general nature of children cannot bederived from the statement. So, II is not implicit.

Page 246: Reasoning Made Easy - pdfstores.files.wordpress.com · REASONING MADE EASY 6 Geetika, Jasmine, Hema, Brinda, Ipsita and veena, once again not following any order. The following are

REASONING MADE EASY www.BankExamsToday.com

246

6. Option ESince a surcharge has been levied to fund drought relief programmes, itfollow the Government does not have sufficient money for the same. So, Iis implicit. Besides, the percentage of surcharge must have been decidedafter studying the expected inflow in relation to amount of funds required.So, II is also implicit.

7. Option BNothing is mentioned about lather formation by the detergent. So, I is notimplicit. Also, detergents should be used as they clean clothes better andmore easily. So, II is implicit.

8. Option EThe statement lays stress on enforcing attendance. This implies thatchildren in villages do not attend school regularly. So, I is implicit. Besides,the statement calls one school for every village a substantial achievement.So, II is also implicit.

9. Option AThe fact given in I directly follows from the phrase -------- for a better fiscalmanagement in the statement. So, I is implicit. However, the publicresponse to the new policy cannot be ascertained. So, II is not implicit.

10. Option AIt follows from the statement that books on this topic were available beforealso but they were not lucid. So, I is implicit. But a general comment as IIcannot be made from the given statement. So, II is not implicit.

11. Option AThe amount of compensation must have been decided keeping in mind themonetary position of the Government. So, I is implicit. However, nothingcan be said about the frequency of railway accidents in future. So, II is notimplicit.

12. Option B‘Films are indispensable’ does not mean that they are the only means ofentertainment. So, I is not implicit. II follows from the statement. So, it isimplicit.

13. Option DNeither the volume of readership of the ‘Times’ in areas other than theMetropolis nor the reason for its huge acclamation is evident from thestatement. So, neither I nor II is implicit.

14. Option E

Page 247: Reasoning Made Easy - pdfstores.files.wordpress.com · REASONING MADE EASY 6 Geetika, Jasmine, Hema, Brinda, Ipsita and veena, once again not following any order. The following are

REASONING MADE EASY www.BankExamsToday.com

247

The statement makes the first assumption clear though educational value isnot to be ignored. So, I is implicit. That the educational value must not beignored also shows that educational value is not realised properly. So, II isalso implicit.

15. Option AIt is because children consider teachers as their model that they are moreinfluenced by them. So, I is implicit. It is not necessary that the children areinfluenced by teachers because they spend much time in school. So, II is notimplicit.

16. Option AA has advised B the route to Jammu. This means that B wishes to go toJammu. So, I is implicit. The statement mentions only A’s advice to B. So,II is not implicit.

17. Option ASince inequality can be reduced, it means that it is not natural but created.So, I is implicit. Nothing is mentioned about people’s response. So, II is notimplicit.

18. Option EAccording to the statement, the campaign did not get any response fromcitizens. This means that people are not interested in keeping the city cleanand the campaign has failed. So, both I and II implicit.

19. Option BThe district administration has issued a circular to make the farmers awareof hazards that indiscriminate use of pesticides poses to ground water andplead them to refrain from the same. So, II is implicit. However, I cannot beassumed from the given statement and so it is not implicit.

20. Option B

21. Option BThe statement presents the issue of ‘not reaching airport in time’ as aproblem. This means that reaching airport in time is necessary. So, I is notimplicit. Besides, it is mentioned that reaching airport in time has becomedifficult due to large number of potholes in road X. This implies that road Xis the only possible way. So, II is implicit.

22. Option BThe statement talks about the safety and health practices in Indiancompanies being far below international standards. It is clearly a criticismof Indian organizations not paying considerable attention to these aspects.

Page 248: Reasoning Made Easy - pdfstores.files.wordpress.com · REASONING MADE EASY 6 Geetika, Jasmine, Hema, Brinda, Ipsita and veena, once again not following any order. The following are

REASONING MADE EASY www.BankExamsToday.com

248

So, II is implicit. The intenationalstandards demand perfection and are, inno way, non-achievable. So, I is not implicit.

23. Option BThe statement stresses on the fact that though civic governance is the taskof the municipal body, but all the tasks done come ou to be more fruitful ifthe general public lends a helping hand in the same. So, only II is implicit.

24. Option EI mentions the aim for which the step talked about in the statement, hasbeen undertaken while II mentions the essential requirement for it. So, bothI and II are implicit.

25. Option AThe statement mentions that the company intends to provideaccommodation only to outside candidates. This means that localcandidates would have to arrange accommodation on their own and that thecompany may select local as well as outside candidates. Thus, only I isimplicit.

26. Option AThe fact that historians have done harm by distorting truth, means thatpeople believe what is reported by the historians. So, I is implicit. II doesnot follow from the statement and is not implicit.

27. Option DThe organizers are adopting this policy not to reduce the sale but to cope upwith great demand so that everyone can get the ticket. So, I is not implicit.Also, due to great demand, the maximum number of tickets one person canget has been reduced to five So, II is also not implicit.

28. Option AComputer educabe started at the school level only if it is easy. So, I isimplicit. In the statement, nothing is mentioned about the link between jobsand computer education. So, II is not implicit.

29. Option AThe statement mentions that he will pass if he is intelligent. So, I is implicit.Further, this means that it is not necessary that he will pass. So, II is notimplicit.

30. Since the narrator talks ofsatisfying himself by just looking at a picture inencyclopaedia, it means that pink headed ducks are not to be seen alive. So,I is implicit. But II does not follow from the statement and is not implicit.

Page 249: Reasoning Made Easy - pdfstores.files.wordpress.com · REASONING MADE EASY 6 Geetika, Jasmine, Hema, Brinda, Ipsita and veena, once again not following any order. The following are

REASONING MADE EASY www.BankExamsToday.com

249

Chapter – 22

Statement and Conclusions

Directions (Q. 1-5): In each question below is given a statement followed by twoconclusions numbered I and II. You have to assume everything in the statement tobe true, then consider the two conclusions together and decide which of themlogically follows beyond a reasonable doubt from the information given in thestatement.

Give answer:a) If only conclusion I followsb) If only conclusion II followsc) If either I or II followsd) If neither I nor II follows ande) If both I and II follow

1. Statement: In a one day cricket match, the total runs made by a team were200. Out of these 160 runs were made by spinners.Conclusions:I. 80% of the team consists of spinners.II. The opening batsmen were spinners.

a) Only conclusion I followsb) Only conclusion II followsc) Either I or II followsd) Neither I nor II followse) Both I and II are follow

2. Statement: The old order changed place to new.Conclusions:I. Change is the law of nature.II. Discard old ideas because they are old.

a) Only conclusion I followsb) Only conclusion II followsc) Either I or II followsd) Neither I nor II followsa) Both I and II are follow

3. Statement: Government has spoiled many top ranking financial institutionsby appointing bureaucrats as Directors of these institutions.Conclusions:I. Government should appoint Directors of the financial institutes

taking into consideration the expertise of the person in the area offinance.

Page 250: Reasoning Made Easy - pdfstores.files.wordpress.com · REASONING MADE EASY 6 Geetika, Jasmine, Hema, Brinda, Ipsita and veena, once again not following any order. The following are

REASONING MADE EASY www.BankExamsToday.com

250

II. The Director of the financial institute should have expertisecommensurate with the financial work carried out by the institute.a) Only conclusion I followsb) Only conclusion II followsc) Either I or II followsd) Neither I nor II followsa) Both I and II are follow

4. Statement: Population increase coupled with depleting resources is goingto be the scenario of many developing countries in days to come.Conclusions:I. The population of developing countries will not continue to

increase in future.II. It will be very difficult for the governments of developing countries

to provide its people decent quality of life.a) Only conclusion I followsb) Only conclusion II followsc) Either I or II followsd) Neither I nor II followsa) Both I and II are follow

5. Statement: Prime age school going children in urban India have nowbecome avid as well as more regular viewers of television, even inhouseholds without a T.V. As a result, there has been an alarming decline inthe extent of readership of newspapers.Conclusions:I. Method of increasing the readership of newspapes should be

devised.II. A team of experts should be sent to other countries to study the

impact of T.V. on the readership of newspapers.a) Only conclusion I followsb) Only conclusion II followsc) Either I or II followsd) Neither I nor II followsa) Both I and II are follow

Directions (Q. 6-10): In each question below is given a statement followed by twoconclusions numbered I and II. You have to assume everything in the statement tobe true, then consider the two conclusions together and decide which of themlogically follows beyond a reasonable doubt from the information given in thestatement.

Give answer:a) If only conclusion I followsb) If only conclusion II followsc) If either I or II follows

Page 251: Reasoning Made Easy - pdfstores.files.wordpress.com · REASONING MADE EASY 6 Geetika, Jasmine, Hema, Brinda, Ipsita and veena, once again not following any order. The following are

REASONING MADE EASY www.BankExamsToday.com

251

d) If neither I nor II follows ande) If both I and II follow

6. Statement: The manager humiliated Sachin in the presence of hiscolleagues.Conclusions:I. The manager did not like Sachin.II. Sachin was not popular with his colleagues.

a) Only conclusion I followsb) Only conclusion II followsc) Either I or II followsd) Neither I nor II followsa) Both I and II are follow

7. Statement: Women’s organizations in India have welcomed theamendment of the Industrial Employment Rules 1946 to curb sexualharassment at the work place.Conclusions:I. Sexual harassment of women at work place is more prevalent in

India as compared to other developed countries.II. Many organizations in India will stop recruiting women to avoid

such problems.a) Only conclusion I followsb) Only conclusion II followsc) Either I or II followsd) Neither I nor II followsa) Both I and II are follow

8. Statement: Nation X faced growing international opposition for itsdecision to explode eight nuclear weapons at its test site.Conclusions:I. The citizens of the nation favoured the decision.II. Some powerful countries do not want other nations to become as

powerful as they are.a) Only conclusion I followsb) Only conclusion II followsc) Either I or II followsd) Neither I nor II followsa) Both I and II are follow

9. Statement: In a highly centralised power structure, in which even seniorcabinet ministers are prepared to reduce themselves to pathetic countries oryesmen airing views that are primarily intended to anticipate or reflect thePrime Minister’s own performances, there can be no place for anyconsensus that is quite different from real or contrived unanimity of

Page 252: Reasoning Made Easy - pdfstores.files.wordpress.com · REASONING MADE EASY 6 Geetika, Jasmine, Hema, Brinda, Ipsita and veena, once again not following any order. The following are

REASONING MADE EASY www.BankExamsToday.com

252

opinion, expressed through a well orchestrated endorsement of the leader’sactions.Conclusions:I. The Ministers play safe by not giving anti-government views.II. The Prime Minister does not encourage his colleagues to render

their own views.a) Only conclusion I followsb) Only conclusion II followsc) Either I or II followsd) Neither I nor II followsa) Both I and II are follow

10. Statement: National Aluminium Company has moved India from aposition of shortage to self sufficiency in the metal.Conclusions:I. Previously, India had to import aluminium.II. With this speed, it can soon become a foreign exchange earner.

a) Only conclusion I followsb) Only conclusion II followsc) Either I or II followsd) Neither I nor II followsa) Both I and II are follow

Directions (Q. 11-15): In each question below is given a statement followed by twoconclusions numbered I and II. You have to assume everything in the statement tobe true, then consider the two conclusions together and decide which of themlogically follows beyond a reasonable doubt from the information given in thestatement.

Give answer:a) If only conclusion I followsb) If only conclusion II followsc) If either I or II followsd) If neither I nor II follows ande) If both I and II follow

11. Statement: People who speak too much against dowry are those who hadtaken it themselves.Conclusions:I. It is easier said than done.II. People have double standards.

a) Only conclusion I followsb) Only conclusion II followsc) Either I or II followsd) Neither I nor II followsa) Both I and II are follow

Page 253: Reasoning Made Easy - pdfstores.files.wordpress.com · REASONING MADE EASY 6 Geetika, Jasmine, Hema, Brinda, Ipsita and veena, once again not following any order. The following are

REASONING MADE EASY www.BankExamsToday.com

253

12. Statement: The national norm is 100 beds per thousand populations but inthis state, 150 beds per thousand are available in the hospitals.Conclusions:I. Our national norm is appropriate.II. The state’s health system is taking adequate care in this regard.

a) Only conclusion I followsb) Only conclusion II followsc) Either I or II followsd) Neither I nor II followsa) Both I and II are follow

13. Statement: Our securities carry market risk. Consult your investmentadvisor or agent before investing.Conclusions:I. One should not invest in securities.II. The investment advisor calculates the market risk with certainty.

a) Only conclusion I followsb) Only conclusion II followsc) Either I or II followsd) Neither I nor II followsa) Both I and II are follow

14. Statement: Money plays a vital role in politics.Conclusions:I. The poor can never become politicians.II. All the rich men take part in politics.

a) Only conclusion I followsb) Only conclusion II followsc) Either I or II followsd) Neither I nor II followsa) Both I and II are follow

15. Statement: Vegetable prices are soaring in the market.Conclusions:I. Vegetables are becoming a rare commodity.II. People cannot eat vegetables.

a) Only conclusion I followsb) Only conclusion II followsc) Either I or II followsd) Neither I nor II followsa) Both I and II are follow

Directions (Q. 16-20): In each question below is given a statement followed by twoconclusions numbered I and II. You have to assume everything in the statement tobe true, then consider the two conclusions together and decide which of them

Page 254: Reasoning Made Easy - pdfstores.files.wordpress.com · REASONING MADE EASY 6 Geetika, Jasmine, Hema, Brinda, Ipsita and veena, once again not following any order. The following are

REASONING MADE EASY www.BankExamsToday.com

254

logically follows beyond a reasonable doubt from the information given in thestatement.

Give answer:a) If only conclusion I followsb) If only conclusion II followsc) If either I or II followsd) If neither I nor II follows ande) If both I and II follow

16. Statement: Any student who does not behave properly while in the schoolbrings bad name to himself and also for the school.Conclusions:I. Such student should be removed from the school.II. Stricter discipline does not improve behaviour of the students.

a) Only conclusion I followsb) Only conclusion II followsc) Either I or II followsd) Neither I nor II followsa) Both I and II are follow

17. Statement: A Corporate General Manager asked four managers to eithersubmit their resignations by the next day or face termination orders fromservice. Three of them had submitted their resignations by that evening.Conclusions:I. The next day, the remaining manager would also resign.II. The General Manager would terminate his services the next day.

a) Only conclusion I followsb) Only conclusion II followsc) Either I or II followsd) Neither I nor II followsa) Both I and II are follow

18. Statement: Only good singers are invited in the conference. No onewithout sweet voice is a good singer.Conclusions:I. All invited singers in the conference have sweet voice.II. Those singers who do not have sweet voice are not invited in the

conference.a) Only conclusion I followsb) Only conclusion II followsc) Either I or II followsd) Neither I nor II followse) Both I and II are follow

Page 255: Reasoning Made Easy - pdfstores.files.wordpress.com · REASONING MADE EASY 6 Geetika, Jasmine, Hema, Brinda, Ipsita and veena, once again not following any order. The following are

REASONING MADE EASY www.BankExamsToday.com

255

19. Statement: To cultivate interest in reading, the school has made itcompulsory from June this year for each student to read two books perweek and submit a weekly report on the books.Conclusions:I. Interest in reading can be created by force.II. Some students will eventually develop interest in reading.

a) Only conclusion I followsb) Only conclusion II followsc) Either I or II followsd) Neither I nor II followse) Both I and II are follow

20. Statement: Applications of applicants who do no fulfil eligibility criteriaand/or who do not submit applications before last date will be summarilyrejected and will not be called for the written test.Conclusions:I. Those who are called for the written test are those who fulfil

eligibility criteria and have submitted their applications before lastdate.

II. Written test will be held only after scrutiny of applications.a) Only conclusion I followsb) Only conclusion II followsc) Either I or II followsd) Neither I nor II followse) Both I and II are follow

Answers:

1. Option DAccording to the statement, 80% of the total runs were made by spinners.So, I does not follow. Nothing about the opening batsmen is mentioned inthe statement. So, II also does not follow.

2. Option AI directly follows from the given statement. Also, it is mentioned that oldideas are replaced by new ones, as thinking changes with the progressingtime. So, II does not follow.

3. Option EAccording to the statement, Government has spoiled financial institutionsby appointing bureacrats as Directors. This means that only those personsshould be appointed as Directors who are experts in finance and areacquainted with the financial work of the institute. So, both I and II follow.

4. Option B

Page 256: Reasoning Made Easy - pdfstores.files.wordpress.com · REASONING MADE EASY 6 Geetika, Jasmine, Hema, Brinda, Ipsita and veena, once again not following any order. The following are

REASONING MADE EASY www.BankExamsToday.com

256

The fact given in I is quite contrary to the given statement. So, I does notfollow. II mentions the direct implications of the state discussed in thestatement. Thus, II follows.

5. Option DThe statement concentrates on the increasing viewership of T.V. and doesnot stress either on increasing the readership of newspapers or makingstudies regarding the same. So, neither I nor II follows.

6. Option DThe manager might have humiliated Sachin not because of his dislike buton account of certain negligence or mistake on his part. So, I does notfollow. Also, nothing about Sachin’s rapport with his colleagues can bereduced from the statement. So, II also does not follow.

7. Option DThe fact that a certain rule has been more welcomed in a certain coutry doesnot imply that the problem is more prevalent there. So, I does not follow.Also, the amendment seeks to discourage only sexual harassment of womenand shall in no way discourage employment of women. So, II also does notfollow.

8. Option DNeither the citizens response to the decision nor the reason for oppositionby other nations can be deduced from the statement. So, neither I nor IIfollows.

9. Option AAccording to the statement, even senior cabinet ministers are always readyto conform to the Prime Minister’s views. So, I follows. However, IIcontradicts the given statement and so does not follow.

10. Option EAccording to the statement, National Aluminium Company has movedIndia from a position of shortage in the past to self sufficiency in thepresent. This means that previously, India had to import aluminium. So, Ifollows. Also, it can be deduced that if production increases at the samerate, India can export it in future. So, II also follows.

11. Option EThe statement implies that it is easier to say than to do something and whatpeople say is different from what they do. So, both I and II follow.

12. Option B

Page 257: Reasoning Made Easy - pdfstores.files.wordpress.com · REASONING MADE EASY 6 Geetika, Jasmine, Hema, Brinda, Ipsita and veena, once again not following any order. The following are

REASONING MADE EASY www.BankExamsToday.com

257

Whether the national norm is appropriate or not cannot be said. So, I doesnot follow. However, more numbe of beds per thousand population areavailable in the state. So, II follows.

13. Option BInvestment in securities involves risk. This does not mean that one shouldnot invest in securities. So, I does not follow. Since the statement advisesone to consult investment advisor before investing. So, II follows.

14. Option DNeither the poor nor the rich, but only the role of money in politics is beingtalked about in the statement. So, neither I nor II follows.

15. Option DThe availability of vegetables is not mentioned in the given statement. So, Idoes not follow. II is not directly related to the statement and so it also doesnot follow.

16. Option DI cannot be deduced from the statement, Also, nothing about discipline ismentioned in the statement. So, neither I nor II follows.

17. Option CIt is mentioned in the statement that either the managers should resign bythe next day or their services would be terminated. So, either I or II follows.

18. Option EThe statement asserts that a good singer always has a sweet voice and onlygood singers are invited in the conference. This implies that all thoseinvited in the conference have sweet voice and those who do not have sweetvoice are not invited. So, both I and II follow.

19. Option BThe new scheme intends to develop interest in reading by incorporating thehabit in their routine. So, only II follows while I does not.

20. Option EThe statement mentions that fulfilling the eligibility criteria and submittingthe application before the stipulated date are both essential to avoidrejection. So, I follows. Also, since it is given that the candidates whoseapplications are rejected shall not be called for written test, so II alsofollows.